Democratic Underground Latest Greatest Lobby Journals Search Options Help Login
Google

yes, dear souls, you CAN prove a negative. and you CAN be an atheist

Printer-friendly format Printer-friendly format
Printer-friendly format Email this thread to a friend
Printer-friendly format Bookmark this thread
This topic is archived.
Home » Discuss » Topic Forums » Religion/Theology Donate to DU
 
enki23 Donating Member (1000+ posts) Send PM | Profile | Ignore Sun Dec-12-04 05:17 PM
Original message
yes, dear souls, you CAN prove a negative. and you CAN be an atheist
Edited on Sun Dec-12-04 05:20 PM by enki23
Not just an agnostic!

I can prove I don't have a beard to anyone in the same room with me. It's very easy. In fact, it's exactly as easy as it is to prove I'm clean-shaven. Odd, that.

I might not be able to prove I didn't have an "invisible" beard (unless I let someone feel my face). Same goes for proving I don't have a beard to anyone *not* currently in the room with me. But that's because of limits on one's ability to observe the beard in question, not because of some special logic that applies to negative statements (or invisible things.)

In the same way, I can't prove there isn't a cosmic gumby currently hashing it out with pokey somewhere in the farthest reaches of the galaxy, or a gaseous, bearded, Jewish man who controls the entire universe and who wants my neighbors to stone me for having sex with a married woman. But the only reason I can't prove those don't exist has nothing to do with this silly "can't prove a negative" idea. It's because I can't observe everything about the entire universe, and so I can't "prove" YHVH doesn't exist because the god idea gives him so much wiggle room that one can always find a corner of reality, or imagined reality, in which he could be hiding. If, however, you actually made YHVH into something which could reasonably be expected to be observable, then you could prove it either way. If my observational abilities were great enough, any question, negative or no, could be answered.

(Note that I don't go so far as to say a statement has to be "falsifiable," to be "science" in some popper-esque sense. But that's another issue...)

And then there's the obvious bit. The one which says I have no reason, in fact, to believe he *does* exist. And so I assume, like I do for the infinite number of other imaginary things one might begin to catalogue, that he does not, until I see actual physical evidence in his favor. Any evidence, even by inference would be fine. But it should be less than completely ambiguous, if its to be called evidence in his favor. Till then, he lives with the gnomes, and the trolls, the spiritual salad tongs, Xenu, Thor, Hannibal Lecter, Mary Poppins, Pope Joan, and the flying, electric, black handled ice cream scoop of doom. None of which i can prove doesn't exist somewhere in the universe, and none of which I have any reason whatsoever to believe does.

Further, with the understanding we have of the universe so far, it truly appears NOT to be the product of an "intelligent" designer, much less be under the full (or even substantial) control of a cosmic puppeteer. Therefore, I am even more willing to say I believe, based on the evidence we have, that there is sufficient reason to disbelieve any of the god ideas i've ever heard is true. That's why i'm an atheist, rather than an agnostic. I realize that in some sense one has to admit to being agnostic about *everything* in life, but that by no means applies to a god any more than anything else. I'm also not certain any of you I'm writing at actually exists, if you really have to be stupid about it. But since radical solipsism makes for a shitty way of trying to deal with perceived reality, there's not much point in obsessing over it. Same thing for god.

I'm at least as sure there is no christian god (or anything remotely like him) as I am that Elvis not alive. Very few people try to make me feel guilty for saying "Elvis died" without adding "though I'm not absolutely certain" every time. Why do so many think I'm being irrational about being equally sure of the first? I'm not going to get all agnostically silly just for one imaginary entity, after all, or even a fuzzy class of them.
Printer Friendly | Permalink |  | Top
JitterbugPerfume Donating Member (1000+ posts) Send PM | Profile | Ignore Sun Dec-12-04 06:02 PM
Response to Original message
1. thanks enki
cool post!
Printer Friendly | Permalink |  | Top
 
Stunster Donating Member (984 posts) Send PM | Profile | Ignore Sun Dec-12-04 06:50 PM
Response to Original message
2. It depends on what you mean by 'observe'
Just because some people haven't had one, it doesn't follow that there's no such thing as an orgasm. People who haven't had one probably can't imagine what it's like. Same with the taste of Scotch. You have to experience certain things for oneself. But the way an orgasm feels or the way Scotch tastes are not visible, material objects.

And I'll grant you, God is not a visible, material object of any kind (and especially not an old man with a beard, etc). But, as a great man once said..., so what?

Can you observe a thought? A number? A moral obligation? An emotion?

Can you observe someone else's perception of color?

Can you observe a law of physics?

Can you observe the curvature of space? Can you observe an electron?

Come to think of it, I've never directly experienced an electron. But I have directly experienced God, twice, and both times quite unmistakeably.

Yet, the scientific materialist insists upon claiming that electrons are real and God isn't.

Well, sorry, mister scientific materialist, but I have got better grounds for believing in God than I have for believing in electrons. So take your scientific materialist ideas.... and see if you can adequately account for those ideas in purely materialist terms. Bet you can't!

As for inferences---not that I need to infer the existence of God, since I've experienced God---consider the following:

You are on a train from London to Glasgow. Just after Carlisle, you see a sign that says WELCOME TO SCOTLAND. Is your immediate inclination to think that the sign being there is best explained by i) purposeful, conscious intelligence, or ii) by sheer random chance movement of atoms; or iii) by some impersonal cosmic law or computer code that inexorably dictates that the sign will be there?

Now compare the WELCOME TO SCOTLAND sign with the entire physical universe, with all its laws and all its principal features. What is more ordered and complex---the sign, or the universe?

Personally, I think the answers to both questions are obvious. And so is the inference that to be drawn therefrom---that the creation of the universe was an act of purposive intelligence. You're free to disagree of course. Just as I'm free to disagree with you

Printer Friendly | Permalink |  | Top
 
trotsky Donating Member (1000+ posts) Send PM | Profile | Ignore Sun Dec-12-04 08:55 PM
Response to Reply #2
8. A question regarding this.
What about those people who have tried - with sincerety - to experience "god" and don't?

Much like some people are physiologically incapable of having an orgasm, are some people "hard-wired" so that they are unable to experience "god"?

I was raised a Christian, prayed a lot, listened a lot, tried to feel SOMETHING, but never did. I gave "god" plenty of opportunities to reach out to me, confirm my belief, but never got a response.

Now either you can be an arrogant bastard and say that "god" really DID respond and I was too stupid/bullheaded/etc. to listen, or you can admit that some people are unable to experience "god," which to me seems to be a serious design flaw of creation.

Which shall it be? Or is there a third option?

(By the way, your "Welcome to Scotland" sign is just a rehash of the Watchmaker's Argument - feel free look it up to see the various problems with it.)
Printer Friendly | Permalink |  | Top
 
Stunster Donating Member (984 posts) Send PM | Profile | Ignore Mon Dec-13-04 02:03 AM
Response to Reply #8
10. I don't think you can experience God...
... by trying to experience God.

The great Protestant theologian, Karl Barth, would have a fit about such a notion--and for good reasons, in my opinion.

This is not a systematic answer, but a few unsystematic thoughts... and they may not apply to you in any way, shape, or form.

One is that I do not think that 'experiencing God' is a 'right' that we have.

I suspect that God chooses not to grant any extraordinarily direct experiences of God's reality to many people, because God doesn't want to manipulate us. Love and manipulation are incompatibles. That goes both ways.

God is not under our control. Nor does God choose to control us. God grants us autonomy. Given these facts, our experiencing God in any extraordinary way is not guaranteed, or guaranteeable.

That leaves open the possibility of experiencing God in the ordinary--and hence, doubtable---ways. But those ways are still good ways.

I think a number of writers on contemplative prayer have a lot of insight into your question. There are many difficulties and potential pitfalls in the human search for God---and often they're there for a good reason. One is that there's a danger that we come to love, not God, but the way God makes us feel.

Does a man love a woman if he only 'loves' her on condition that she makes him feel good?

"God, I will not believe in you, love you, or serve you, unless you give me the wonderful mystical experiences of you that I am now demanding of you." And God said, "WTF?"

Ask not what God can do for you. Rather, ask what you can do for God. People who only ask the former question often complain that God isn't doing anything for them--that God seems completely absent. People who ask the latter question often marvel at what God does for them.

I'm reminded of the story about the man who was a devout Christian, and who heard that a hurricane was on the way. He said, I'll be ok. God will protect me. I put all my trust in God. Then the hurricane came and destroyed his house completely, he lost everything, and he was badly injured. He asked God, why didn't you protect me? God replied---didn't you get the hurricane warnings? The weather forecast? Didn't the police tell you to evacuate? Didn't your relatives tell you to get out of there? You ignored all of the protection I was trying to give you!

God sometimes works in ordinary ways, as well as mysterious ways.

I remember talking with a man who told me, considerably to my surprise and even mild discomfort, that he had once had a vision of Christ. I asked him why, in his opinion, God had granted him that vision of Christ. He laughed and said, "Probably because God knew I'd be talking to you about it now."

Does God owe me in this life a vision of Christ? No, I don't think so.

A lot of people are given to different types of experience, but not to other types. Some folk get their kicks in math. Others in art. Others in family life (not that these are exclusive). Maybe God likes diversity, and so arranges things that some people are contemplative mystics, others find God in family life, or or in their work helping others, or in great moral concerns, or in some other way. Some even find God in church. Some find God in the depths of their own sinfulness, or brokenness.

Maybe you just haven't found the best way for you to experience and come to know God----or rather, the best way for you to being open to receiving such experience, if God wants to give you an experience. But maybe God doesn't think it's important for you to have an experience of God. Maybe God thinks it's much more important that you learn to serve God in some way. Maybe you're already doing that, and so the experience bit isn't necessary for you.

I think people often don't experience God because there is nothing in reality answering to their concept of God. This doesn't mean that God doesn't exist. It means they need to ditch their lousy concept of God.

I don't know the answer in your case. But I'd suggest it's worth searching for God---not for 'an experience of God'. But for God. God is not our experience of God, any more than a cat is our experience of a cat.









Printer Friendly | Permalink |  | Top
 
trotsky Donating Member (1000+ posts) Send PM | Profile | Ignore Mon Dec-13-04 08:31 AM
Response to Reply #10
14. As I suspected.
Blame the seeker, not the god. You arrogantly assume that I must have done something wrong. Nothing new there, unfortunately. Don't you have something more than canned theological answers like "god works in mysterious ways"?

And can you reconcile this bigoted attitude with the outrage expressed by Christians here on DU (maybe even yourself) when atheists choose to express their non-belief?
Printer Friendly | Permalink |  | Top
 
Ando Donating Member (112 posts) Send PM | Profile | Ignore Tue Dec-14-04 11:48 AM
Response to Reply #14
31. Leap of Faith
The "Seeking of God" is a very difficult thing to try and standardize. A revelation of God is predicated by a true faith. I can try and seek God as an answer to my doubts or questions or preconceptions. In essence this is a scientific endeavour. "I'm going to try and seek God because I want to know once and for all if he exists. If he doesn't reveal himself to me, I'll have my answer." In the Christian sense, this essentially amounts to a testing of God. I guess my point is that revelation does not precede faith, belief, and application. Revelation is the result of a process grounded in faith, not the cause of a process grounded in faith. If proof is required for faith, then faith ceases to exist as a concept. "I don't believe because I can't hear, see, or prove what I don't believe." That's an odd statement isn't it? All of the harshest judgements uttered by Jesus were preceeded by a comment about lack of faith or unbelief. This is not the language of bigotry, it is the language of theology and the crux of these types of discussions. I'm not blaming the seeker, I'm just saying you weren't seeking what God was offering.
Printer Friendly | Permalink |  | Top
 
Ignoramus Donating Member (610 posts) Send PM | Profile | Ignore Tue Dec-14-04 12:17 PM
Response to Reply #31
32. Insanity is believing
The point where someone finally goes bonkers is when they actually believe their fantasy. Replace "God" with "the divine space lobster" in your statement. You just have to have faith that a divine space lobster has created the universe. People that operate on such a level are impossible to reason with, and potentially very dangerous (and yes the same goes for people who "believe" that such and such scientific dogma is true and unquestionable).

What if their faith says that the divine space lobster commands them to kill non-believers. Then it's not just a matter of allowing people to have their beliefs, their belief impacts all of us. That is the crux of the horrible situation that we are in right now, in the world!

There is something that feels healthy about what you say though. It feels similar to someone saying they will not accept that they are defeated, and they will have faith that they will pull through. But in that case they really mean "hope" not "faith". You can choose to give yourself courage toward some end and that is healthy. You can also choose to "trust" that something will be as you expect it to, and that can be healthy.

I trust that my incredibly strong feeling that it is correct to "love" people and incorrect to abuse and kill people. But, when you "believe" something, it means you have gone over the brink and are compulsive and suffering from insanity (and the side effects of that insanity depend on what the subject is of your insanity).
Printer Friendly | Permalink |  | Top
 
Ando Donating Member (112 posts) Send PM | Profile | Ignore Tue Dec-14-04 12:55 PM
Response to Reply #32
33. Revolutionary
That's a pretty strong statement! I don't buy the argument that belief = insanity. However, I'll choose not to argue it in an act of protest! :) I've noticed that the arguments in this forum tend to devolve into logic 101, I would like to see more discussions which assume a fundamental lack of logic. Hence my protest. I love logic and logic serves me well, but I believe logic reaches its limit when attempting to deal with the illogical. The threads on here which attempt to prove or disprove God via logic are fairly laughable. What's the use of a God who does not defy logic other than to further dumb-down the ideal?

I'm going to stick my head out onto the choping block here and present a position (it's not my own, I borrowed it from G.K. Chesterton) Let's talk about progressive and conservative (this is NOT politics, this is philosophy, please don't make a connection to politics as I don't intend to go there!). The TRUE conservative cherishes and values his white picket fence. In order to keep his white picket fence he must be in a constant state of revolution. In essence, to keep the old white fence he must constantly recreate the new white fence or it will turn gray, black, or brown. The TRUE progressive disregards the ideal, instead of valuing the white picket fence they constantly redefine what they value so as to never reach an ideal. I guess the crux of this argument is Chesterton's statement that "Christianity has not been tried and found difficult, it has been found difficult and left untried." Belief assumes an unchanging state, an ideal. There are those who live their lives so as to reach that ideal (those who you would call lunatics) and there are those who redefine the ideal to make themselves comfortable or happy. I'm happy to call myself a lunatic.

Everyone should read "Orthodoxy" by G.K. Chesterton, it is the best treatise on Christian philosophy I have ever read and it would make for some great discussions on this board, with or without that damned logic.
Printer Friendly | Permalink |  | Top
 
Ignoramus Donating Member (610 posts) Send PM | Profile | Ignore Tue Dec-14-04 02:18 PM
Response to Reply #33
37. worship is disrespect
Okay, so you vow ignorance. What am I to do when you come to kill my friends and family? I will defend myself as best as I can.

Meanwhile, I reach out to you to find a common ground. Without being able to use analogy and something like reason, we are lost. It's just my assertion vs. your assertion.

There's something there that I suspect from your words. Something like an interest in awareness of a way that things are, rather than just what's on your dinner plate. So, I feel like there should be some way to appeal to that.

You ask "what's the use of a God who does not defy logic other than to further dumb-down the ideal"? I'm just asking you to allow that you might not know what the ideal is. If you can do that, you will have escaped from your insanity, at least in that area.

You don't have to assert that God must match your logic, or else he doesn't exist, rather I suggest that you allow that your logic and belief might not be adequate to tell you that God exists.

The same interest that looks to me like part of your religious faith can go to seek awareness of nature, or whatever, you can even call it God if you like, without it having to be Mr. God. But, if you don't allow yourself to doubt and have questions, than you don't have respect for your ideal. You are inventing God in your own image.

Respect and reverence come from questioning and suspending your assumptions, not worshiping the image you created or accepted on faith.

Printer Friendly | Permalink |  | Top
 
Ando Donating Member (112 posts) Send PM | Profile | Ignore Tue Dec-14-04 02:48 PM
Response to Reply #37
40. I'm out to get you!
Wow, I'm going to come and kill you and your family? Because I believe in something? It's my view that belief is intrinsic. Please tell me how you can function without believing anything? It's a circular argument, do you believe that you do not believe? We can ride that merry-go-round for hours. That's almost as bad as defining peace.

Authentic Christianity is really very simple, it is the daily struggle to model and exhibit the likeness of Christ. I don't worship a created image, I worship a person, a creator. That is not disrespect, it is the outward sign of my faith. Oh, yes, I do believe in changing the world, but I (like Christ) choose to change it through relationships, family, and friends. One at a time, it's called discipleship. So if I do show up at your door, it will not be to kill you or your family, it will be for coffee and a nice chat. We all have questions, I have tons of questions. We can sit around and debate the questions for eternity, but at some point a step must be taken. There are answers somewhere. Christianity allows me to step into the void without fear. If that makes me insane, then that's ok. I crave an ideal, it gives me something to move towards. But you don't have to worry about me killing anybody. And also never forget: Semper Ubi Sub Ubi.
Printer Friendly | Permalink |  | Top
 
Ignoramus Donating Member (610 posts) Send PM | Profile | Ignore Tue Dec-14-04 03:19 PM
Response to Reply #40
42. Yikes!
No, I don't actually think you are coming to kill me. Others "of faith" who worship your deity, are coming to kill all of us.

In anycase, I was making a point. If you are "of faith" you can't be reasoned with. So, what can I do about you if your faith runs into conflict with my life? If you are truly of faith, there's nothing I can do to communicate.

About belief, I went on about it in a branch of the main thread adjacent to this one. Tou can read that one, it's titled belief and language or something like that.

No, I don't think I can function without belief, but I don't see any circular argument. I don't believe that I don't believe, nor do I have such a theory. I think belief is acceptance of something as true, and that that is contrary to awareness.
Printer Friendly | Permalink |  | Top
 
Stunster Donating Member (984 posts) Send PM | Profile | Ignore Tue Dec-14-04 04:16 PM
Response to Reply #42
46. That's a false dichotomy
Everybody has beliefs which they can't prove, but are necessary for mental sanity.

Science itself, not to mention numerous aspects of everday life, are based on faith. That's not contary to reason. In fact, it would be completely unreasonable not to have faith in what people generally tell you, or what it says in a reputable history book, or in the evidence of your own eyes, etc.

Faith that logic and mathematics are not systematically deceptive.
Faith that our sensory and perceptual and cognitive apparatuses are not systematically malfunctioning.

There is a sense in which we don't know that there exist minds other than our own. It seems to be logically possible and coherent to suppose that there are no other minds than our own, and that what seem to be other people are in fact just zombies, with no inner experience or consciousness at all. But who thinks this? Solipsism is an extremely rarely held belief! But if that is what a radically empiricist definition of knowledge must lead to, doesn't that indicate that there's something wrong or highly inadequate about that
type of definition?

A very common but not very convincing idea is that unless you can demonstrate something beyond all possible or all reasonable doubt to everyone's satisfaction, then it can't be an item of knowledge.
But isn't this, in fact, a quite ridiculous idea? Ludwig Wittgenstein came to think so. He thought that it was nuts to think that you didn't know this was your hand, even though you can't 'prove' it. In fact, he thought it was not so much wrong to think that you might not know if this was your hand, but that it didn't *even make sense* to doubt it.
No 'proof' offered (and what the heck could such a proof even look like?) could possibly be as strongly credible as the belief that this is your hand *itself, without the proof*.

Never mind God! What does it *mean* to say that you don't know that
this is your hand, or that you don't know that there exists an external material world independently of your mind, or that you don't know that there exist minds other than your own? The very possibility
of meaning, language, thought, reasoning *presupposes* that you exist, in a material world, and that unless you're an amputee, that this is
indeed your hand, and that other people have minds which can know this fact about you too.

Thus the mere fact of the logical conceivability of there not existing any bodies, or any minds, is surely not adequate grounds for doubting
claims to knowledge of the existence of bodies or of minds in general. But if this is so, then the mere fact of the logical conceivability
of God not existing is surely not adequate grounds for doubting claims to know that God exists. If this is supposed to be the justification for agnosticism, it's a lousy one.

When evaluating knowledge-claims, we must look at what is reasonable to believe given the necessary background presuppositions of all possible experience, what makes sense of all the data, what best explains the data, what is logically coherent, logically elegant, logically predictive of phenomena, and so forth. Knowledge is a holistic enterprise. It cannot be reduced to just pure logic, nor pure observation. It cannot be held to impossibly high standards of demonstrability. And it must rely on faith---faith in the general reliability of our cognitive equipment, including both our
powers of inference and our powers of observation. These powers cannot (non-circularly) be demonstrated systematically to be reliable, since any purported demonstration would itself rely on them.

There is an argument deriving from Descartes, a sophisticated version of which has been developed by Alvin Plantinga in modern times,
that for knowledge to be possible, God must exist. Plantinga develops the argument in his book WARRANT AND PROPER FUNCTION, and in a number of related articles. Basically, he argues that for a belief to be rationally *warranted*, we must have good grounds for believing that
our belief-formation system is *functioning properly*; and that if it is functioning properly, it will generally lead us to have *true* rather than false beliefs. How could a properly functioning belief-formation system generally lead us to truth unless it was designed to do so? Could evolution, by sheer chance, and without
intelligent guidance, cause the emergence of cognitive equipment which, when functioning properly, reliably led to truth? Given that
there are many ways that adaptive *behavior* could have evolved *without* us having reliable cognitive faculties with regard to *true belief* (i.e. many adaptive *behaviors* are logically compatible with systematically false *beliefs* about the world---our ancestors had many, and maybe we do too)---the belief that evolutionary processes operating by sheer chance alone could have generated such faculties is either improbable, or its probability is at best unknowable. Hence,
if our cognitive faculties evolved by sheer chance alone, without intelligent guidance, then we have no knowledge. So, if we *do* have knowledge, evolution must have been intelligently guided, and this gives us grounds for believing in God.

A simplistic objection to knowledge-claims regarding God is that we do not see God, or otherwise have sense perception of God. This is the 'invisible pink dragon' objection. But it's a lousy objection because invisible pink dragons play no explanatory role in any theory of reality. There are, however, lots of invisible things which do play an explanatory role in a theory of reality and are consequently accepted
as being real. Physicists postulate many of them, even though they don't just happen to be invisible, but are *intrinsically* so: quarks, gluons, gravity, black holes, and indeed anything that existed less
than 300,000 years after the Big Bang, since it was only then that photons could travel freely and so render the universe *visible at all*.

So we have plenty of examples of invisible entities being postulated by science--these are only a tiny few from a long list. They are accepted as legitimately real, because they are the conclusions
of inferences to the best explanation of what we can see or experience physically.

But even all that we can experience physically is only part of what we ordinarily take to exist. Our conscious minds are not objects of physical experiences. But we take it that many minds exist. Ordinarily we take rational thoughts, emotions, concepts, numbers, goodness,
moral obligations, meanings, logical relations, beauty and so on to exist. These, not just physical things we can see, form a central part
of our experience of reality. Hence if physicists are allowed to postulate invisible things in making inferences to the best explanation of the physical phenomena we perceive (and they do,
by the barrowload), then non-physicists need not in principle be precluded from postulating invisible realities in making inferences to the best explanation of these other, non-physical phenomena or 'data' (reason, value, consciousness, meaning, etc). I.e, there is no lapse in rationality here, at least none in principle. Unless of course you think that physicists are equally guilty of irrationality in postulating invisible entities to explain *their* data.

You might not agree with a *particular* inference made, but it would not be sufficient to justify this disagreement *merely* to say that the inference involved the postulation of an invisible entity. Such postulations are a commonplace in all of rational inquiry, including physical science.

But given this picture of what knowledge is, then agnosticism about God's existence is lent no support by any *general* epistemological
considerations. In short, there appear to be no generally acceptable reasons for holding that God's existence, any more than the existence
of anything else, is unknowable. If anything is knowable, if in other words knowledge is possible at all, then it is *possible* to know whether God exists, and agnosticism in the strict sense is false.

This doesn't preclude faith. On the contrary, *all* knowledge presupposes faith. One cannot, by definition, prove the axioms of logic, nor the foundations of mathematics, nor the reliability of sense-perceptionFurthermore, at least in the Catholic tradition, faith is not needed to believe in the existence of God. There are rational grounds for believing that there is a God (St Thomas Aquinas and others developed rational arguments for theism). Rather, in the Catholic tradition, faith is faith in what God has revealed of Godself in and through the person of Christ. Since this depends on the veracity of the apostolic witness, it is appropriate to talk of faith. I believe the writers of the New Testament were essentially telling the truth about how they had experienced the divine revelation that they experienced rather profoundly because of the life, death, and resurrection of Jesus.

And in spirituality 'faith' has another meaning, more akin to trust among persons, like a child trusting its parent, or a wife her spouse. This has nothing to do with existence-claims. The child or the wife is not doubting the *existence* of the parent/husband. They are trusting that they are loved by the other. And this is what faith tends to mean in the spiritual life.
Printer Friendly | Permalink |  | Top
 
Ignoramus Donating Member (610 posts) Send PM | Profile | Ignore Tue Dec-14-04 08:32 PM
Response to Reply #46
52. leaving a brain ajar
I don't know what to say about a lot of your post, because I don't think it applies to me. For example, I don't argue that you have to prove something or else it doesn't exist.

By belief and faith I mean an acceptance of something as true, as opposed to a tentative trust that something is true.

Awareness is supposed knowledge of reality. Acceptance of something as true is not awareness, it is committing to an idea (the cessation of curiosity about a matter).

I assert that it is good to allow for the possibility that there can be something that you can't conceive of that is true. This does not prevent me from operating, which it seems I can only do under the influence of my beliefs, with these being acquired subconsciously.

Solipsism is significantly symbolic, I think. A FOX news devotee is not unlike a brain in a jar being fed hallucinations. And in fact, I think we all are really.

It is entirely possible to perceive things that are not "real" vividly and the same as anything that you would think is "real".

What can be left to hold onto, if you can't trust your perception? You can start with the notion that it's possible that your perception is flawed and proceed from there. Hopefully, in that case, if the divine space lobster commands you to kill non-believers, you will have a frame of reference from which to doubt the validity of that.

I suggest that it is unlikely that there is an omnipotent anthropomorphic being that is the sole creator and governor of all reality which creates people, by whim (as being the sole creator, there could not be any reason other than it's whim), and gives them awareness and gives them genitals and chooses to have emotions about what the people do with their genitals.

I think it is more likely that such a story matches what people are concerned about, so perhaps it is more reflective of what some people were concerned about, who wrote about it.

Still philosophies don't have to be 100% correct in order to be profound. You can be in awe of reality and study it and value it, and you can try to learn from people's philosophies.


For the part about the unlikelihood of nature occurring by chance, that simply pushes the issue up a level to the intelligence that supposedly governs reality, which by the same logic couldn't exist without a cause. If you argue that that being was not caused, than the same argument applies to the notion that reality doesn't have to have been caused.

There doesn't have to be a cause of reality if you consider time to be a dimension. From our perception of time, we are drawn to the notion that there has to be something before the start of time, but it makes no sense. Just as it's non-sensical to talk about something being external to it's own width, it could be non-sensical to talk about time before it's own dimension.
Printer Friendly | Permalink |  | Top
 
Zhade Donating Member (1000+ posts) Send PM | Profile | Ignore Fri Dec-24-04 04:00 PM
Response to Reply #46
167. The old "do you believe Spain exists?" argument.
One flaw: the belief in Spain can be tested. By GOING THERE.

You cannot prove the existence of any god, however. At least, not at this point in time. To suggest otherwise is not only intellectual dishonesty, but injurious to the faith involved, since it's supposed to be based on faith, not proof.

Printer Friendly | Permalink |  | Top
 
Tafiti Donating Member (1000+ posts) Send PM | Profile | Ignore Wed Dec-15-04 10:32 AM
Response to Reply #42
63. A small point on "belief".
As MJK writes in the liner notes of Tool's 1997 album "AEnima":

"Beliefs are dangerous. Beliefs allow your mind to stop functioning. Belive in nothing..."

In other words, if you have a FIRM belief in something, you are no longer AS open to entertain facts, reason, or alternative views/ideas because you've essentially said, "Case closed. I believe." Perhaps you aren't shut off completely, but certainly you would have a tendency to let other ideas bounce off, so-to-speak, even if only subconsciously.

This is not to say that belief is "wrong" per se, but they should be held very cautiously, and loosely. Have ideas, not beliefs.
Printer Friendly | Permalink |  | Top
 
Brentos Donating Member (230 posts) Send PM | Profile | Ignore Tue Dec-14-04 01:44 PM
Response to Reply #32
35. Your only problem with this...
is that the Divine Space Lobster is real! I belong to the Crustacean Church, ELCB, Northern District, 2nd tenure, under Fishop Terraclawmaniac III.

I will start a flame war if I see any more denigration of the Church of Crust! :-)

BTW: all snails must be killed.

Brentos, the Crustaceanite

Please note: the following parody and silliness is brought to you in part by the good-will and funny bone of Brentos. All insulting insinuations are your own, and management does not accept any responsibility for the way neurons shoot around in your cranium. Please address all complainst to the prophet Spongebob. That is all.

BTW: good post. While I will disagree with belief = insanse, blind belief sure can (when common sense and good reason fail...sorry world!). I like to think that I am pretty rational and not insane. I also think that lack of belief = belief in nothing in particular; but still, is a belief.
Printer Friendly | Permalink |  | Top
 
Ignoramus Donating Member (610 posts) Send PM | Profile | Ignore Tue Dec-14-04 02:38 PM
Response to Reply #35
39. the word belief and language problems
I always seem to run into language problems. Probably not just because of the ambiguity of words. It's being inarticulate, to some extent.

Anyway, here are some word definitions I've been assuming:

belief - unintentional acceptance of an idea
theory - intentional entertainment of an idea
trust - intentional tentative acceptance of a theory
faith - belief in a doctrine

Of course, word meanings are variable, so context is important. And there are other definitions for the above words, which I would agree with. Also, the various meanings probably get mixed up when I talk.

I can't decide to believe that I can walk on air. And I can't decide not to believe that I will not break my leg if I jump off of the roof. And I could probably train myself to believe that I can fly.

I come to believe things unconsciously as a result of my thoughts and/or experiences.

My theory is that I do not perceive reality accurately and my beliefs are highly suspect. I definately don't believe my own theory though.
Printer Friendly | Permalink |  | Top
 
Brentos Donating Member (230 posts) Send PM | Profile | Ignore Tue Dec-14-04 04:56 PM
Response to Reply #39
47. Let me take a stab...
This is a cool excessive! Let's work together on this...my thoughts..

There are two main axis', Intentional/Unintentional and Evidence/No Evidence (these are the two I'm starting with, we can add more if needed)

Thus, we need for words to describe these possibilities:

Unintentional with Evidence:
Unintentional without Evidence:
Intentional with Evidence:
Intentional without Evidence:

my choices would be:

Unintentional with Evidence: Doctrine
Unintentional without Evidence: Dogma
Intentional with Evidence: Theory
Intentional without Evidence: Belief.

Example, as a kid, I was brought up in my belief of God. This would actually be a Dogma, as I didn't really have a choice until I was old enough to make my own decision to Believe in God (accept Jesus into my heart). I was also brought up under the Doctrine that dinosaurs were cold blooded. It was just accepted. Later theories have shown that they were warm-blooded and had similar ancestors to bird.

I'm not yet sure where Posit would go. I must be missing an axis. Example: I believe in God, but I Posit that only the rejection of God casts you into Hell (Different then only accepting Jesus saves you). I don't fully believe it, yet I'm investigating the thoughts, impact, idea, etc. and haven't discounted it.

Well, I'm still working on this! :-) Let me know your thoughts.

Thanks,
-Brentos, the Thoughtmaker
Printer Friendly | Permalink |  | Top
 
Stunster Donating Member (984 posts) Send PM | Profile | Ignore Tue Dec-14-04 03:40 PM
Response to Reply #32
45. How does your incredibly strong feeling
that it is correct to love people, and incorrect to abuse and kill them, differ from a belief that there is a moral duty to act thus?

Whether it differs or not, can you see, or touch this feeling/duty?
Is it a physical, visible object? Can any conceivable scientific test prove, not just that you have this feeling, or that people believe there is such a duty---but that this feeling is objectively correct, or that there really is such an objective duty--that is, that it is normatively binding upon our conduct, no matter whether the desires and wants you have are those of a Ghandi, or those of a Hitler?

I ask, because it strikes me that the divine space lobster and this feeling/duty are in the same boat as far as being knowable by scientific methods. Of course, they are very different in other ways.

For one thing, there is no divine space lobster, whereas there is a duty not to abuse people--at least, that's pretty much what most--no, all--sane people believe. Unless most people are in fact insane (including me), there seems to be nothing wrong with that. Hence there is nothing wrong in principle with believing in something which we take to be real and important to our lives, but which we can't verify by the methods of physical science.

Most of us think another non-scientifically verifiable, but nevertheless very important real property of human living is that we have, in some significant sense, free will. So that would be another example of something non-physical which most of us, without apparently being insane, believe to be real and important.

So, if this sort of thing is ok in principle, which I think it rather obviously is, then it may well be that one can reasonably, non-insanely, believe in God, even though, like the divine space lobster, God is not subject to physical investigation.

So maybe the moral is that we shouldn't, or needn't, accept being subject to physical investigation as the sole criterion of reality, or of importance in human living.

I am reminded of something Wittgentstein said: "Even when all the possible scientific questions have been answered, the problems of life remain completely untouched."

He also said, "Not how the world is, but that it is, is the mystical."

And: "A man's thinking goes on within his consciousness in a seclusion in comparison with which any physical seclusion is an exhibition to public view."

And: "At the core of all well-founded belief, lies belief that is unfounded."
Printer Friendly | Permalink |  | Top
 
Ignoramus Donating Member (610 posts) Send PM | Profile | Ignore Tue Dec-14-04 06:29 PM
Response to Reply #45
50. the tyranny of common sense
My intentional trust in love is my own doctrine and it is part of my morality, but I don't take it to be a moral duty assigned to me by a being.

The big difference between my morality and that of some Christians is that my morality isn't "faith based". I think it is true, I feel that it's true, I choose to trust that it's true and I find myself believing that it's true, but not because a Mr. God told me so. And I don't consider people that don't share my belief as being heretics.

Yes, I agree that we should not accept being subject to physical investigation as the sole criterion of reality, or of importance in human living. In addition I think "common sense" is suspect.

It is "common sense" that there is no space lobster. It's also been "common sense" that women belong in the kitchen and slaves belong picking cotton in the fields.

I am not arguing that if something can't be scientifically verifiable it doesn't exist. I'm arguing that space lobsters might not exist, even if you assert it.

At the heart of all this is, I suggest that people have doubt about their doctrines. If you think there is an all powerful being that created you and is really concerned about your genitals. I recommend being suspicious about that.
Printer Friendly | Permalink |  | Top
 
Stunster Donating Member (984 posts) Send PM | Profile | Ignore Tue Dec-14-04 08:07 PM
Response to Reply #50
51. I don't see how this is adequate
You wrote: "My intentional trust in love is my own doctrine and it is part of my morality, but I don't take it to be a moral duty assigned to me by a being."

Are you saying that moral duties are things we simply assign to ourselves? But how, then, could it be 'correct' to love other people, and 'incorrect' to kill and abuse them? What is the source of the normativity of moral requirements? I mean, would it be ok for Hitler to say, "Well, you may assign to yourself the obligation to love Jews. But I, on other hand, assign to myself the obligation to eradicate them from the face of the earth. Who's to say that you are correct, and that I, the Reichsfuehrer, am incorrect?" Would Hitler only be incorrect because he lost the war?

One of the questions I asked was this one:

Can any conceivable scientific test prove, not just that you have this feeling, or that people believe there is such a duty---but that this feeling is objectively correct, or that there really is such an objective duty--that is, that it is normatively binding upon our conduct, no matter whether the desires and wants you have are those of a Ghandi, or those of a Hitler?

I draw your attention to the objectivity part.

Now in response, you may wish to say that there is an invisible, objective moral order which is the source of what one might want to call objective moral truth, but that this order is impersonal. This would answer my concerns about the need for morality to be objectively binding, and not just a reflection of contingent individual preferences or whims. However, I find it decidedly odd that morality, which is primarily about how rational beings, i.e. persons, treat one another, could have an impersonal source or metaphysical origin. By definition, an impersonal moral order is incapable of caring about what happens to rational beings, since caring is an attribute of personhood. But if it's not capable of caring, how can such an order oblige or bind our conduct?

Now of course, truth objectively binds our intellects, and so one might say, as Platonists do, that The Good binds our wills, and hence our conduct, but impersonally (on the assumption that the Truth is impersonal also). However, reason and value are not objects we encounter independently of personhood, or at least consciousness. Can a thought about the world be true if there is nobody to think it? Can a painting be beautiful if there's never anybody to appreciate it?
Can a state of affairs be good, if there's never any rational being to judge that it is? Can an action be wrong if there's never any consciousness of its wrongful character?

Seems to me that all the central phenomena associated with both Reason and Value imply personhood in some essential way. And so how could the Form of the Good exist without personhood? Ah, you'll say, but we're here. Well, yes. But my concern was that morality ought to be objective and independent of, not just dependent on the contingent, inconsistent, and fallible judgements of human beings.

I tend to think of of God as the eternal, rational, moral, personal being whose intellect essentially knows the truth, and whose will essentially wills the good. And that makes more sense to me than there being some impersonal moral order which, somehow unconsciously, unknowingly and unwillingly (these adverbs apply because it's impersonal) objectively binds or obliges us in terms of the way we live our lives.
Printer Friendly | Permalink |  | Top
 
Ignoramus Donating Member (610 posts) Send PM | Profile | Ignore Tue Dec-14-04 09:31 PM
Response to Reply #51
53. willy nilly
Within the idea of not breaking your arm, it is correct to not break your arm and incorrect to break your arm.

As for my morality, a lot of it is based on such reasons, but the core of it does not appear rational. I don't know of a test that can prove that it is "right" in an absolute sense to be like Ghandi and "wrong" to be like Hitler.

If you plant seeds of an invasive plant on a hillside and it takes over and kills the other plants, it is natural in a sense, it is also natural to plant the seeds, but it is dissonance within the feedback loop of an ecosystem that creates an "unhealthy" bias, I suppose.

In the case of the invading plant, I would not really take it personally that the plant invaded. It's not heretical for the plant to invade, it's just unhealthy. An arbitrary morality coming from a doctrine would tend to be abstracted from the situation and view violations as an affront, rather than simply being unhealthy. The subtle difference I think explains the punishing nature of some religions.

I don't think I love because it gets me something in return, in such basic terms. Love could be a form of communication, not unlike magnets drawing toward metals, or plants sending spores out to other plants. I don't have to assume an anthropomorphizing notion for the basis of my morality. I reject the idea that because I can't rationalize that core morality that that implies that I should pick one arbitrarily.

If someone's doctrine advocates driving an awl through your slaves ears to designate them as your property, or shaving the heads of the females you capture in battle, and leaving them alone for a day to mourn their dead husbands, before you do with them as you wish, I would argue that that is immoral based on arguments involving a given that luckily many people share, that it is wrong to cause people suffering gratuitously, even if that argument would conflict with their doctrine.

About: "I find it decidedly odd that morality, which is primarily about how rational beings, i.e. persons, treat one another, could have an impersonal source or metaphysical origin. By definition, an impersonal moral order is incapable of caring about what happens to rational beings, since caring is an attribute of personhood. But if it's not capable of caring, how can such an order oblige or bind our conduct?"

I am part of nature, and having morality as part of that.

About the notion that morality has to have come from a God: if there were a being that created and governs all reality, the same issue about where morality comes from would apply to it. Arbitrarily accepting it's notion of morality would not solve any problem with where morality ultimately comes from.
Printer Friendly | Permalink |  | Top
 
Stunster Donating Member (984 posts) Send PM | Profile | Ignore Wed Dec-15-04 12:03 AM
Response to Reply #53
56. Still having problems with your analysis
About the notion that morality has to have come from a God: if there were a being that created and governs all reality, the same issue about where morality comes from would apply to it. Arbitrarily accepting it's notion of morality would not solve any problem with where morality ultimately comes from.

This is a poor objection, given what you had said previously.

You had said previously that you personally regard morality as something arising out of your own will and intentions---that you, as it were, legislate morality for yourself, by some intentional act. You seem to think this is, in principle, a perfectly adequate way of accounting for how morality arises in the human world. Yet in your quoted remarks above, it would appear that you are not allowing this to be an option for God----you're demanding that morality must have a source or origin external to God, which God then has to take account of, on pain of having morality be 'arbitrary'. But isn't this inconsistent? Why isn't it arbitrary in your case? And if it isn't arbitrary in your case, even though your own mind is the source of moral claims you impose upon yourself, why can't God's intentions be the source of morality without thereby being arbitrary?

Now, you might object that I had held that morality must be objective for human beings, otherwise highly counter-intuitive consequences follow, whereas there is no 'higher God' legislating objective morality for God. But this objection fails to take account of the fact that humans are imperfect, fallible beings, whereas (by definition) God is not imperfect or fallible.

So yes, a perfect rational being could legislate morality, as it were, by his own intentions. And being perfectly rational, then of course, this perfect being wouldn't make mistakes of reasoning or judgement, nor be swayed by unruly emotions.

Morality, according to Kant, precisely IS what would be legislated for themselves by perfect rational beings.

At this point you might object, ok, but then God would still have to be guided by something external to God, namely reason.

But, of course, in classical theism, God is self-subsistent Reason, or the very being, nature and essence of Reason.

So in that case, the charge that God would still have to be guided by something external to God falls to the ground.

Printer Friendly | Permalink |  | Top
 
Ignoramus Donating Member (610 posts) Send PM | Profile | Ignore Wed Dec-15-04 02:33 PM
Response to Reply #56
78. hamster unity
I did not say, or I do not mean that I regard morality as coming out of my own will. You were arguing that morality is very personal, and so it must come from an intelligent being rather than being a result of nature, I was arguing against that.

I think the core of my morality is not rational. The source could be partly instinct, it could be a property of human life, I'm not sure.

I also was not arguing that a God couldn't invent it's own morality, really. You were arguing that morality can only come from a God because morality is personal. I was saying that if you make that argument, it would also have to apply to a God, because by your argument, morality can't come out of nowhere, so the God couldn't create it for itself.

What I meant by an arbitrary morality, is one that is adopted simply because it is dictated by a doctrine.

The argument about willed fundamental morality being possible for a God and not for humans, because the God is perfect and humans are not, is questionable I think.

First, I didn't make either of the claims that morality must come from your own will, or from something external, so I don't think the argument applies to what I had said. My theory is that the core of my morality is part of my nature.

As an argument in and of itself, it makes sense in a contrived scenario, for example there is a robot and some hamsters, you arbitrarily accept that there can be nothing external to the robot and the hamsters and state that the robot is God and morality is the mental suggestions broadcast to the hamsters by the God robot.

But, outside of the contrived scenario, the argument doesn't work because of the fatal problem with the argument that there can be a God that is a human-like being that governs and creates all of reality.

Outside of the contrived scenario, the robot is part of nature. The robot and the hamsters are all perfect, in that they are exactly what they are, perfectly.

It is a non-sensical notion to have there by a substance that is something that is called nothing, in which there is a being called God, which creates another thing called reality. Nothing doesn't exist, by definition. There can not be nothing.

Similarly, there can't be a thing that is everything, except within word problems. Everything that exists, does exist. So, there can't be a thing that creates another thing that is everything.
Printer Friendly | Permalink |  | Top
 
trotsky Donating Member (1000+ posts) Send PM | Profile | Ignore Tue Dec-14-04 01:34 PM
Response to Reply #31
34. Yes, you are blaming the seeker.
you weren't seeking what God was offering

Pardon my French, but how the hell do you know what I was seeking? Why do you presume to know I was seeking God in a scientific, "testing" way? I was quite religious and had a deep faith at the time. Slowly I lost that faith as it stopped feeling "real" to me, and when I started truly reading the bible, that pretty much killed it for me. It's not like I shouted out an ultimatim - "God, give me a sign!" It was a slow process of reason & my own sense of morality that led me to atheism.
Printer Friendly | Permalink |  | Top
 
Ando Donating Member (112 posts) Send PM | Profile | Ignore Tue Dec-14-04 02:13 PM
Response to Reply #34
36. Whoa!
Edited on Tue Dec-14-04 02:15 PM by Ando
Easy, I'm not trying to belittle you or presume anything. First, I don't look down on anyone because they do not share my beliefs. I'm not superior to anyone else because of my beliefs. It's ok, I don't bite.

Back to the discussion. Let me make my point this way. Let's say I make the best darn chocolate cake in the world. I know my cake, I know the recipe by heart and I can give that recipe to anyone. If I give someone the recipe and they come back and say "It didn't work, the cake exploded in my face!", I can reasonably assume that there was a mistake in the baking of the cake. Notice there is not a value judgement here, it's just simple reason. I know my faith and it has a very simple recipe. You just baked a different cake, I'm sure you don't feel superior to me bacause I didn't follow your recipe.

I'm not out to get the atheists or try to wow them with my superior knowledge. Every decision for Christ is a decision of the soul, not the intellect. Christianity levels the playing field, it doesn't play favorites. Intelligence, Culture, Time, and Circumstance are all variable, it's the human soul that's universal. Thus Christianity is made in the soul and nowhere else.
Printer Friendly | Permalink |  | Top
 
trotsky Donating Member (1000+ posts) Send PM | Profile | Ignore Tue Dec-14-04 02:25 PM
Response to Reply #36
38. You're still blaming the seeker.
And you obviously can't see it.

So now it's a matter of someone's "soul," and if their soul just doesn't see it, it must still be their fault because Christ/God/the Cosmic Lobster are there and waiting!

Regarding your cake analogy, if I had actually seen or tasted your cake at least I would know it CAN be made. Problem is, your amazing chocolate cake can only be seen or tasted by you.
Printer Friendly | Permalink |  | Top
 
Ando Donating Member (112 posts) Send PM | Profile | Ignore Tue Dec-14-04 03:06 PM
Response to Reply #38
41. I think we've reached the end of this one
We'll just have to disagree and move on I believe. If you're asking me whether or not I think people are responsible for their relationship with God then I say yes. I don't blame God for anything. And as for the cake, blind faith is a bitch isn't it? I only have my life to offer as an example, that's the interesting thing. But for a lot of people who are struggling to understand the hurt this world contains, a life of peace is a powerful thing.
Printer Friendly | Permalink |  | Top
 
trotsky Donating Member (1000+ posts) Send PM | Profile | Ignore Tue Dec-14-04 03:24 PM
Response to Reply #41
43. I agree to disagree.
I will only note that people of other faiths and even atheists can also lead exemplary lives of peace (Ghandi, MLK, Carl Sagan) - it's more of a commitment to one's fellow human beings than to any gods. Either you have it or you don't, and clearly no amount of faith will change that. (See right-wing fundamentalists.)
Printer Friendly | Permalink |  | Top
 
Ando Donating Member (112 posts) Send PM | Profile | Ignore Tue Dec-14-04 03:35 PM
Response to Reply #43
44. agreed
It's been a good discussion, but discussions only get you so far. I really recommend that you read "Orthodoxy" by G.K. Chesterton. If I could recommend one book to anybody who's ever had an interest in theology and doctrine, it would be that one. Thanks for the thoughts!
Printer Friendly | Permalink |  | Top
 
Brentos Donating Member (230 posts) Send PM | Profile | Ignore Tue Dec-14-04 05:08 PM
Response to Reply #41
48. About the cake....
Well, actually it is the Great Cosmic Lobster that makes the holy Chocolate Cake. It's in the 3rd Book of Calamari, v3-5.

:-)

I would agree that people are responsible for their relationship with God, as that is the choice that they are given.

What I can't do, is condemn someone to Hell. That is only God's providence. While I can say what I believe is the way to Heaven, it is possible (and probable) that I don't know all of God's plan, and maybe different people/cultures/time periods have different or multiple ways to reach heaven. Maybe just belief is needed. Example: Belief in being good.

Thanks for the interesting discussion!

Brentos, Calmarist High Poobah, the Church of Crust, 2nd Order :-)

Printer Friendly | Permalink |  | Top
 
Brentos Donating Member (230 posts) Send PM | Profile | Ignore Mon Dec-13-04 10:40 AM
Response to Reply #10
24. Beautiful Post!
I also have "felt" God, maybe not in a literal, "God touched me "sense, but in the way that my life has changed for the better the closer to God that I try to be. I enjoy life more, I see beauty I've never seen in day-to-day life before, my relationships are better, my prayers help keep me humble and grounded, etc. An atheist could look at this and claim to get the same benefits through a philosophy of some sort, and that's fine. I'm not making fun of atheists or putting them down, I just feel that they are wrong as they feel that I am wrong. Honest open debate is good, denigration on either side is wrong. If I (or an atheist, or member of another religion) feel that we want to get our point across and maybe open up some thoughts to someone with an opposing viewpoint, how can we be negative towards them? So, my atheistic, Buddhist, Jewish, Islamist, etc. friends, my heart and mind are open to you all, my hopes and dreams are much like yours, and I am curious as to your beliefs, as I believe we should all be. God does not frown upon questioning or exploration, God wants us to see all the facts and choose Him.

Sorry for the ramble,
-Brentos, the Freshmaker
Printer Friendly | Permalink |  | Top
 
Zhade Donating Member (1000+ posts) Send PM | Profile | Ignore Fri Dec-24-04 03:49 PM
Response to Reply #10
166. Interesting. Another "sees" Jesus, and it makes you uncomfortable.
Yet you claim to have experienced God, though it's all in your own head (it DID happen in your own head, regardless of whether it actually happened or not).

Sounds like "spiritual jealousy" to me.

Printer Friendly | Permalink |  | Top
 
Stunster Donating Member (984 posts) Send PM | Profile | Ignore Mon Dec-13-04 02:32 AM
Response to Reply #8
11. A poetic follow-up to my first reply
In everyone there sleeps
A sense of life lived according to love.
To some it means the difference they could make
By loving others, but across most it sweeps
As all they might have done had they been loved.
That nothing cures.


----Philip Larkin, "Faith Healing" (1964)
Printer Friendly | Permalink |  | Top
 
Stunster Donating Member (984 posts) Send PM | Profile | Ignore Sun Dec-12-04 07:02 PM
Response to Original message
3. More on inference
In an interesting recent book by the British Astronomer Royal, Martin Rees, entitled JUST SIX NUMBERS, Rees considers the odds against the universe being physically suitable for human habitation. He agrees that if this is the only universe there is, the odds against it being compatible with human life are fantastically overwhelming--those six numbers have a very small range compatible with the emergence of any life, let alone human life.

To get around this problem, he proposes instead the Multiverse Hypothesis---the idea that there are untold billions upon billions, maybe even an infinity, of other universes---all except ours being unobservable by us, of course. This hypothesis (which, since it's not based on any conceivable experiment, is a decided departure from scientific method) renders the odds much more favorable to the emergence of life.

If you are the only person who buys a national lottery ticket, it's very unlikely that you'll buy the winning ticket if the winning number is randomly generated. But if 100 million tickets are bought, it is much more likely that someone will win (though it still probably won't be you). That's the thinking behind the Multiverse idea. If there is just one universe, life is extraordinarily unlikely to exist within it, if it's chance that originates life. But if there is an infinity of universes, then some universes will have life, with a probability of 1, even if it's chance that generates life.

You see, if there is just one universe, ours, then it is mind-blowingly improbable that the 6 numbers referred to by the title take the values they do by chance. It would be like having to pick
a straw out of billions of billions of straws, all of differing lengths, and being shot dead instantly if you did not pick out the
shortest one----and then claiming that you survived by having the good
fortune to pick out the shortest straw--by sheer chance. No rational person would believe you, because it is far more likely, and hence much more rational to believe, that the drawing of the straw was designed or fixed in some way to ensure the outcome that allowed you to survive.

The question one is asking, in other words, and which elicits the
inference, is: what explains the fact that the numbers are all within
the appropriate range? It is impossible to derive any one of the
numbers from any combination of the others. They are six independent
parameters. Physicists can predict what would happen if you changed the values of the numbers. In most cases, the resulting universes would be very short-lived, or would fail to form the elements needed for life, or wouldn't have stars, etc. Yet there is nothing incoherent about the idea that a possible universe could exist with different values for the 6 numbers. Rees in fact thinks that the values they have in our universe were fixed shortly after the Big Bang. But he also thinks that many other values could have been set at that point, and thus that a huge number of universes are logically possible from the point of view of physics, but only a tiny fraction of them are compatible with the emergence of sentient life.

Not to beat about the bush, but Rees can see very well from the scientific evidence that the universe was very probably intelligently designed, but he doesn't want to accept this conclusion, and so in order not to have to posit one unobservable being, he posits, with unwitting irony, an infinity of unobservable beings. That's the only way he can explain why the values of the 6 key numbers are what they are in our universe. If this was the only universe, it would be astounding that it was compatible with the emergence of life, if that fact were due to chance. It would be more astounding, for instance, than if a giant automated watch-making factory were to gradually assemble itself in virtue of the chance, random movements of atoms.

It is not just the 6 key numbers that are important, of course. Rees
focuses on these because they have to have the values they have, or
very close to them, for life even to be possible. But even with the
values they have, life is not inevitable. Many other factors have
to be just right for life to emerge, evolve, and survive for a long time.

So when we calculate the odds against life forming by chance, they are
practically infinite. From a scientific point of view, this is as
good as saying that life didn't form by chance. Unless, that is, something like the Multiverse Hypothesis is true.

But far from thinking that it might be true, I regard the hypothesis as actually a sign of mental pathology. It implies, just to take one feature of it, that there is someone who is exactly like me in every respect except the number of eyelashes I have, living in another universe, typing this exact same sentence on an identical computer. And in actual fact the only real reason Rees has for accepting such an implication is because he's prejudiced against accept the truth of theism, or at least deism.

I should probably add that the only rational way to say that it's more likely that the universe is the way it is by sheer chance than by divine design---given the colossal improbability of the former---is to say that God's existence is impossible (since in that case, the odds against God existing would be even greater than the odds against a life-sustaining universe occurring by chance).

Up until fairly recently, there were two basic grounds for holding the
impossibility of theism. One was the so-called 'logical' problem of evil. However, this has now been widely conceded to have failed as an argument against the possibility of theism. Most philosophers of religion think that Alvin Plantinga has demonstrated its failure.
Hence most atheists have turned instead to the 'evidential' problem of
evil. But of course, that's useless as an argument for holding that theism is impossible, since it is inherently a probabilistic argument.

The other line was simply a general philosophical commitment to materialistic naturalism. But this is now in deep trouble also, because of the 'Hard Problem' of consciousness. (See David Chalmers, the New Mysterians, etc). After decades of the hardest materialism, more and more philosophers are coming to admit that consciousness is irreducible to material reality, ontologically and/or epistemologically.

But if that is so, the rationale for the objection to theism (which
takes consciousness as the ontologically ultimate) is undercut.

Given the irreducibility of consciousness, and given the strong
evidence for design, the theistic inference is now staring open-minded people in the face.

People like Antony Flew, for example.
Printer Friendly | Permalink |  | Top
 
papau Donating Member (1000+ posts) Send PM | Profile | Ignore Sun Dec-12-04 07:59 PM
Response to Reply #3
5. EXCELLENT POST - very good points - the creation of the universe is
where science folk might hang their hat for feeling their is a God.

But in the end, being an Atheist is as much a matter of faith and belief as being a Christian. Only their choice of buzz words is "logic" - I don't like blowing through the pretend "logic" because I also do not like non-Christian's blowing through those parts of my belief that are based on faith and feeling.

I think the atheist should be afforded the loudest speaker phone he can get to spread his faith and explain his feelings and belief. And if he is a fundie in his atheism, and wants to tell all non-atheists that they are on the wrong path, then treat him like any other fundie.

:-)
Printer Friendly | Permalink |  | Top
 
Ignoramus Donating Member (610 posts) Send PM | Profile | Ignore Sun Dec-12-04 08:07 PM
Response to Reply #3
6. the who created God idea
I haven't read the article that you site, but there is a simple flaw in the idea that all reality was created by a God.

It is possible that a being could be the creator of what we consider the universe and physical laws, etc., given that there could be something outside of the universe and physical laws. It is a non-sensical argument in that case to say that all reality had to be created by a being, because it then begs the question, what created that being?

The notion that something had to create our reality comes from our perspective as beings within time. If you think of time as a dimension, it becomes non-sensical to ask what occurred before the beginning of time.

For example, if instead of "before" and "after" as directions in time, you use "narrower" and "wider" as directions in width, try to picture the concept of "before the beginning of 0 width", but in terms of width instead of time; it becomes nonsensical (in an absolute sense) to ask what is narrower than 0 width.

A being creating what we call our reality, would still exist and so be natural, which is very significant.

One of the implications of an intelligent being creating our universe, is that someone can pose rules supposedly commanded by the being that we are to obey, these can and do apparently tend to be arbitrary and suspiciously similar to something a human culture would want to impose on people.

A natural law on the other hand is less human and arbitrary generally. It's not immoral to break your ankle when falling off the roof because you "disobeyed the laws of gravity", it's just unfortunate.

The belief in an omnipotent being tends to come with a disrespect for nature. The chance of conflicting with reality are obviously reduced if you have respect for nature, so it's useful to have that respect instead of a belief in a omnipotent human-like God entity that can dictate arbitrary rules.
Printer Friendly | Permalink |  | Top
 
trotsky Donating Member (1000+ posts) Send PM | Profile | Ignore Sun Dec-12-04 08:45 PM
Response to Reply #3
7. There are many flaws with this line of thinking...
the first of which being, we just don't know! Perhaps there WERE other universes where the "six constants" (which is Martin Rees' creation, not some sort of scientific axiom) were not suitable for life or even for a functional universe. There may have been billions of "attempts" before ours happened. Your argument then becomes an argument from the unknown, an amazingly weak foundation indeed!

The problem of evil has been dismissed? I must admit, that's news to me. Perhaps you could share?

The primary objection to theism is that the universe works without any appearance of a god behind the scenes. Why add the unnecessary complexity of a deity - which, by the way, doesn't add anything to the explanation of the universe?

But it's interesting you think that it's only closed-minded people who could at this point doubt the existence of a supreme being. That attitude fits quite well with the bigotry you displayed on the "invisible gardener" thread.

It is easy to see where the fundamentalist mindset comes from.
Printer Friendly | Permalink |  | Top
 
Stunster Donating Member (984 posts) Send PM | Profile | Ignore Mon Dec-13-04 01:14 AM
Response to Reply #7
9. As Jesus said to the Sadducees...
...you are very much mistaken.

Printer Friendly | Permalink |  | Top
 
trotsky Donating Member (1000+ posts) Send PM | Profile | Ignore Mon Dec-13-04 08:46 AM
Response to Reply #9
16. Well that ends that, huh?
Great discussion. Thanks for the insightful response.
Printer Friendly | Permalink |  | Top
 
Stunster Donating Member (984 posts) Send PM | Profile | Ignore Mon Dec-13-04 02:35 AM
Response to Reply #7
12. A scientific follow-up to my first reply

Evolution, presumably, occurred through the normal operation of
natural laws. But this was only possible, as I shall argue in the
next chapters, because the laws of nature are themselves quite
special. The biologist Richard Dawkins, referring to William Paley's
"watch argument", calls the universe the "Blind Watchmaker". The
"watches", for Dawkins, are the intricate structures of living things.
The universe, mindlessly following its mechanical laws, has succeeded
in crafting these astonishing structures by repeated trial and error.
What Dawkins does not seem to appreciate is that his Blind Watchmaker
is something even more remarkable than Paley's watches. Paley finds a
"watch", and asks how such a thing could have come to be there by
chance. Dawkins finds an immense automated factory that blindly
constructs watches, and feels that he has completely answered Paley's
point. But that is absurd. How can a factory that makes watches be
less in need of explanation than the watches themselves? Paley, if
still alive, would be entitled to ask Dawkins how his Blind Watchmaker
came to be there. Perhaps Dawkins would answer that it was produced
by a Blind "Blind Watchmaker" Maker.


--- Stephen M. Barr, MODERN PHYSICS AND ANCIENT FAITH, (University of Notre Dame Press, 2003), p.111
Printer Friendly | Permalink |  | Top
 
trotsky Donating Member (1000+ posts) Send PM | Profile | Ignore Mon Dec-13-04 08:34 AM
Response to Reply #12
15. Do the research yourself.
You will find that is among many flaws in the watchmaker's argument, and that Paley's objection doesn't even succeed.
Printer Friendly | Permalink |  | Top
 
Stunster Donating Member (984 posts) Send PM | Profile | Ignore Mon Dec-13-04 10:16 AM
Response to Reply #15
19. Do the research myself?
I've been doing the philosophical research, teaching and publishing on these issues for a living, for the last twenty years. I'm quite familiar with the critique of Paley, other forms of the design argument, and the rebuttals and counter-critiques.

Has it struck you that possibly Flew and others might have seen something you've missed?

Printer Friendly | Permalink |  | Top
 
trotsky Donating Member (1000+ posts) Send PM | Profile | Ignore Mon Dec-13-04 10:19 AM
Response to Reply #19
20. Here.
From http://www.infidels.org/news/atheism/arguments.html#design

The Watchmaker analogy suffers from three particular flaws, over and above those common to all Arguments By Design. Firstly, a watchmaker creates watches from pre-existing materials, whereas God is claimed to have created the universe from nothing. These two sorts of creation are clearly fundamentally different, and the analogy is therefore rather weak.

Secondly, a watchmaker makes watches, but there are many other things in the world. If we walked further along the beach and found a nuclear reactor, we wouldn't assume it was created by the watchmaker. The argument would therefore suggest a multitude of creators, each responsible for a different part of creation (or a different universe, if you allow the possibility that there might be more than one).

Finally, in the first part of the watchmaker argument we conclude that the watch is not part of nature because it is ordered, and therefore stands out from the randomness of nature. Yet in the second part of the argument, we start from the position that the universe is obviously not random, but shows elements of order. The Watchmaker argument is thus internally inconsistent.

Apart from logical inconsistencies in the watchmaker argument, it's worth pointing out that biological systems and mechanical systems behave very differently. What's unlikely for a pile of gears is not necessarily unlikely for a mixture of biological molecules.


Has it struck you that possibly Dan Barker and other preachers-turned-atheists have figured out something that YOU'VE missed?
Printer Friendly | Permalink |  | Top
 
Stunster Donating Member (984 posts) Send PM | Profile | Ignore Mon Dec-13-04 10:33 AM
Response to Reply #20
22. You're misunderstanding 'creatio ex nihilo'
The term 'pre-existing materials' implies temporality, and in particular, implies that there was a time when there were no materials.

I think Hawking has suggested in a well-known book why we needn't accept this idea.

I could go on to show why the other objections are awfully weak, but I have to go to work. It's 7.30am on the West Coast. The freeways are clogging up in Los Angeles.

Have a nice day, and life.

Printer Friendly | Permalink |  | Top
 
trotsky Donating Member (1000+ posts) Send PM | Profile | Ignore Mon Dec-13-04 10:46 AM
Response to Reply #22
27. But you're not even understanding the objection.
The point is to demonstrate how the analogy fails. "Pre-existing materials" illustrates how we know the watch was manufactured from metal, glass, etc.

So whether or not you invoke "creatio ex nihilo" the problem remains - the creation of a watch is vastly different than the creation of the universe.
Printer Friendly | Permalink |  | Top
 
catbert836 Donating Member (1000+ posts) Send PM | Profile | Ignore Wed Dec-22-04 08:34 PM
Response to Reply #15
154. What aboout Pascal's Wager?
Being all of us pop theologists, there's probably no need to explain what that is.
Printer Friendly | Permalink |  | Top
 
Az Donating Member (1000+ posts) Send PM | Profile | Ignore Fri Dec-24-04 02:28 PM
Response to Reply #154
164. Pascal's Wager was myopic
Pascal's wager was based on a binary problem. He presumed that there were two choices. Believe in God or Don't believe in God. This was compared to another binary factor, namely that God did exist or didn't exist. This matrix is flawed.

There are a multiplicity of possible gods. From polytheistic notions to ambiguous notions. Pascal only considered one aspect of one particular theistic proposition of God. And many religions postulate God as a jealous and angry person. Worship the wrong god and you are going to piss off the right god. Thus Pascal's matrix is seriously flawed in nature of dimension.

Once adjusted to include the multiplicity of gods the safe answer becomes to believe whatever you can determine for yourself to be true. Trying to force yourself to believe in a singular god proposed by someone else is potentially damning. In the end honesty turns out to be the best policy.

A further flaw is Pascal's understanding of belief. We cannot choose what we believe. We can choose paths of experience that may lead to a change in our beliefs but belief always arises from the culmination of our experiences. I can no more choose to believe in god than you can choose to believe you can walk through walls. So for Pascal to assume that one can choose to believe in God based on the advantages of it clearly demonstrates his lack of understanding of the matter.
Printer Friendly | Permalink |  | Top
 
catbert836 Donating Member (1000+ posts) Send PM | Profile | Ignore Fri Dec-24-04 07:11 PM
Response to Reply #164
170. He lived in different times, didn't he?
There weren't too many atheists around then. He did what he could with the knowledge he had in that time. And he wasn't suggesting that someone SHOULD choose God based on the advantages. Instead, he was saying that it was BETTER to. Obviously, people can't force themselves to believe something they don't. The Wager was intended for spiritual searchers who were pretty much open to either side, not people who were already sure of the existence/absence of a god.
Printer Friendly | Permalink |  | Top
 
Stunster Donating Member (984 posts) Send PM | Profile | Ignore Mon Dec-13-04 09:03 AM
Response to Reply #7
17. More on epistemology
Edited on Mon Dec-13-04 09:03 AM by Stunster
the first of which being, we just don't know! Perhaps there WERE other universes

How do I know that I am having this conversation with a conscious human person? All I see are patterns on a computer screen. I'll probably never meet you, but even if I did, how would I know that you have a conscious mind? You might just be a very sophisticated robot.
In fact, if I open up your body and peer inside, that's essentially all I'll see---that you're some kind of robot. I certainly won't see a conscious mind in there. Yet I think it's perfectly reasonable to say that I know you have a conscious mind, just from the evidence of this computer conversation.

The problem arises because we allow ourselves to be misguided by the demand to eliminate all 'Cartesian doubt', to eliminate all other bare logical possibilities that can't be proven to be false merely by scientific investigation, but which radically resist empirical falsification. There is a large philosophical literature on all this. But there is a broad consensus that the term 'knowledge' is appropriately used to apply to states of cognition which yet fall short of having eliminated all logically possible alternatives states to those we believe actually obtain.

How do I know that these are my hands I'm typing with, and that I'm not dreaming or hallucinating? One doesn't disprove this, one rejects it as not even making it into the conceptual framework within which things like proving procedures can even be thought to apply. And we choose our basic conceptual frameworks on grounds that favor simpler, less extravagant ontologies. I know this is my hand, because that's the simplest, most straightforward, least counter-intuitive, explanation of the phenomena.

How do I know that the world around me exists, and that I'm not just a brain in a vat of chemicals, which produces a complex series of illusions that there is a world external to my consciousness?

How do I know that aliens haven't abducted me while I was asleep, medically inducing a coma, and then taken me to the moon, and then wiped all trace of that journey from my brain?

How do I know that the world didn't just pop into the world five minutes, complete with everyone's memories of a seemingly longer past, historical documents, and fossils?

So, how do I know that there isn't an infinity of alternative universes a la Rees' multiverse hypothesis? Because there's a far simpler, more straightforward, less counter-intuitive explanation for the phenomena to be explained---that the universe I live in was the created by an intelligent, consciousness. But that is essentially the same criterion which lead me to believe, 'this is my hand, and the person I'm having this debate with has a conscious mind, and the world didn't pop into existence five minutes ago, and I've never been to the moon...'

Printer Friendly | Permalink |  | Top
 
trotsky Donating Member (1000+ posts) Send PM | Profile | Ignore Mon Dec-13-04 09:14 AM
Response to Reply #17
18. Simpler?
In order to solve a complicated problem, you've created an even MORE complicated entity. How does that explain anything?

And counter-intuitive is the name of the game in this universe, I'm afraid. It's counter-intuitive that light is both a particle and a wave, yet it is. It's counter-intuitive that the speed of light is constant no matter how fast you're moving, yet it is. For you to insist that a more "intuitive" explanation is more likely right is extremely questionable.

(By the way - all your other examples have real-world, repeatable experience to confirm them. They're good examples, but have nothing to do with the god hypothesis.)
Printer Friendly | Permalink |  | Top
 
Stunster Donating Member (984 posts) Send PM | Profile | Ignore Mon Dec-13-04 10:25 AM
Response to Reply #18
21. 'Simpler explanation'
Often, in science, a more ordered or complex entity or property provides a simpler, more elegant, more economic, more coherent explanation.

So I'm glad you introduced quantum mechanics, because as regards particle-wave duality, what's more intuitive is that it must have this duality (a more complex form of order) in order to more simply account for things like the double-slit phenomena, than that there are hidden variables (which are less exotic types of thing).

Similarly, curved space is a more complex kind of space than flat space. But it provides an intuitively simpler explanation for grativational phenomena than does the Newtonian account of gravity as involving a substantial force that acts at a distance.
Printer Friendly | Permalink |  | Top
 
trotsky Donating Member (1000+ posts) Send PM | Profile | Ignore Mon Dec-13-04 10:35 AM
Response to Reply #21
23. I'm afraid you're distorting what "simple" means.
Duality and QM don't just "more simply account" for the double-slit, or tunneling, or other issues - they are the only things that DO explain them.

What postulating a creator does is introduce an infinitely more complex "answer" than the question it is trying to address. And then the "answer" is unobservable, unknowable, and all questions about it are dismissed with the waving of hands. "He works in mysterious ways." "He has always existed." "I've personally experienced him even though millions of others have not, and I cannot show them how to." Etc.

Color me disappointed. Not surprised, but still disappointed.
Printer Friendly | Permalink |  | Top
 
Stunster Donating Member (984 posts) Send PM | Profile | Ignore Mon Dec-13-04 10:41 AM
Response to Reply #23
25. Wrong
Edited on Mon Dec-13-04 10:46 AM by Stunster
Hidden variables theories are an alternative explanation accounting for the same empirical data.

The rest of your comments are hand-waving exemplars of 'petitio principii'. You're stating all of it as if it were accepted fact, which it isn't. Even remotely.
Printer Friendly | Permalink |  | Top
 
trotsky Donating Member (1000+ posts) Send PM | Profile | Ignore Mon Dec-13-04 10:42 AM
Response to Reply #25
26. But if they don't really explain anything,
they're not much of an "explanation," now are they?
Printer Friendly | Permalink |  | Top
 
Stunster Donating Member (984 posts) Send PM | Profile | Ignore Mon Dec-13-04 10:47 AM
Response to Reply #26
28. Dear, oh dear
Edited on Mon Dec-13-04 10:47 AM by Stunster
That is so off-base! Bye!
Printer Friendly | Permalink |  | Top
 
lazarus Donating Member (1000+ posts) Send PM | Profile | Ignore Tue Dec-21-04 05:39 AM
Response to Reply #3
140. Nonsense
Complete gibberish.

The likelihood that life would arise in the universe = 100%

That's because we have a sample of 1. If we had a sample of two universes, one with life and one without, then the likelihood would be 50%.

Why people miss this basic fact is beyond me. Well, not really. People are constantly trying to "prove" that god must exist, and are willing to overlook and ignore anything to do it.
Printer Friendly | Permalink |  | Top
 
Ignoramus Donating Member (610 posts) Send PM | Profile | Ignore Sun Dec-12-04 07:23 PM
Response to Original message
4. no, it's impossible to prove anything...
... you might be confused about what you perceive.

Aside from it being accurate to call yourself an agnostic, there is a practical difference between an atheist and an agnostic. Just like a person "of faith", promises to resist information that contradicts their belief, an atheist is affirming that they will resist contrary information.

Reason itself falls down, around issues related to infinity, for example. Our mentality consists of at least 2 things: emotions that ebb and flow and reason that churns around, these generate perception. There is a trap in following down the road that tends to view reality as a giant clock mechanism, in that there is a tendency to succumb to an agenda that sees the world as a resource to be exploited.

Someone posted a quote from Einstein on here in the last few days, on the means of science matched with the ends of religion. It's interesting.

Religion and Science are in the same domain. In a sense, they are both Science, in that they look at the universe and reality. Adherents of either typically get religious about it, but neither necessarily has to require it.

You don't have to buy something 100% in order to learn from it. You can make use of the "Socratic method", without also having to buy into the idea that everything comes from it's opposite, for example. Part of what it affirms to say you are an agnostic, is that you are willing to learn from things even if they don't fit with your religion.


About the proving a negative issue. There is the type of negative like "black is the opposite of white", and there is a different type of negative like the false statement "never is the opposite of always".

The negative regarding the existence of "God" is the later type of negative. The opposite of always isn't never, it's not always.

Say you see a red car park in front of your house, you can construct a proof that red cars sometimes park in front of your house, by pointing at the red car as your proof. You can't however, say that blue cars never park in front of your house.

"I believe in God because no one has proven that he doesn't exist", is a poor argument for a similar reason.
Printer Friendly | Permalink |  | Top
 
ayeshahaqqiqa Donating Member (1000+ posts) Send PM | Profile | Ignore Mon Dec-13-04 07:12 AM
Response to Original message
13. I find this interesting
because what you are dealing with are concepts of God. Like your understanding we have of the universe. I assume you would agree that science has proven the existance of atomic and sub-atomic particles, since they've seen them via microscopes, etc. What if my concept of God is not of some white bearded man controlling everything, but rather the sub-atomic makeup of everything? That how It works is still to be discovered, and may never be discovered? Would you then say that my concept of God is irrelevant?

Not trying to be antagonistic, just like to debate.
Printer Friendly | Permalink |  | Top
 
Brentos Donating Member (230 posts) Send PM | Profile | Ignore Mon Dec-13-04 12:23 PM
Response to Reply #13
29. Cool!
Very interesting! What if God exists in Schroedinger's Cat and quantum physics (which I'm not fully sure I believe in, but that is a different argument)? That the physical act of trying to observe God alters our perception of God? God is akin to a quantum mirror. If I "believe" God is a white-bearded white guy from the US, my evidence will show that, if I "believe" God is a multi-limbed being who helps me through various lifetimes, he is that; If I "believe" God is a non-entity, but is itself the very fabric of space-time and quantum physics, that is what I'll see.

The key point is belief. Even the atheist has a belief in a concept of "God" (a being/entity/force bigger than I, controlling the universe).

Along these lines, all of us here can agree on the concept of "God". Maybe we just can't agree on Yahweh, Jesus, Buddha, Science, Quantum Mechanics, etc. This would make it a more semantical argument and we can debate further our perception of "God". I enjoy seeing how other perceive the universe and "God". It doesn't threaten my beliefs, and, at times, actually adds to my beliefs.

Thanks everyone for these wonderful debates! Keep them coming! (But please, attach the idea, not the person).

Thanks,
Brentos, the Freshmaker
Printer Friendly | Permalink |  | Top
 
struggle4progress Donating Member (1000+ posts) Send PM | Profile | Ignore Mon Dec-13-04 11:48 PM
Response to Original message
30. Jürgen Moltmann: on Ernst Bloch
<snip> Bloch has written a book about atheism and Christianity ; it first appeared with the subtitle "Only an Atheist Can Be a Good Christian." I mentioned that it should be the other way around: only a Christian can be a good atheist. Bloch then used that statement as the second subtitle of his book. He meant that only an atheist who does not worship false religious and economic gods can be a good Christian. I meant that only a Christian who believes in the crucified Jesus is free from the pressure to create gods and idols for himself. On this issue Bloch and I have come near to each other. <snip>

http://www.darkfiber.com/atheisms/atheisms/moltman.html
Printer Friendly | Permalink |  | Top
 
Stunster Donating Member (984 posts) Send PM | Profile | Ignore Tue Dec-14-04 06:04 PM
Response to Original message
49. Observability, motivation, and atheism
A bunch of thoughts on these issues....

If there are 80,000 people who watch a football game, and they each are asked to give a description of the game, and each description
is different, does that means that there was no football game that they all attended and observed? If it was a game between fierce rivals, the accounts would probably be markedly different. But that wouldn't mean that they hadn't witnessed the same game, or that the game couldn't possibly have taken place.

But hey, why stop at 80,000? Let's suppose the game was the World Cup soccer final, watched by two billion on TV. They all say what they think or write a report about it afterwards. There would be lots of similarities, lots of differences. Does this mean the World Cup
does not exist? Or rather, would the fact that 2 billion people claim to have seen it on TV not count as evidence that there was a World
Cup final, even though it wasn't witnessed by the other 4 billion inhabitants of the planet?

Why can't this be analogous to religious experience? A divine reality, which lots of people experience, but when they give accounts of it, and act in response to it, it comes out markedly different,
though also with considerable commonalities. Not everybody experiences it.

I've never experienced Australia. But if I took the trouble to go there, I think I would. In the meantime, I believe the people who tell me it's there, that they're really filming these pictures there, that this is a photo taken in Australia, etc. Am I wrong to believe them? Is it irrational to think that there really is a place called Australia, and that the many people who say so are not all lying and/or deluded?

Again, Pascal has a lot to say about this. If you never read anything about Australia, or perform any other act that takes the possible reality of Australia seriously, and take no trouble to open up any
means of communication with that country, far less travel to it, and indeed set your mind against even the possibility that there is any such place, then you're not likely to experience Australia. But that wouldn't mean that Australia doesn't exist.

It might be objected that it is possible to travel to Australia for anyone, but lots of people try to journey to find God, and don't. But spiritual journeys of necessity are rather different from physical journeys, though it's possible to get lost along the way even on a physical journey. Nonetheless, there are many accounts of spiritual journeys which proclaim that the traveller has indeed found God.

God, I think, makes his presence known in lots of subtle ways. But these can be ignored or missed. And God doesn't usually intrude on people who want to be left alone. Atheists often accuse religious believers of believing in God because they want to believe (they want a fatherly Protector, they want to live beyond death, they want good to triumph over evil, etc). But the same charge could equally well be levelled at atheists---that perhaps they don't believe because they don't want to believe. Religious belief, if taken seriously, tends to be spiritually and morally demanding. Why would someone want to be burdened by heavy spiritual and moral burdens? Wouldn't it be easier, more natural even, to want the non-existence of God? To want there to be no definitive judgement of one's life in moral terms, to want there to be no higher authority than oneself, to want not to inconvenienced by duties of prayer, church attendance, and to do whatever one feels like doing, as far as one can get away with it? I don't think this applies to all atheists. But it strikes me that in some cases the motivations for their atheism are suspect.

Ever since Marx and Freud, even people who are not Marxists or
Freudians frequently offer those thinkers' theories as to the illusory
nature of religious belief. It is important to see how atrocious these sorts of psychologizing arguments are, as *arguments*. They are, despite their popularity, amzingly weak to the point of being rationally unusable.

Let me quote from philosopher Nicholas Rescher:


Of course, there still remains the well-trodden prospect of
antitheological psychologizing. The general line is all too familiar:
"You see the traditional monotheistic God as desirable merely because
he answers a psychological need of yours. You have a psychological
yearing for acceptance, validation, support. Your God is a mere
parent-substitute to meet the needs of a weak and dependent creature."
So argues the psychologizing opponent of axological theism.
But this sort of facile sort of psychologizing ultimately cuts both
ways. For the axological theist can readily respond along the
following lines: You see the traditional monotheistic God as
undesirable because you find the very idea threatening. You atheists
too are "God fearing," but in a rather different sense. You are afraid
of God. You have an adolescent's fixated fear of and a condemnation by
authority. Your atheism roots in self-contempt. Recognizing what an
imperfect creature you yourself are, you have a fear of being judged
and found wanting. The very idea of God is threatening to you because
you fear the condemnation of an intellige nt observer who knows what
you think and do. You are enmeshed in an adolescent aversion to
parental disapproval.

So runs the psychologizing counterargument. And this line is not
without surface plausibility. Many people are in fact frightened by
the prospect of a belief in God because they ultimately have a
contempt of themselves. They feel threatened by a beli ef that God
might exist, because they feel that, were it so, God would not approve
of them. For them, atheism is a security shield of sorts that protects
them against an ego-damaging disapproval by somebody who "knows all,
sees all." Atheists are not inf requently people on whose inmost
nature the vice of self-contempt has its strongest hold. Pretentions
to the contrary notwithstanding, the atheist's actual posture is
generally not a self-confident independence of spirit, but a fear of
being judged.

In this regard, then, there is simply a standoff in regard to a
Freud-style psychologizing about religion. Those psychologizing
arguments that impute rationally questionable motives that can be
deployed against the believer are not difficult to revise and redirect
as arguments against the atheists. Psychologizing is a sword that cuts
both ways in regard to axiological theism. Both sides can easily play
the game of projecting, on a speculative basis, a daunting variety of
intellectually non-respectable motives for holding the point of view
that they oppose.


And Alvin Plantinga says this:

Freud's jejune speculations as to the psychological origin of
religion and Marx's careless claims about its social role can't
sensibly be taken as providing argument or reason for... the
nonexistence of God; so taken they present textbook cases (which in
fact are pretty rare) of the genetic fallacy. If such speculations and
claims have a respectable role to play, it is instead perhaps that of
providing a naturalistic explanation for the wide currency of
religious belief, or perhaps that of attempting to discredit religious
belief by tracing it to a disreputable source. But of course that
doesn't constitute anything like evidence for God] or a reason to think theism false. One might as well cite as
evidence *for* the existence of God St. Paul's claim (Romans 1) that
failure to believe in God is a result of sin and rebellion against God.


In other words, if it is open to the atheist to speculate about the psychology that underlies theism, it is just as open to the theist to speculate about the psychology that underlies atheism. And of course some theists (e.g. St Paul, Pascal) have done just that. But at least they, unlike some atheists, don't make the blatant logical error of thinking that their psychological speculations are *evidence* or *arguments* for the falsity of atheism. No theist argues that atheism
is false because of the psychology of atheists.

Would that the reverse were true!

An atheist is someone who believes that no God exists. Why *shouldn't* they have to argue for that? If I said no ghosts exist, and someone
says they've seen one, I ought to be able to provide an argument if I reply, well, you're wrong, there is no such thing as ghosts. Let's
say 132 million people say they've seen a ghost, and they mostly seem sane, normally functioning honest adults. Is it then sufficient to say to all of them, "Look, there are no ghosts, and I don't need to provide an argument. Prove to me that there are!"?

I don't think that is sufficient. You have to come up with a credible alternative explanation if you're going to say to 132 million people,
"You're all wrong." Well, far more than 132 million people have claimed to have sensed or experienced or encountered God in some way or other.

In addition, there are the inferential grounds for believing in God. For no matter how much science describes the world, we're still left with the question why is there anything at all, and not nothing?
And why is it this way, producing physical order, life, intelligence, morality, reason, conscious experiences of goodness and beauty and love and of the presence of God?

Being consciously aware that you are communicating with and in the presence of a mind other than your own is normally taken as good evidence that you are communicating with a mind other than your
own. But it is logically possible that there are no minds other than your own, and that everyone else is just a zombie or robot. So that awareness is not logically reducible to observations of their bodies. Yet solipsism is a form of insanity.

So, how do we know we're communicating with other minds? Well, you can make an abductive inference which concludes that there being other
minds is what you are genuinely aware of as it is the best explanation of the phenomena experienced. Or (more philosophically fashionable these days), you can just take belief in other minds as basic, and not inferred from anything else. You seem to be aware of them. You say, I'm aware of other minds. It's a basic belief. And, what's more, it wouldn't be wrong to say you know there are other minds, even if it's not based on an inference.

A thought experiment: if half the people in the world were aware they were communicating telepathically with aliens, and half weren't,
I wouldn't say that there was no evidence for the existence of aliens, even if I was not one of those in telepathic communication with them. I'd be inclined to say that there are aliens
in such circumstances. Even if I didn't understand how telepathy worked, and even if I had no desire to communicate with aliens,
I'd still say that half the human race claiming to be in communication with aliens was decent, albeit not conclusive, evidence for the existence of aliens.

So if half the human race said they were in communication with God, why wouldn't that be analogously decent, albeit not conclusive, evidence that there was a God?

Please don't say that you could meet aliens to check out if they were there. You can't check out a body to see if a mind is there!
Minds are invisible! That's why there is a mind-body problem in the first place! And remember, the thought experiment claimed that the communication was telepathic. So there'd be, ex hypothesi, no physical signal that could be experimentally detected.

Now you'll say, impossible. Telepathy is impossible. Yeah, I agree with you in the case of telepathy---but that is only because most people don't claim to be telepathically communicating a lot of time with space aliens or with people on this planet. But that's not
the point. The point is, not count as evidence?

I can increase the percentages. Let's say 95% said they were communicating with aliens by telepathy, and you were one of the 5% who weren't. Would you still say, "Nope, there's no evidence
that there are any aliens or that telepathy is possible"? Really? If you concede that that would count as evidence (even if it wasn't absolute proof), I suggest you now do a global poll on the frequency of people claiming to be in communication with God by something analogous to telepathy.

And just to make the thought experiment really watertight, we'll imagine that the aliens in question live beyond the event horizon of a black hole, and hence are absolutely unobservable. So there'd be no possibility of seeing them from anywhere else in the universe.

Now of course, I'm not saying that God is a space alien, or that telepathic communication is the right sort of analogy for talking about how humans might communicate with God. (For one thing, God is 'within' us, in a spiritual, non-spatial sense of 'within', and not located somewhere else in the universe because God is not literally localizable in a spatial sense.) But what I am saying is that using observability-in-principle is not an adequate criterion of knowledge. Firstly, we don't literally observe other people's minds, yet we know, in a very reasonable sense of 'know', that there are other minds. And one can imagine circumstances in which such minds wouldn't be observable in any physical sense---space aliens might be able to communicate with us by telepathic means, evidence for which would be the circumstance of most humans claiming to experience their communications. So, it is far from obvious that one couldn't know one was in communication with another mind in general, without this being physically testable.

But if that is so, then there's no reason in principle why we could not be in communication with God, and on that basis know that God exists, even if not everyone was engaging in and experiencing such communication.

And of course, it's just a fact of the sociology of religion, that many, many people claim to have experienced such communication. What I'm suggesting is that there is nothing wrong or impossible in principle with such claims. And secondly, I'm claiming that they are common enough, or sufficiently well-attested (which again, a survey of the sociology of religion demonstrates is the case), that are prima facie evidence for the existence of, and reasonableness of belief in, God.
Printer Friendly | Permalink |  | Top
 
trotsky Donating Member (1000+ posts) Send PM | Profile | Ignore Tue Dec-14-04 09:32 PM
Response to Reply #49
54. Extraordinary claims require extraordinary evidence.
Wow. I really don't know if I should be insulted or simply amused.

Most of us have seen football games, if not one specific game. There is very little doubt that different people will see different parts of the game from different perspectives, because a sporting event is clearly defined, with agreed-upon rules, starting times, and so on. And the various claims of events witnessed at a football game are all easily within the realm of possibility, because a football game is a mundane experience. There is nothing unusual about it.

On the other hand, when people start talking about things like ghosts, unicorns, or "god", we require a bit more proof. There are no standard definitions. There are no agreed-upon rules. That's the problem with all these quaint little analogies theists come up with to try and explain away why no one can agree on IF there is a god, let alone what its characteristics are. Considering that the fate of one's eternal "soul" can be absolutely dependent upon exactly HOW one defines "god," I'd say that should be important enough for an all-powerful deity to dispell ANY confusion about its true nature.

And then we come back to religious bigotry - accusing atheists of being morally lazy. Oh you qualify that you don't mean ALL atheists, but you say their motivations could easily be attacked as being self-centered. Speaking for myself, it was not an easy thing for me to discard religion - it was painful, difficult, and it took several years. But now, the time and money I *could* waste on church is given directly to charities that help people without the religious baggage added on. I have found that most atheists are like that. When you acknowledge that this is the only life we get - that no cosmic grandpappy is gonna make everything alright in the afterlife - morality and social harmony take on an ENHANCED priority, not a reduced one.

Your excerpt from Rescher contains more atheist-bashing. If I posted similar commentary on Christians, I'd be flamed beyond belief on here. I'm not even going to comment on it based on its extreme bigotry. I am well aware it is written to try and counter atheistic criticisms that theists believe out of "comfort." But that criticism at least has some merit, because I'm willing to be that you would admit that religious belief can and does bring comfort to many. I have yet to meet an atheist whose decision not to believe was self-centered as Rescher describes.

The rest of your post consists of several restatements of the argumentum ad populum. Surely if you've been teaching and researching this stuff for 20 years you've heard of that, and how it has no bearing on the relative truth of a statement?

It wasn't all that long ago that the vast majority of people on earth thought our planet was flat. I mean, all their millions of observations pointed to that fact, didn't they? Surely all those people couldn't be wrong! The person who thought in the minority (namely, that the earth was round) should bear the burden of proof, certainly. Well fortunately in the case of the shape of the world, it's quite easy to prove one way or the other. There are actually several ways to demonsrate the curvature of the earth.

But here's what's different: I ask you - how *could* an atheist prove there is no god? You explain how I can do that, and I'll try. But you can't, because proving a negative is impossible. Prove to me that reindeer can't fly. For every reindeer you throw off a building, I can find some excuse why it didn't fly. It wouldn't matter how any people believe they can fly - either they can or they can't. The person making the extraordinary claim - "Reindeer can fly" or "An all-powerful, all-knowing eternal entity exists" - bears the burden of proof. I could give you a hundred ways you could prove god's existence to me. You'll dismiss each and every one of them, of course, because we aren't allowed to "test" god. So you see the no-win situation that atheists face.

You can't provide any falsifiable proof, only simple stories, appeals to emotion - and attacks on atheists and their ridiculous demands for some sort of evidence before they believe. And so you have to find some way to twist it back onto the shoulders of the non-believers - how DARE they demand proof when SO MANY people believe?

One of the most powerful arguments against there being a god is that we are all born atheists. We have to be told that there are gods, what they are like, and what we must do to please them. This is why the vast majority of people on this planet adopt the same belief system of their parents! If this were such a universal truth, wouldn't some of the specifics be genetically programmed into us regardless of our upbringing instead of having such a high correlation with the physical location on earth where we were raised? That would certainly have gone a long way towards heading off most of the religious wars, hatred, and strife in human history, a goal that if I were God, would be one of MY top priorities.
Printer Friendly | Permalink |  | Top
 
Stunster Donating Member (984 posts) Send PM | Profile | Ignore Tue Dec-14-04 11:41 PM
Response to Reply #54
55. It depends on what you consider to be extraordinary
Most people around the world, all through recorded history, have not thought that the existence of supernatural and/or divine reality was an extraordinary claim.

It might, of course, be regarded as extraordinary against the background of a prior disposition to cling to a materialist worldview. But most people haven't been committed to a such a worldview. Anthropologically, religious belief is the norm, not the exception, and the claim that the world has a divine origin has not generally been regarded as extraordinary at all.

Now you might object that it is extraordinary, because materialism is true, or prima facie true. But that's a blatantly obvious example of 'petitio principii'.

Also, your last paragraph strikes me as incoherent. If people have to be told to be religious, how could religion possibly get started, unless there have always been human beings, which of course is not the case?

Printer Friendly | Permalink |  | Top
 
trotsky Donating Member (1000+ posts) Send PM | Profile | Ignore Wed Dec-15-04 12:45 AM
Response to Reply #55
57. In other words...
if I just assume the truth of your claim, I won't see how unfounded it really is! Is that supposed to be an effective argument for faith?

The history of religion is ultimately unknowable. Primitive beliefs arose long before written language, and so we'll never know exactly how they came about. But I think we can make some reasonably educated guesses.

It is clear that early humans had the intellectual capacity to analyze their environment, but nearly total ignorance of how natural processes could account for all of it. Wonder came before knowledge. Our brains, being the pattern-seeking devices they are, ascribed reasons for phenomena whether they were valid or not. Lightning, tornadoes, earthquakes, volcanoes - all awesome forces of nature, but humans had no way of knowing what caused them. What other option did they have than to assume it was some unknown powerful force - maybe a powerful being - behind them?

Sidenote: We do also find atheistic "religions" in history as well - not every tribe observed the world and concluded gods existed. Some were even right here in the Americas. (http://www.darkfiber.com/atheisms/atheisms/native.html)

Labeling your god an extraordinary claim is not a materialistic bias, but instead a simple rationalistic position. For instance, if you believe that Jesus is the Messiah, there are billions of people who will now say that yes, you are indeed making an extraordinary claim. So obviously by manipulating the numbers (as you do) and going back into history one could whittle down into a minority just about every group of people making a claim - yes, even those who think there is but one god. Because as you probably know, monotheism is a relative newcomer to human religions. People believed for millenia in multiple gods! In fact, that was by far the most common starting point for early humans.

So the question we need to ask ourselves is, how do we determine what's true? Clearly the only logical position to start at is the atheistic one - and any claims of a supernatural existence should be subjected to analysis and require some sort of evidence for justification.

I merely ask, "Why believe?" or "Where is the evidence?" Is that a materialistic bias? And your answer to this point is nothing more than, "Because more people (roughly) agree with me." That's an argumentum ad populum - and one that I can turn the tables against you on nearly every facet of your faith.

Again, I ask you, if it's not my place to demand that you defend theism (which you clearly have communicated), could you please lay out a process by which I could disprove it? If it's going to be my responsibility to hit the target, at least tell me where it is and what YOU'RE going to consider a bullseye. Is that unreasonable?
Printer Friendly | Permalink |  | Top
 
Stunster Donating Member (984 posts) Send PM | Profile | Ignore Wed Dec-15-04 01:41 AM
Response to Reply #57
59. No, you don't have to assume the truth of my claim
You just have to stop making numerous question-begging arguments against it.:eyes:
Printer Friendly | Permalink |  | Top
 
Stunster Donating Member (984 posts) Send PM | Profile | Ignore Wed Dec-15-04 01:49 AM
Response to Reply #57
60. I never used argument 'ad populum'
Edited on Wed Dec-15-04 01:50 AM by Stunster
to show that theism is true. I used it to show that your regarding theism as somehow obviously being on the same plane as belief in unicorns is unwarranted.

To then say it's not a materialist bias, but just stating a rationalist position, is amusing. As if the Rationalists themselves were materialists, or atheists!
Printer Friendly | Permalink |  | Top
 
trotsky Donating Member (1000+ posts) Send PM | Profile | Ignore Wed Dec-15-04 08:37 AM
Response to Reply #60
62. However one chooses to use the argumentum ad populum,
it is still invalid. It doesn't matter whether the claim being justified is purported to "explain" something, such as the difference you're trying to make between unicorns and gods. If you base your statement on the "fact" that more people believe it, it's argumentum ad populum. Sorry.

But also, consider this: in every aspect of your life save the religious one, you rely on materialistic, naturalistic, rational laws. You eat food to fuel your body, you drive a car to go places, you sit in a chair that you trust will not break apart, you use a computer, you watch TV, and so on and so on ad nauseum. Every minute of every day you rely on the same "materialistic bias" you accuse me of - EXCEPT for those moments when you think about your god.

Now tell me again who is making the extraordinary claim.
Printer Friendly | Permalink |  | Top
 
Stunster Donating Member (984 posts) Send PM | Profile | Ignore Wed Dec-15-04 05:55 PM
Response to Reply #62
83. I don't know about you, but
in much of my behavior I rely on morality to guide me. That, and conscious thought in general.

Neither strikes me as being material, far less obviously so.

Printer Friendly | Permalink |  | Top
 
trotsky Donating Member (1000+ posts) Send PM | Profile | Ignore Wed Dec-15-04 08:08 PM
Response to Reply #83
85. Nope, but they are products of a material mind.
Gotcha.
Printer Friendly | Permalink |  | Top
 
Stunster Donating Member (984 posts) Send PM | Profile | Ignore Wed Dec-15-04 09:22 PM
Response to Reply #85
88. Morality is the product of a 'material mind'?
:thumbsdown:

You have to do more than simply assume the truth of materialism, otherwise there's no friggin point.


:boring:
Printer Friendly | Permalink |  | Top
 
trotsky Donating Member (1000+ posts) Send PM | Profile | Ignore Thu Dec-16-04 08:35 AM
Response to Reply #88
93. You bet it is.
It's quite simple to see how morality can evolve naturalistically.

Yes, we go back to evolution. A creature with a sense of morality - i.e., concern for its fellow group members - is more likely to survive on two levels:

1) That creature is less likely to be expelled by the group for conduct detrimental to the group. And especially for creatures like humans, group survival was FAR more likely than individual.

2) That creature is more likely to have its genes propagated by the group s/he helps protect. A group of individuals who care deeply about each other is more likely to survive than a group of individuals who don't care as much about each other.

See? A simple, naturalistic explanation for morality. No god or supernatural forces necessary.
Printer Friendly | Permalink |  | Top
 
Stunster Donating Member (984 posts) Send PM | Profile | Ignore Thu Dec-16-04 10:19 AM
Response to Reply #93
96. But then
immorality must also be explained by naturalistic evolution.

Darwinian evolution has to explain both morality and immorality on the same evolutionary basis. Trouble is, this gives us no principled way of justifying morality. The contrast between morality and immorality becomes vacuous.

If something is adaptive (such as killing weaker members of the species) then that becomes the right thing to do. In order to avoid this conclusion, the evolutionist has to say that it's not 'really'
adaptive. But all this does is to make the supposed contrast between adaptive and non-adaptive behavior vacuous. So either the evolutionist
preserves a non-vacuous contrast between adaptive and non-adaptive behavior, but at the price of sacrificing a non-vacuous contrast between morality and immorality. Or vice-versa.

Most philosophers of a scientific naturalist bent see this all too clearly. The honest ones then become strong anti-realists about morality. Which conflicts with our convictions about morality, as
they readily admit. But that's what I said. They bite the bullet. They admit that if scientific naturalism is true, then our deepest moral convictions are not true. Which is what I said about scientific
naturalism's consequences. I now give an argument as to why this is inconsistent with scientific naturalism's basic presupposition in favor of the validity of the data delivered by our conscious minds.

Science is systematically incapable of justifying our moral convictions adequately. The 18th century Scottish philosopher David
Hume and the 20th century English philosopher G. E. Moore both showed
how it is impossible to deduce any moral conclusion from a description
of all the natural facts about the world, even construed broadly
enough to include subjective experiences (such as experiences of
pain). One might point out that shooting a bullet into someone's
head has a high probability of resulting in the person's death. But
it doesn't follow logically that one ought not to shoot the bullet.
One might be able to show that setting small children on fire will
most likely cause them to experience severe pain. But this does not
prove just by itself, as a matter of logic, that one ought not to set
small children on fire. That conclusion would only follow logically
if one added a further premiss of an evaluative rather than merely
descriptive nature, for example, "Causing severe pain without a good
reason is bad."

Some have argued that the terms 'good' and 'bad' can be given a
reductionist analysis and/or definitions in terms of purely
descriptive features of the world. One attempt to define 'good', for
example, is to say that it just means, 'desired (or desirable) upon
reflection.' But the Holocaust was desired upon reflection by senior
Nazis. It would only be possible to exclude the Holocaust from being
judged good if the 'reflection' itself was to include moral reflection. But then you'd have a vicious circle. Even if one could refine one's attempt at defining 'good' so as to exclude such counter-intuitive outcomes without circularity, there would still
arise the question, why *ought* we to do what we desire or find
desirable upon reflection? Why is *that* something that binds our
conduct? Why is it morally wrong to do things which we don't desire
upon reflection? We may not, upon reflection, desire to spit at
strangers (they might harm us if we do). But just because our rational instincts tell us to do or not do X, why is it morally obligatory to pay attention to our rational instincts?

Many naturalistic thinkers will answer the last question by saying
that *all there is* to moral obligation and value is the functioning
of rational instincts and desires. The first problem with this reply
is that instincts, dispositions, and desires vary tremendously among
humans--some are instinctively aggressive, others instinctively
deferential and compliant, some are extremely egoistic and cruel,
others loving and altruistic. They vary from ethnic cleansing to
caring for lepers. The second problem is that if reason (the 'rational' part of 'rational instincts') is only instrumental---that
is, if reason only enters the picture as the process by which agents
deliberate about and choose between various possible *means* to their
various ends, then the naturalist is left having to face the fact that
some people's ends are truly horrifying from a moral point of view.
But in that case, one can't reduce morality to the ends people are
disposed to pursue.

If, on the other hand, reason enters into the picture by actually adjudicating which *ends* ought to be pursued and which ought not to be, then one is back in a vicious circle. One has smuggled moral reason and moral judgement in to sort out the varying ends between which the naturalist, contemplating a factual description of the great variety of people's dispositions and desires, must choose in order to give any remotely plausible account of the content of morality.

The biological perspective is simply that people have different urges
to do different things. But biology provides no criteria for deciding
why one set of urges should be labelled more `moral' than another.
We would be left describing the atrocities of the Pol Pot regime as
yet another `interesting' manifestation of humankind's factual
dispositions.

Similarly, the attempt to derive morality from evolution is logically
flawed. Evolution is simply a descriptive theory. Morality is a
*prescriptive* theory---it PREscribes certain kinds of conduct for
humans, and PROscribes others. But if evolutionary biology is to
explain morality, it must show the link between morality and adaptive
behavior. The trouble with this is that a very large range of human
behavior is agreed to be immoral, while evolution has to hold that
nearly *all* behavior derives from the adaptive features of our
genetic makeup. From this it would follow that much, perhaps even
all, immoral behavior is adaptive. But then adaptiveness cannot be
that in terms of which moral behavior is defined, or that from which
specifically moral (as against immoral) behavior springs.

Some naturalists are prepared to bite the bullet about this. That is,
they are ready to say that there is no such thing as morality in any
robust sense. There are just human wants and human inclinations to
talk a certain way about them. Morality, if the term is to be
retained at all, simply refers to whatever happens to be the majority
of, or most commonly possessed, sets of dispositions and ways of
speaking with regard to inter-human conduct.

The problem with this view is that it falls foul of naturalism's most
basic starting point---human experience. Naturalism privileges
science as a form of knowledge because it relies on the most immediate
data yielded by our consciousness of the world. Among these data are
most certainly the deliverances of our sensory and perceptual
abilities. We experience a patch of green and call it grass. We
hear a sound and interpret it as indicating a wave is moving at a
certain speed through a large body of air molecules. We look at a
dial and determine by its measurement the mass of a subatomic particle.

But these are not the only kind of data of consciousness. There is
also the utter conviction that shooting defenceless innocent children
as they attempt to escape is something we are morally bound to
condemn---that it is *prohibited* to act thus, whether anyone wants to
or not. There is the absolute certainty we find our conscious mind
giving us that leaving a man to die of thirst in the desert while
driving off in a full water-tanker is an abhorrent act of callousness
that violates an ineluctable moral obligation (unless one is racing to
save the lives of others who would die if you stopped to help---in
that case one's obligation is different--saving the others--but it's
still an obligation).

In other words, naturalism rests its case on the sheer force and
given-ness of sensory experience. But that force and given-ness is
at least, if not *more*, present in the case of people's consciousness
with respect to major moral duties and moral values. One is *more*
ready to attribute an experience of green to optical illusion or
bodily malfunction (such as color-blindness) than to give up as
illusory the idea that one must not kill kids or leave dying men in
the desert. One is *more* ready, in a laboratory, to attribute the
position of the dial to a random electrical disturbance rather than
the properties of the object being studied, than one is to attribute
the notion that we should not rape our grandmothers to a mere lack of
desire to do so.

Naturalism, in order to dismiss morality as a projection or illusion
with no real objective claim upon us, ends up having to deny the
validity of the only thing that would even render itself
(naturalism) plausible in the first place---the deliverances and
character of the subjective conscious experiences of human beings.

Naturalism is therefore false because it has to be. Not 'has to be'
in the sense of wishful thinking that it is not true. No, 'has to
be' in the sense of violating the demand of reason that what *is* true
cannot be so multiply incoherent and self-refuting.



Printer Friendly | Permalink |  | Top
 
trotsky Donating Member (1000+ posts) Send PM | Profile | Ignore Thu Dec-16-04 10:48 AM
Response to Reply #96
101. What exactly are you cutting-and-pasting from?
I was just wondering, because your posts have a tendency to break into formatting differences. If you are copying from someone, you should probably give the source credit. Maybe it's from things you've already written, and that's cool, I'm just curious.

Anyway, to your point about immorality. Easily explained as well. An organism's primary goal is survival. Secondary, but close to it, is reproduction. Morality is something that has a high rate of correlation to each of those, but not a 100% correlation. There is "wiggle room," so to speak. An individual who sees an opportunity to advance its own survival, or even reproductive ability, at the expense of the group (especially if the likelihood is high that the group will be unable to determine s/he did it) has a good chance of carrying through with that.

It's much like the evolution of capitalism. Adam Smith's "invisible hand" works in many cases - but not all. Market forces generally work to keep supply & demand in balance and competition alive, but sometimes opportunities arise and people take advantage of them in defiance of the invisible hand.

Alternately, we need to consider how human groups and society have changed over the years. For most of our 2 million year existence, we were almost entirely dependent on group survival. Our morality grew out of that time. Only recently, since the development of larger societies, has it become less likely that decisions going against the group will result in the individual's not surviving or failing to reproduce. It became much easier to wreak havoc in one group and then find another one to move into. Such a tactic could actually confer a reproductive advantage in some cases, and thus it has some evolutionary value.

There are competing forces at work, since both morality and immorality can confer natural selection advantages. But then so can light skin and dark skin. It all depends on the environment. We don't point to the simultaneous selective advantages of light or dark skin as invalidating the natural selection of skin tones, so why should simultaneous selective advantages of morality or immorality invalidate the naturalistic origin of morals?
Printer Friendly | Permalink |  | Top
 
Stunster Donating Member (984 posts) Send PM | Profile | Ignore Thu Dec-16-04 10:52 AM
Response to Reply #101
104. My own past emails (n/t)
Printer Friendly | Permalink |  | Top
 
Stunster Donating Member (984 posts) Send PM | Profile | Ignore Thu Dec-16-04 10:55 AM
Response to Reply #101
105. I should add
that they are emails to a yahoo forum called ncchuddle (where anything other than Celtic Football Club can be discussed). 'Ncc' stands for No Celtic Content. There are other forums for that.

And if you read through my emails here on DU, you'll see that I do credit the source when I'm quoting---I've done so NUMEROUS times, in fact.

Printer Friendly | Permalink |  | Top
 
Stunster Donating Member (984 posts) Send PM | Profile | Ignore Wed Dec-15-04 01:38 AM
Response to Reply #54
58. If I were positing unicorns
to account for the phenomena of reason and value, then perhaps you'd have a point.

But what is at issue is not whether unicorns best explain those phenomena. It is whether taking one's fundamental ontology to comprise material reality, or taking it to comprise God, best explains those phenomena.

And no amount of bluster makes it 'obvious'---obvious in the way that it's obvious there are no unicorns---that the materialist worldview explains them far more cogently than a theistic worldview. Indeed, it is very far from obvious how to account for reason and value from a materialist starting point.

Personally, I think the idea that blind matter operating by sheer chance generates binding moral or rational norms is very much in unicorn territory. I also think that the existence of God is far more obvious and evident than the truth of materialism.

Atheists seem to think that it is 'obvious' that the existence of God is not obvious (or that God's non-existence is obvious), and that the truth of materialism is obvious. But actually, there's no obvious reason to think any of those things. If it were that obvious that there's no God, and that obvious that materialism is true, then most people would be atheistic materialists. But they're not. And so the atheist needs to show more than simply beg the question as to the supposed obviousness of materialism, and the supposed obviousness of the non-existence of God.

It is sometimes thought that arguments for theism are like a goldfish thinking that he was created by a transcendent Big Goldfish living outside and beyond the goldfish bowl. But this is to miss the point altogether. If the goldfish thought that there must be a Big Creator-Goldfish, that would be analogous to a human being thinking that there must be a transcendent Big Human Being who created him. But theists don't think that! (God is an immaterial spiritual rational nature, says the theist.) The key property to be explained by the divine creator hypothesis is not the physical nature of the being doing the explaining, but the fact that the being making the inference (or doing the explaining, be it goldfish or human) is possessed of a rational nature---which of course, as a matter of fact the goldfish isn't. But if the goldfish were sufficiently rational to make inferences about what created the goldfish bowl and itself, it would likely conclude not that there was a transcendent, creative Goldfish, but rather that there was a transcendent, creative, rational intelligence. And human theists don't infer that there is a transcendent, creative human being, but that there is a transcendent, creative rational intelligence.

And if goldfish were moral creatures, capable of understanding morality as we do, they would probably add to their inference that the creator was a moral being too.

Now, while it would be silly to suppose that the creator must be a goldfish, or a human, it would not be silly to suppose that the creator must be supremely endowed with reason and value, and indeed the source of reason and value in the world. The criticism that theism rests on anthropocentric self-favoring prejudice is itself one that purports to rely on reason. If one engages in rational argument about theism, one is presupposing the validity of reason. So, while the atheist can rightly dismiss the mental provincialism that would make God anthropomorphic, the atheist cannot so easily dismiss the idea that the origin of the humanity and the world is rational. For if it is not, then his own reasoning processes are the mere product of random chance, and thus he risks undermining the reliability of reason as such. (See below).

But if the atheist insists that reason is a reliable guide to reality as a whole, then he is admitting that reason is not just one feature among others that call for explanation, but a very central feature. So the choice of a being supremely endowed with reason, as the conclusion of an inference to the best explanation, is not at all arbitrary, and is not really species-specific in this sense: that if goldfish were possessed of it, reason, not goldfish-hood, would be the feature they'd home in on in to characterize the nature of their creator.

Materialism holds that those phenomena arise from matter, which is not as such endowed with reason or value. Theism holds that reason and value are ontologically basic, and that matter is designed by reason for a purpose, in order to realize certian kinds of value. It is not at all obvious to me that materialism is true, or even could be true. In fact, for myself, it seems rather obvious that materialism is false. So the idea that theism is as extraordinary a claim as that there are unicorns strikes me as gratuitous bullshit.

Here are some thoughts about the difficulties of reconciling naturalism with a belief in the validity of reason:

THE PARADOX OF THE STONE

Let's explore another example, this one from recent history, proposed
by Richard Taylor in his small but important book, Metaphysics. Taylor
asks us to imagine a stone that's just been dug out of the ground, and
covered by peculiar markings. You suppose that these markings are
accidental, and simply the result of millions of years of erosion,
until a friend of yours who happens to be a professor of ancient
languages arrives upon the scene and promptly renders a translation of
as follows: HERE KIMON FELL LEADING A BAND OF ATHENIANS
AGAINST THE FORCES OF XERXES. Now one can, to be sure, still maintain
that the marks are accidental, that they are only scratches left by
volcanic activity, and that it is only a singular coincidence that
they resemble ... some intelligible message. Nature sometimes produces
effects hardly less interesting and arresting as this. The point ...
however, is this: if anyone having a knowledge of this stone
concludes, solely on the basis of it, that there was someone named
Kimon who died in battle near where this stone was found, then he
cannot, rationally, suppose that the marks on the stone are the result
of the chance or purposeless operations of the forces of nature. He
must, on the contrary, assume that they were inscribed there by
someone whose purpose was to record an historical fact.5

Here, Taylor is not claiming that nature is the result of purposeful
design, or requires a designer. It is entirely possible for the rock
to have accumulated various and peculiar markings during vast periods
of time, and that these markings are purely accidental and not
purposefully inscribed. However, it would be a grave mistake, says
Taylor, to also believe that these markings "reveal some truth with
respect to something other than themselves"6 about the world. In other
words, the markings cannot be both the result of chance forces and
indicative of any truth beyond the mere fact that there happen to be
peculiar markings upon a certain stone.

COSMIC IMPLICATIONS

Taylor discusses this example within the general context of human
rationality: how is human consciousness any different from the
accumulation of accidental markings upon a stone? Perhaps, given
enough time, nature is bound to evolve something as marvelous and
astonishing as human consciousness. But we cannot have it both ways,
says Taylor. We simply cannot consistently claim that human
consciousness is both the chance outcome of blind, accidental causes
and a reliable belief-forming apparatus by which we discern truths
about the world.

Now, one could mend this dilemma by saying that none of our beliefs is
ever reliable or true. But notice that the truth of this previous
statement entails that at least one of our beliefs is true, namely,
that none of our beliefs is ever reliable or true, which is apparently
self-contradictory. Perhaps one might choose the other horn of the
dilemma, and abandon the naturalistic view of a blind, purposeless
world altogether.

CONSCIOUSNESS IN THE CAVE

Bertrand Russell once said, "Man is a part of nature, not something
contrasted with nature. His thoughts and his bodily movements follow
the same laws that describe the motions of stars and atoms."7 If this
is true, then there can hardly be any grounds for distinction between
the blind, accidental forces of nature, and human consciousness. If
our belief-forming faculties are simply a function of blind,
accidental nature, it would certainly follow that the deliverances of
our belief-forming processes are just as blind and accidental.

Yet, how could we ever come to know (via accidentally-caused
belief-forming faculties) that our belief-forming faculties are the
result of blind, accidental causes? This also appears to be
self-contradictory. Knowledge implies a certainty about the way things
are that is not blind or accidental. In order to achieve this
certainty, it seems human consciousness would require the ability to
"step outside" or otherwise transcend the blind, accidental nature of
the system in order to objectively evaluate that system. But this is
just what naturalism denies: nothing transcends the system.

Why not simply allow for the possibility that human consciousness is
not exclusively the result of blind, accidental causes? If
consciousness is a part of nature, and consciousness is not blind --
could it just be possible that not all of nature is blind accident?

WHY I AM NOT A NATURALIST

Although I believe the arguments I have just developed are intuitive,
they are not very strict. But what I have hoped to show is that it is
not obviously clear that naturalism makes for a coherent worldview. If
reality is just cosmic, expansionary forces, what about things like
numbers, or ideals like justice? It is not clear how these could be a
contingent, expansionary function of matter; but yet the naturalist
must somehow maintain that they are. Wouldn't it just be better to
admit that some features of the world do not seem to be subject to
change or the result of blind, accidental causes? Quite frankly, there
doesn't seem to be anything foundational enough within the landscape
of naturalism upon which to establish the unchangeableness of things
like number, or justice. For, according to evolutionary theory,
everything is subject to change, to progress -- except for,
apparently, the doctrine of naturalism itself.
Printer Friendly | Permalink |  | Top
 
trotsky Donating Member (1000+ posts) Send PM | Profile | Ignore Wed Dec-15-04 07:47 AM
Response to Reply #58
61. And if your god actually explained phenomena, you'd have a point.
But "god" only adds a unwrranted layer of complexity to the problem.

Did the universe arise through naturalistic processes, or did it evolve from some point AFTER an eternal, uncaused, infinite, unknownable, unseen "god" created it?

What exactly does postulating a "god" explain? Anything?

Goldfish analogy: Umm, hate to break it to you, but the Christian religion is focused around a god-man. For most of its history, theism has assumed gods to be quite like humans, only extra-powerful. Only recently, when faced with the inherent weakness of viewing gods as super-humans has theism come up with new concepts of things like a "transcendent, creative rational intelligence."

Every observable, repeatable experience in our lives is subject to rational processes. It is the theist who proposes that there is one thing so out-of-the-ordinary, so supremely infinite, that we have nothing else in life to compare it to. It transcends all, is responsible for all, and knows all. That's quite the claim, and it has nothing whatsoever to do with unicorns.

The burden of proof is uncomfortable, but it is all yours.

Unless you can, as I have repeatedly asked you to (and you have repeatedly ignored), provide some sort of criteria by which I can "prove" atheism. Can you, or are you admitting that putting the burden of proof on the atheist is logically impossible?
Printer Friendly | Permalink |  | Top
 
Stunster Donating Member (984 posts) Send PM | Profile | Ignore Wed Dec-15-04 10:36 AM
Response to Reply #61
64. WTF?
Edited on Wed Dec-15-04 10:52 AM by Stunster
Think about how we explain things!

You are on a train from London to Glasgow. Just after Carlisle, you see a sign that says WELCOME TO SCOTLAND. Is your immediate inclination to think the sign being there is best explained by purposeful, conscious intelligence, or by sheer random chance movement of atoms, or by some cosmic law or computer code that inexorably dictates that the sign will be there?

If you pick the first of these, are you saying that's not a genuine explanation, because it introduces an 'added layer of complexity'????

WTF????

Look, suppose one is asked to explain matter. And suppose you say, 'Matter is ontologically basic'. That's an explanation! Positing a fundamental ontology is a form of explanation. Materialists have their fundamental ontology. Theists have theirs. Now relative to the fundamental ontology of materialists, it is quite hard to explain, say, the existence of rational conscious minds--in fact, the New Mysterians, David Chalmers, Galen Strawson, etc say that materialism can't explain it. Relative to the fundamental ontology of theism, the existence of rational conscious minds is not impossible to explain, nor is it surprising. But notice that in neither case is that which is ontologically fundamental explained in terms of something ontologically different. That's just what it means to say that something is ontologically fundamental! Sheeesh!

Now, atheists want to say something like this:

"Why can't we just accept that the universe is the way it is, and
leave it at that? Why does there have to be a reason?"

Now, it's important to unpack the underlying assumptions here. As
far as I can see, there may be one of two possible assumptions
operating in this response:

1. One possible assumption is that whatever causes the physics of the
universe to be the way it is, the resulting physics is *necessitated*
by the cause, whatever it is (though they're sure it's not God). That is, as a matter of fact, no other physics is possible. It has to be that way, because the physics-selection mechanism only admits of one possible selection. No alternative physics is even possible. That's just the way it is.

And hence, when the atheist questioner asks, why does there have to
be a reason, he actually means that there is a reason: the reason the
physics is the way it is is because it has to be. That's a reason.
To say of something which you are asked to explain, "it was necessitated" is to explain it. That counts as giving an explanation. The water had to go downhill. Gravity necessitated its course. That's an explanation. That's a reason why the water flowed where it did---it didn't have any other option. Now behind the necessitating mechanism there may stand something else. But ultimately, there may be nothing behind it. After all, all explanations are finite. They come to a halt. And one is making the not unreasonable assumption that all apparent series of explanations that constitute an infinite regress, are not in fact genuinely explanatory of anything at all. And so, says the atheist, why can't there just be a necessitating, but non-divine, mechanism that generates with inexorable inevitability the laws of physics that actually obtain? Ok, that's the first interpretation, and I'll come back to it.

2. The second possible underlying assumption is that it was just
sheer random chance that was responsible for this outcome, but *many
other outcomes were possible* . There *could* have been many, many
ways for the physics to be. Many sets of physical laws that would
govern a universe are possible. There was no reason that one should
obtain, rather than any other one. All were possible. Only one got
realized, and it's just a fluke that it happens to be a physics that
is suitable for intelligent life, etc. In that sense, there was no
reason why it happened that way. It could have happened any other
way. But it just happened this way.

Ok. Notice that these are two quite different ways of framing the
atheistic response. In one case, only one physics is even possible,
and furthermore it is necessarily instantiated. In the other, lots of different physics are possible, and chance alone made it this one rather than any other one. It is this second interpretation that is targetted in the theistic argument on the basis of the scientific findings about the improbability of life emerging by chance. When
one says that the odds against the actual outcome are astronomical,
one is saying that they are astronomical if it's due to chance, and if it's the case that *other universes might have emerged instead*, most of which would not have supported life. *That* is the target of the straw-drawing analogy. The straw-drawing analogy obviously doesn't apply to the first interpretation, the necessitarian one. In the straw drawing analogy we're assuming that the person could have picked out any straw, and then imagining that he just happened to pick out the
shortest one. Applying this to the argument, we would say that any
universe might have sprung into existence---it just happened to be one
of the very few possible ones capable of supporting life. The theistic argument is that this is mind-bogglingly improbable, in just the same way that it would be mind-bogglingly improbable to pick out the shortest straw from a batch of 10 billion---AND THAT THE RATIONAL
INFERENCE TO MAKE IN THAT CASE IS THAT THE DRAWING WAS IN FACT
DESIGNED TO ENSURE THE OUTCOME, SINCE THAT IS FAR MORE LIKELY THAN YOU
JUST GETTING LUCKY.

The straw-drawing analogy does not apply at all to the first
interpretation. So let's examine the situation if one adopts a type-1 assumption. What that assumption in essence says is that only one type of universe is possible, and the universe is necessary. Now, as mentioned, that IS an explanation. When we say that only one answer to a math problem is possible, and the math necessitates the answer, we're not avoiding an explanation or giving a reason. We're saying something quite explanatory.

Now most physicists reject a type-1 assumption, because they seem to
think that many models of alternative physics are perfectly coherent,
from a physics point of view. So this is some scientific evidence
that a type-1 assumption is actually invalid. But let's suppose the
physicists in question are mistaken. Let's suppose that only one
physics is possible, and that physics is necessitated by a non-divine
necessitating physics-instantiator.

Now, previously, I have suggested that this is implausible because
such an instantiator would have to exist in a non-physical abstract
realm--after all, it is producing physics, so it can't already be
itself physical, since to be physical implies that the physics has
already been instantiated, and there is a general metaphysical
principle which says that if X causes Y, X cannot be identical with Y.
So I am imagining that this physics instantiator is like a computer
program. It is software, not hardware, a sort of cosmic code. Ok, the obvious question is, how does the software run? What makes it do anything causal?

In the real world, you need hardware to make software do anything.
But in the hypothesis being considered, there's no hardware to run it
on---the software has to be running already, on its own. And so we
can see, I think, the difficulty of conceiving of a Platonic or
abstract entity causing a physical world to come into being. But
I'll ignore that for now. (By contrast, we are very familiar with
conscious minds making decisions which result in our body moving.) And if the atheist is going to have to retreat from materialism into Platonism ('the cosmic laws are eternal and necessary', 'the Eternal Form of the physical underlies and generates the physical world', it makes you wonder why they think this is preferable to positing an Eternal Mind, since we only ever encounter abstract entities as the contents of minds.)

But I think there's an even more basic problem with the non-divine
physics-instantiator; it is that it is devoid of rational consciousness, and devoid of value. And hence I am strongly inclined to the view that such a thing is inherently unintelligible.

By that I mean that we couldn't really understand it. It is, in a sense, opaque to reason. Nothing in the physical world is intelligible to us except insofar as it is governed by intelligible laws and other mathematical relations which we understand. But those laws and relations are not entities out there in spacetime, they are in our minds. But a physics-instantiator which was just there, not contained in any mind, and thus devoid of reason and value, but simply as a brute, motiveless, reasonless, purposeless abstract but inexorable fact, necessitating the physics we have---that wouldn't seem to have anything understandable about it. Our minds would still want to scream, 'But why?' And this hypothesis is saying that there's no why.

In other cases where we explain something by saying that it's
necessitated, we understand the rational relation, 'Given this, then
that has to happen'. If you understand the natures involved, you
will make the connection, and find it satisfying to the intellect.
But that's because you're not at that moment asking why about the
'given' part. Ah yes, I understand gravity, and I understand water,
and hence I see why the water has to flow downhill instead of uphill.
Fine, but now ask a tougher question: why is there any such thing as
gravity? Ok, maybe there is an answer (possibly to be yielded by
string physics). But now you're getting to rock bottom of the
ontological pile. And what do you come up against---well, assumption
1 says you come up against something whose sole nature consists in
necessitating the instantiation of the actual physics that obtains in
the universe. But such a nature, if it is truly ontologically
ultimate, is, I'm suggesting, inherently unintelligible. The thing
is devoid of purpose, value, sense, meaning, life, consciousness and reason---the very phenomena we're most interested in explaining.

Mind, by contrast, is intrinsically intelligible, and that's because
mind is precisely the locus inhabited by purpose, value, sense, meaning, consciousness and reason.

Now we might not know how the mind arises from the brain, etc. But
even if we don't know that, we do understand our own minds in the
sense that we understand our own mental contents. We understand
thoughts and emotions, reasons and meanings, numbers and logic,
principles and values----we understand what it is to understand, because we are directly acquainted with it every time we understand something.

Mind can also understand matter (up to a point). But matter can't understand mind. Nor do abstract entities understand anything, but rather are the objects of the mind's understanding.

Mind can design things, from a shovel to a spaceship to a software
program. Can matter operating by random chance create a software program?

Seems to me, then, that if you're looking for the nature of the
ultimate explanatory reality, then prima facie at least, mind has a lot more going for it.

And that's why most people believe in some kind of God, I suspect.
The only thing that ultimately accomodates intelligibility is
understanding. And understanding is a property of mind, not of
material objects or of abstract entities. Atheism fails as an
understanding of the world precisely because what it does is to deny
that there is anything ultimately understandable about it. It is
saying, in effect, that there is no ultimate understanding to be had,
and thus, that at bottom, that reality is unintelligible.

Now reason itself rejects this. Reason by its very nature demands that the objects of reason, including reason itself, be intelligible, and rejects the unintelligible as being not truly real. In other words, it makes intelligibility a *criterion* of reality. Nihilistic celebration of unintelligibility goes against the grain of our own rational nature, or against reason itself. One can still choose to go that route. But the only *rational* route to choose, is the road that leads to theism, because that, by positing mind---and hence, meaning, reason, value, logic, understanding, thought, consciousness---as that which is ontologically fundamental, is the sole way to secure the *ultimate and complete* intelligibility of reality.

Let me quickly pre-empt one objection. It will be objected that
human minds cannot fully understand God. But that does not threaten the complete intelligibility of reality, because on the theistic hypothesis, God understands God---and hence everything is understandable, even if it's not understandable by everyone.

Let's return to the WELCOME TO SCOTLAND sign. Positing a conscious, purposive, causally active intelligence to explain it is perfectly natural. It's also a good, indeed correct, explanation of the sign.
By contrast, if we say it got there by sheer random movements of atoms, or by some necessitating cosmic computer code, we'd be deeply puzzled. But by postulating rational consciousness as the explanation, we're immediately satisfied. We don't feel a need to go further. We assume that mind is fundamental. Nobody starts off as a materialist about the mind. We think, we feel, we reason, we intend, we understand. Now the materialist comes along and does something supremely ironic. He demands that we genuinely explain the world, and says that God doesn't genuinely explain it, because God is too high and complex. He demands, in other words, that what we posit as our fundamental reality be genuinely intelligible. Then he posits something that is utterly devoid of thought, reason, intention, understanding, meaning, purpose and value---in other words, something that's about as unintelligible to rational minds as one can get except insofar as it obeys the mathematical thoughts we have in our minds about it---and says, see, that's what you should have posited to provide genuine explanation and intelligibility. Don't posit something that's too much like a mind! Minds need to be rendered intelligible by something that's not a mind--that's the only way! As if minds could be rendered intelligible by something not a mind, when in fact the idea that rational thought is caused by the random chance movements, operating without purpose, design or reason, of material atoms, is possibly one of the most unintelligible ideas anyone has ever proposed!

As I say, the irony is supreme.


Printer Friendly | Permalink |  | Top
 
trotsky Donating Member (1000+ posts) Send PM | Profile | Ignore Wed Dec-15-04 11:04 AM
Response to Reply #64
66. Ack! The Watchmaker argument again?!?
Proposing that someone made a "Welcome to Scotland" sign is the NORM. THAT is the naturalistic explanation. I know that you desperately need to create your strawman to say that the naturalistic position means that every phenomena must first be considered to be a totally random occurrence, but it just ain't so, no matter how much you try. I'm sorry to have to be the one to break it to you.

Since you refuse to address any of the other points I brought up but instead go with the "verbal overload" approach to try and avoid them entirely, I consider this discussion at an end. You have demonstrated no evidence whatsoever why theism should be considered anything but an extraordinary claim but instead engage in one logical fallacy after another out of sheer desperation.
Printer Friendly | Permalink |  | Top
 
Stunster Donating Member (984 posts) Send PM | Profile | Ignore Wed Dec-15-04 11:25 AM
Response to Reply #66
69. Ack! The giant automated watch-making factory argument again
"Ha", says the atheist, "the Watchmaker argument again! How much more obvious, intelligible, and rational it is to posit as an explanation for the existence of watches, not a conscious, intelligent watchmaker, but rather a giant, automated watch-making factory that simply got to be there by the random chance movements of molecules, and atoms. That's a much better explanation."

Everybody else says, "WTF?"

Thank you for ending the argument.
Printer Friendly | Permalink |  | Top
 
trotsky Donating Member (1000+ posts) Send PM | Profile | Ignore Wed Dec-15-04 12:22 PM
Response to Reply #69
73. Let me correct your misstatement.
It matters not where the watch came from - the primary objection to the watchmaker's argument is that making a watch is completely different than making the universe. Your analogy fails mightily.
Printer Friendly | Permalink |  | Top
 
Stunster Donating Member (984 posts) Send PM | Profile | Ignore Wed Dec-15-04 11:15 PM
Response to Reply #73
91. Analogy, schnalogy
making a watch is completely different than making the universe. Your analogy fails mightily.

Designing a watch is different from designing an aircraft. It doesn't follow that they are not analogous on the dimension of designing.

The analogy is along the dimension of 'appearing to be designed'---watches appear to be designed, and so does the universe.

In every analogy, of course, there are differences between the items being analogously compared. We say things like, driving a car is analogous to riding a horse. But it would be ridiculous to say that it's not analogous because a horse is a living animal and a car isn't.

As for pre-existing materials: one might as well say that the Big Bang couldn't have produced the universe, because according to Big Bang cosmology, there were no materials pre-existing the Big Bang. You might reply, ah, but there was something in existence at the Big Bang. Well, first of all, that's a matter of some considerable speculation--there have been a number of cosmologists who say that the Big Bang emerged as a quantum fluctuation out of 'nothing', though the sense of 'nothing' being invoked isn't always clear. But even if you insist that there was never nothing, and that at the Big Bang, there was something, the theist could say (as Saint Augustine essentially did), I agree with you. At the creation of the universe, there was something, not nothing, and there was no 'before the creation' at which there was nothing. And now Hawking pretty much says the same. As far back in time as we can go, there was something, and never nothing.

So, you are wrong to suppose that there was a time when God had nothing to work with. First, there was never a time when there was physically nothing. Second, there was never a time from God's point of view when there was nothing either, since God's point of view isn't a temporal point of view. One therefore should not imagine that God was looking at no world, and then came a moment in time when out of that no-world state of affairs, God made a world. As I say, Augustine understood this---15 centuries ago.

To create is to cause to be. The watchmaker causes the watch to be and to have the form it does. Analogously, God causes the universe to be and to have the form it does.

Printer Friendly | Permalink |  | Top
 
trotsky Donating Member (1000+ posts) Send PM | Profile | Ignore Thu Dec-16-04 08:31 AM
Response to Reply #91
92. Let me try once again.
Your analogy fails because a watch is made from pre-existing materials. We are familiar with watch design, we've all seen watches, some of us have even seen them made.

You propose the universe was created ex nihilo by your god. No witnesses, nothing.

If you can't see how different those two methods are, I'm sorry. But it still invalidates the watchmaker analogy.
Printer Friendly | Permalink |  | Top
 
Stunster Donating Member (984 posts) Send PM | Profile | Ignore Thu Dec-16-04 10:22 AM
Response to Reply #92
97. 'No witnesses'? That's your answer?
:boring:
Printer Friendly | Permalink |  | Top
 
trotsky Donating Member (1000+ posts) Send PM | Profile | Ignore Thu Dec-16-04 10:34 AM
Response to Reply #97
98. Yup.
I can produce plenty of witnesses who have seen a watch made. But you can't find me any who saw your god create the universe. This says nothing about the validity of your claim, only that it's completely different than creating a watch, and thus once again, that the watchmaker argument (from design) is exposed as bogus.
Printer Friendly | Permalink |  | Top
 
Stunster Donating Member (984 posts) Send PM | Profile | Ignore Thu Dec-16-04 10:42 AM
Response to Reply #98
100. I've decided
that if this represents the intellectual level of your contribution to the debate, there's no point in my debating you.

It's up there with the Soviets claiming that when Yuri Gargarin became the first man in space, he did not see God, thus 'proving' that there was no God in heaven.

Witnesses to the creation? WTF?
Printer Friendly | Permalink |  | Top
 
trotsky Donating Member (1000+ posts) Send PM | Profile | Ignore Thu Dec-16-04 10:51 AM
Response to Reply #100
103. Let me try once again.
In this specific branch of our discussion, I am not talking about "proving" or "disproving" your god.

I am only pointing out how the watchmaker argument (and it alone) FAILS to prove your god. I am saying nothing about anything else.

Understand yet?
Printer Friendly | Permalink |  | Top
 
Ando Donating Member (112 posts) Send PM | Profile | Ignore Wed Dec-15-04 10:51 AM
Response to Reply #61
65. Fides et Ratio
My simplistic analysis of the argument at hand is this: There never was nor will there ever be a rational argument for the existence of God. If God could be found by pure reason, He would cease to be extraordinary. To claim victory in this argument, I don't have to convince a rationalist to believe in God rationally, I have to convince the rationalist to place spirituality and the supernatural on the same plane as reason. The biblical worldview is at odds with modernism and post-modernism in our culture today. An example of this is the umpire analogy, I can't remember who came up with it. The modern (rational) umpire calls them as he sees them, the post-modern (secular humanist) umpire calls them as he wants them, and the umpire of orthodoxy calls them as they are. I guess the argument is that the rational and the extraordinary must be intertwined to create truth. Chesterton calls it the "Ethics of Elf-Land", the wisdom of faerie tales that accounts for the wonder of life. These aren't arguments of logic, these are fundamentally different ways of viewing the world. What might be more helpful is to explain why we think the way we do rather than to try and defend our thoughts under a different framework.
Printer Friendly | Permalink |  | Top
 
trotsky Donating Member (1000+ posts) Send PM | Profile | Ignore Wed Dec-15-04 11:10 AM
Response to Reply #65
67. You can't claim victory.
The most a theist can hope for is a stalemate. For as soon as you open up your god to testing, it can be dispelled. Instead, you hide it behind terms like "supernatural" and "spiritual" and declare it off-limits to the rules that govern every other aspect of our lives, rules that you accept and live by.

If god exists but works in the universe through "miracles" or "spiritualism" (violations of or exceptions to natural laws), then those miracles become natural laws, as they would be observable.

If god exists and works in the universe via natural laws, then god is detectable as the normal occurence of natural laws - and so what is the difference between a god who behaves exactly as natural laws describe the universe, and no god at all?
Printer Friendly | Permalink |  | Top
 
Ando Donating Member (112 posts) Send PM | Profile | Ignore Wed Dec-15-04 11:27 AM
Response to Reply #67
70. stalemate
Yes, the best we can reach is a stalemate unless one of us is willing to abandon our worldview. That's what this is about, the complete perspective from which we observe the universe. I'm not hiding my God behind "supernatural" or "spiritual", I simply view "supernatural" and "spiritual" on the same plane as "rational". You see it as hiding because you refuse to recognize supernatural or spiritual. I cannot declare victory under your rules, I have to first convince you that my rules are the right rules. That's why these logical arguments never work, we're not even playing in the same stadium! Hence my suggestion that we talk about the stadiums and not the mechanics of the game.
Printer Friendly | Permalink |  | Top
 
trotsky Donating Member (1000+ posts) Send PM | Profile | Ignore Wed Dec-15-04 12:27 PM
Response to Reply #70
74. I'm not asking you to abandon your worldview,
I'm only asking if there is some way your claims can be tested. If your worldview says "no testing allowed," then it's ultimately non-falsifiable and therefore utterly useless. It says nothing. I might as well postulate the invisible pink unicorn created the universe, and declare that untestable but equally valid to all other worldviews.
Printer Friendly | Permalink |  | Top
 
Ando Donating Member (112 posts) Send PM | Profile | Ignore Wed Dec-15-04 02:09 PM
Response to Reply #74
77. pink unicorns
Yes, you can suggest your pink unicorn. Then write a book telling me how the pink unicorn created the world and saved humanity. Build a church to your pink unicorn and convert the world. Come back to me in another 2,000 years and I'll let you know if your pink unicorn is as valid as the concept of Christianity. You will of course call this an argument ad populum, I could care less. The ideas worth keeping stand the test of time and weather the storms with ease. Why does Christianity persevere? It is because Christianity presents to many the epitome of idealism. The rationalist forms his world on the basis of what he can test and see, the Christian builds his world on what he feels and dreams. The rationalist sees the church of the pink unicorn as equally absurd as the church of God. I think history shows us that not all ideas of theism are equally absurd. I refuse to sacrifice the miraculous and the beautiful at the altar of the intellect.

The historic case against miracles is also rather simple. It consists of calling miracles impossible, then saying that no one but a fool believes impossibilities: then declaring that there is no wise evidence on behalf of the miraculous. The whole trick is done by means of leaning alternately on the philosophical and historical objection. If we say miracles are theoretically possible, they say, "Yes, but there is no evidence for them." When we take all the records of the human race and say, "Here is your evidence," they say, "But these people were superstitious, they believed in impossible things."

Even what we call our material desires are spiritual, because they are human. Science can analyse a pork-chop, and say how much of it is phosphorus and how much is protein; but science cannot analyse any man's wish for a pork-chop, and say how much of it is hunger, how much custom, how much nervous fancy, how much a haunting love of the beautiful. - G.K. Cheaterton

My faith is not untestable, I have tested it and found it to be firm, comforting, and powerful. I have friends who have tested my faith and found it to be truth as well. You have tested it and found it wanting. I believe we are just testing with different aspects of ourselves. I have tested rationalism and secular humanism and found them both wanting. My contention is that logic is not the only test of validity.
Printer Friendly | Permalink |  | Top
 
trotsky Donating Member (1000+ posts) Send PM | Profile | Ignore Wed Dec-15-04 04:26 PM
Response to Reply #77
79. Ooh I would love to take you up on that challenge.
But you know as well as I do that's impossible, which is of course why you offer it up as a defense.

So why does Judaism persevere? Islam? Buddhism? Or the oldest "religion" - ATHEISM?

And why do we regard some ideas of theism as absurd - wouldn't that merely be a function of how widely they were accepted? Look at the claims of Xianity: A virgin birth. Three gods who are actually one. Consuming - not just symbolically, but via the eucharist, ACTUALLY consuming the body and blood of their god. Are those aspects not absurd to a neutral observer? Absurdity truly is in the eye of the beholder.

Science of course does not pretend to answer "What is beautiful?", because that concept only has meaning within the reference frame of one individual's mind. Just as "god" does. However, where things break down is when theists insist their idea of god extends outside their own mental constructs, and into the real world. Just as I can never experience your love of a pork chop, I can never experience what you call god - because both exist solely in your mind.
Printer Friendly | Permalink |  | Top
 
Ando Donating Member (112 posts) Send PM | Profile | Ignore Wed Dec-15-04 04:54 PM
Response to Reply #79
80. I think it's time to stop once again
I think this little tangent has broken down as well. The end result is that you view these "constructs" of the individual mind as non-binding in the materialistic sense. How can a concept exist "outside" of an individual's mind? Isn't your logical assessment of the world around you filtered by your brain? Don't all of your ideas construct themselves in your head or is that just a property us Theists have? You have a group of people (rationalists, atheists) who uphold your worldview. I have a group of people (Christians) who uphold my worldview. My idea of God does extend outside of my own head, I attend a church full of people who hold the same beliefs, have the same experiences, and view the world much like I do. We "see" the effects of Christianity in the "real" world in the same way you "see" the effects of your materialistic philosophy in the "real" world.

I offered up my challenge because it shows the fallacy of the pink unicorn line of thinking. Judaism, Buddhism, Islam, and Atheism all survive because they hold "truth" for their respective followers. Look at David Koresh and Jonestown, along with a host of other cults, they all worked themselves out in the end. the longevity of certain belief systems elevates them to a status above that of pink unicorns and poisoned kool-aid. They resonate with an aspect of humanity and shouldn't be ignored. That a large portion of the human population does not view each of these "faiths" as absurd is meaningful in reality if not in logic.

My conclusion is this: We fundamentally disagree as to the nature of the world around us. I believe in a spiritual realm, miracles, and God. You do not. We can both just laugh as we view each other as equally absurd. That's the fun of philosophy isn't it?
Printer Friendly | Permalink |  | Top
 
trotsky Donating Member (1000+ posts) Send PM | Profile | Ignore Wed Dec-15-04 05:12 PM
Response to Reply #80
81. And then there's Scientology.
A religion that was totally fabricated by a science fiction writer, and yet enjoys a large and devoted following to this day. There's the invisible pink unicorn staring you right in the face.

Considering the older faiths got to arise during a time of primitive superstition and ignorance, it should be no surprise that they were successful in getting ingrained. I consider your "longevity" argument a variation on the argumentum ad populum. If you really wanted to make that comparison, you would have to take a uniform population with no beliefs, then introduce one unique religion to each and every person, and see which wins out. But then of course we have to consider, the more "correct" religion doesn't always win, does it? Maybe the roughest toughest guy in that population would simply threaten people to agree with him or beat them up.

Regarding the "it's all good" conclusion you try to reach: The reason why I think your viewpoint on this fails is that you accept my worldview completely, but then add on the invisible spiritual realm. My worldview is plenty good enough for you... up to a certain point, at which you veer off and say no, I need to add this and this.

The conclusions reached by the worldview you embrace in 99% of your daily life are suddenly declared unacceptable when you get to that 1% where you need to believe in something else.

If that's philosophy, let me quote H.L. Mencken:

The thing that makes philosophers respected is not actually their profundity, but simply their obscurity. They translate vague and dubious ideas into high-sounding words, and their dupes assume, as they assume themselves, that the resulting obfuscation is a contribution to knowledge.
Printer Friendly | Permalink |  | Top
 
Stunster Donating Member (984 posts) Send PM | Profile | Ignore Wed Dec-15-04 11:20 AM
Response to Reply #65
68. Reason and extraordinariness
If God could be found by pure reason, He would cease to be extraordinary.

Time dilation, and its consequences, such as the Twin Paradox, was found by reason. Einstein's. I still regard time dilation and its consequences as utterly extraordinary.

But maybe that's just me.
Printer Friendly | Permalink |  | Top
 
Ando Donating Member (112 posts) Send PM | Profile | Ignore Wed Dec-15-04 11:37 AM
Response to Reply #68
71. good points
I understand your argument, my point was that I don't think God can be completely defined by reason. Maybe I can put it this way: I did not believe in God because I found his wonder reasonable. It's Because I believe in God that I find his wonder reasonable. I hope that makes sense. And yes, Time Dilation is a very extraordinary thing. Have you read "The Great Divorce" by C.S. Lewis? It is the best explanation of Time Dilation in relation to theology, when you get to the last chapter it just blows your mind.
Printer Friendly | Permalink |  | Top
 
trotsky Donating Member (1000+ posts) Send PM | Profile | Ignore Wed Dec-15-04 12:29 PM
Response to Reply #68
75. Extraordinary, but experimentally verified.
Which is all I ask.
Printer Friendly | Permalink |  | Top
 
Stunster Donating Member (984 posts) Send PM | Profile | Ignore Wed Dec-15-04 05:51 PM
Response to Reply #75
82. Here's what I'll ask in that case
Provide experimental confirmation of this proposition:

"The only valid way of knowing anything about reality, other than truths of logic or mathematics, is by means of the experimental method."
Printer Friendly | Permalink |  | Top
 
trotsky Donating Member (1000+ posts) Send PM | Profile | Ignore Wed Dec-15-04 08:07 PM
Response to Reply #82
84. The experimental method, a.k.a. the scientific method:
Boiled down, it amounts to

1) Observe something
2) Make a hypothesis
3) Test the hypothesis
4) If test does not confirm hypothesis, reformulate it or refine your test.
5) Repeat if necessary.

The beauty of it is, we do this ALL THE TIME without even knowing it. Humans were doing it long, long before it was even called the scientific method. It's simply how the cognitive process works.

1) Ug the caveman sees deer flee the watering hole, and then a tiger arrives.
2) Ug thinks, if the deer run away, maybe that means a tiger is about to come.
3) The next day, Ug sets a trap for the tiger and waits by the watering hole. When the deer run away, he springs the trap.
4) Hypothesis successful, dinner time!

In fact, you even do it with regards to your faith. You "observe" a spiritual phenomenon, you try and fit it into your spiritual system, and although your "test" of said phenomenon does not meet the strict definition of an empirical test, you nonetheless compare your hypothesis with the spiritual "reality," and adjust the hypothesis as needed. It's how you arrived at your personal version of Christianity.

So if you use the experimental method even within your worldview, why do I need to justify it for you?
Printer Friendly | Permalink |  | Top
 
Stunster Donating Member (984 posts) Send PM | Profile | Ignore Wed Dec-15-04 09:43 PM
Response to Reply #84
89. The question was...
Edited on Wed Dec-15-04 09:44 PM by Stunster
What experiment confirms that the experimental method is the only valid form of knowledge about the world?

How are these propositions empirically verified?

"There are minds other than my own".

"Sensory data normally indicate the existence of material bodies outside of one's own, but not always."

"The world did not spring into existence five minutes ago"

It seems to me that all of these could be false, consistent with our empirical observations, no matter how often repeated. And yet, I'd say that, in a very reasonable sense of 'know', we know they're all true.
Printer Friendly | Permalink |  | Top
 
trotsky Donating Member (1000+ posts) Send PM | Profile | Ignore Thu Dec-16-04 08:45 AM
Response to Reply #89
94. It's like this.
Only the experimental method results in workable knowledge. I.e., something that is verifiable and therefore useful.

"There are minds other than my own."

I know my reaction to certain stimuli, I know I am a human being. Therefore when I see other human beings, and see them respond similarly to the same stimuli, the most reasonable explanation is that they have minds too. There are far more complicated explanations available via a simple Google search if you are so inclined.

"Sensory data normally indicate the existence of material bodies outside of one's own, but not always."

That is true. Hallucinations are also the product of a material mind, and so that's where the verifiability comes in. If I can repeat it, and others can repeat it, and experiments verify it, it becomes more and more likely to be real. Can we ever be absolutely, positively sure that it's real? I'll readily admit we cannot. But that's the beauty of science as opposed to religious revelation - it's open to new data and can adjust itself accordingly. (The interesting thing is that even hallucinations can be empirically tested - we KNOW there are certain substances, LSD for example, that will cause the appearance of them.)

"The world did not spring into existence five minutes ago"

That might be true, but it's a non-falsifiable claim. If the world was created five minutes ago but made to appear as if it were billions of years old, what's the difference from a practical standpoint? It's an irrelevant proposition. (And one that only makes sense if you presuppose the existence of a deceitful creator.)
Printer Friendly | Permalink |  | Top
 
Stunster Donating Member (984 posts) Send PM | Profile | Ignore Thu Dec-16-04 10:38 AM
Response to Reply #94
99. Utility and falsifiability
You say religion exists because it's comforting for some people.

Well, that's useful. So if usefulness is a criterion of reality or validity, then thanks for providing a new argument for theism.

Of coure, if theism is true, then theistic belief is useful for getting at the truth, and maybe even achieving ultimate happiness. Just declaring---as if it were self-evident--that it's not useful thus begs the question (again) *sigh*

If what you're after is falsifiability, what I've been pointing out is that there are many important propositions which we generally regard as true, and as known to be true in some reasonable sense of 'know', which are clearly non-falsifiable.

A famous example discussed by Wittgenstein is, "This is my hand"

A famous example discussed by G. E. Moore is, "I've never been to the Moon".

Another one from Moore is, "The world did not spr.." Oh wait, I gave that one already. Ok, another Moore one is, "There is an external world".

So that suggests that demanding that propositions be empirically falsifiable is not going to work as a sole criterion of knowledge. And of course, as I've pointed before, there are many things which are not physical entities that can be investigated empirically.

Numbers
Thoughts
Subjective quality of emotions
Subjective perceptions of color
Meanings
Reasons
Free will
Moral obligations
Logic

Hence propositions about these things will often not be empirically verifiable. But these are all pretty central to human living.


:eyes: :eyes: :eyes:
Printer Friendly | Permalink |  | Top
 
trotsky Donating Member (1000+ posts) Send PM | Profile | Ignore Thu Dec-16-04 11:12 AM
Response to Reply #99
106. Well, if you want to extend "comfort" to "utility"
Edited on Thu Dec-16-04 11:25 AM by trotsky
then you'll have to give validation to EVERYTHING that brings people comfort. Astrology, astral projection, faith healing, and so on. That's a pretty poor criterion on which to judge truth.

The same kind of reasoning extends to your conjecture, which is namely that if there are *some* things that can't be conclusively empirically demonstrated, then we must allow for your specific belief that cannot conclusively empirically be demonstrated.

On edit: I'd like to add that something exhibiting a sense of comfort can basically only be established as a mental creation in a physical mind. I.e., "comfort" exists only in the mind of the comforted. Again, quite different than things like "god" which are purported to exist outside the mind.
Printer Friendly | Permalink |  | Top
 
Stunster Donating Member (984 posts) Send PM | Profile | Ignore Thu Dec-16-04 11:30 AM
Response to Reply #106
107. Grrrrrr
You were the one who introduced the pragmatic criterion, not me. I was just showing you that even if you did, that still wouldn't rule out the possibility of theism.

Ugh!

Basta!

I got to go to work!
Printer Friendly | Permalink |  | Top
 
trotsky Donating Member (1000+ posts) Send PM | Profile | Ignore Thu Dec-16-04 11:40 AM
Response to Reply #107
108. And it was you that extended the umbrella of "pragmatic"
to cover the concept of "comfort."

Just pointing out how that fails. No need to name-call.
Printer Friendly | Permalink |  | Top
 
Stunster Donating Member (984 posts) Send PM | Profile | Ignore Fri Dec-17-04 06:27 PM
Response to Reply #108
117. More on verificationism
In the mid-20th century, inspired by the popularization of the philosophical ideas of the 'Vienna Circle' by leading English atheist philosopher, A. J. Ayer, in his book LANGUAGE, TRUTH AND LOGIC, a number of atheist-minded intellectuals proposed as a criterion of meaningfulness in a proposition that it must be either analytic (i.e. true or false in virtue of its meaning), or else empirically verifiable.

"A statement is meaningful only insofar as it is true or false by definition, or else is empirically verifiable. All statements failing to meet this criterion are meaningless."

Alas and alack, the aforementioned statement of the logical positivist criterion of meaning failed itself to satisfy the criterion.

But when I first learned of it, I remember proposing a different objection, and that was this:

If Christianity is true, and there is an afterlife in which the existence of God is confirmed by experience, then there is a way to allow the criterion to stand and still hold Christian belief to be meaningful. But if atheism is true, and there is no afterlife, then we won't be around to know that it's true. So there's a sense in which atheism is not meaningful, by the criterion on offer---there's no way to confirm it by experience. Or rather, atheism is only meaningful if it's false---that is, if it is confirmed in an afterlife experience that God does exist.

Now I suppose one can come up with other forms of afterlife or personal immortality in which the question of God's existence remains open, or even is somehow disconfirmed---though it's hard to imagine how the latter possibility might be realized. Maybe in Buddhistic nirvana states, one attains eternal peace and a state of blissful consciousness in which one is aware that there is no God. Then again, one might not. It's hard to say for sure.

But if there's no afterlife, it's *certain* that there can be no verification that atheism is true. Which suggests that insofar as we're inclined to accept the logical positivist criterion of meaning, professions of atheism are not meaningful if 'atheism' includes the denial of an afterlife. Which leads to the odd result that if atheism is true, it's not verifiable. Which leads to the even odder result that if atheism is true, and the logical positivist criterion of meaning is correct, then atheism is meaningless.

Of course, the easy way out of this logical quagmire is to deny the logical positivist criterion of meaning, which is what most sensible people ended up doing.
Printer Friendly | Permalink |  | Top
 
Stunster Donating Member (984 posts) Send PM | Profile | Ignore Fri Dec-17-04 01:54 PM
Response to Reply #84
113. Materialism is an unproven faith, and probably false
Most of philosophy is really about whether materialism can possibly be true.

Trying to give adequate, convincing materialist accounts of

1) mathematics

2) logic

3) rationality in general

4) the nature of thought

5) the phenomenal properties of perception

6) the phenomenal properties of sensation

7) knowledge

8) meaning

9) free will

10) morality

11) aesthetic value

12) consciousness in general

13) the laws of nature

and even of

14) material objects themselves

has proved an arduous task, and there is no consensus that it has been, even remotely, successfully accomplished or even has any prospect of success.

Notice how important the items mentioned on this list are to daily human living. The average person usually doesn't think of any of them, except 14, in materialist terms. And ever since Berkeley, there have been serious doubts levelled about materialist understandings of material objects---giving rise to various forms of phenomenalism (even among non-theists such as Russell and Ayer, as well as the in whole phenomenological tradition of Continental philosophy). So when you say that nearly all of us nearly all the time think and act as though materialism is true, I say, "WTF?". Materialism didn't even become a widely held belief until the 19th century. And it has taken a few hard blows in the 20th century, not least from science itself (an enjoyable account is THE MATTER MYTH by Paul Davies and John Gribbin, the first chapter of which is entitled 'The Death of Materialism').

On some interpretations of quantum mechanics, consciousness is invoked to 'collapse the wave-function' and thus generate the stable world of material objects that we seem to see. How ironic that materialism's fundamental science---physics---might need to be supplemented by an irreducible component of consciousness.

I point all this out not to declare adherence to any particular philosophical theory or any particular interpretation of quantum physics, but merely to show how controverted all these issues are. Yet most of your arguments presuppose that materialism is true, and even suggest that it's 'obviously' true. To which my response is,:eyes:
Printer Friendly | Permalink |  | Top
 
trotsky Donating Member (1000+ posts) Send PM | Profile | Ignore Fri Dec-17-04 03:02 PM
Response to Reply #113
114. I'll just have to let your eyes roll and be done.
You have proven to have an endless army of strawmen, and I tire of dispatching them.

All of your arguments presuppose that theism is true, but even worse than materialism, your arguments don't even explain anything. Everything leads to "god did it", which not only doesn't explain, it makes things MORE complicated.

Theism: unjustified and unproven. It is a concept in your own mind, nothing more. And that was kind of the point here all along - you're the one who turned it into an attack on what you think is materialism. You never did address any of my key points, but instead just launched into generic attacks and tired old bogus analogies. And to think you teach this stuff. Yikes.
Printer Friendly | Permalink |  | Top
 
Name removed Donating Member (0 posts) Send PM | Profile | Ignore Fri Dec-17-04 04:16 PM
Response to Reply #114
115. Deleted message
Message removed by moderator. Click here to review the message board rules.
 
Stunster Donating Member (984 posts) Send PM | Profile | Ignore Fri Dec-17-04 05:29 PM
Response to Reply #115
116. Addendum
ON EDIT (but time limit had expired):

Of course, what I expect to happen is that Dawkins & Co. will qualify
and refine their selfish gene theory so that it will accord with your
moral revulsion with respect to setting the kids on fire. Naturally, I find that supremely ironic. Their theory has to be so jiggered about with that it can never be falsified---the very thing that they accuse theists of doing.
Printer Friendly | Permalink |  | Top
 
Stunster Donating Member (984 posts) Send PM | Profile | Ignore Fri Dec-17-04 08:50 PM
Response to Reply #114
118. Do you tell biologists to give up biology
in favor of physics because biological explanations are more complex than physics explanations? Or economists to give up economics because physics provides simpler explanations of the price of oil?

You're assuming that science itself is completely reductionistic, but that itself is far from being established. It's certainly not clear that from a complete knowledge of physics, one could predict the emergence of higher level properties such as life or consciousness or culture. For one thing, there are serious doubts about whether determinism is true. For another, it's possible that higher level properties supervene on physical properties in virtue of laws that are not themselves laws of physics. And that's just for starters.

So harking on with this complaint that theism posits something of a higher order of complexity is like complaining that mathematicians posit sets to explain numbers.

Nor do I need to accept that God is more complex than the world. In classical theism, God is a simple, not a composite being. God is immaterial substance, i.e. is not composed of parts. Matter, by contrast, always seems to possess a complex essence, with a variety of measurable properties such as position, velocity, momentum, angular momentum, etc. Simplicity is really a vague, not a well-defined, notion, and I suspect that when used to characterize explanations, it's inherently subjective. But when we use it to characterize, not explanations, but beings, then I think that rational consciousness is less obviously complex (in the sense of being composite or divisible into discrete parts or properties) than is material reality. It's certainly not obvious that mind is more ontologically complex than matter.

But even if mind or God was more ontologically or explanatorily complex than material reality, one would still have to show why a correct explanans of any given thing has to be less complex than the explanandum. It seems to me that in science, we often explain something by positing something more complex than the thing being explained. For example, we see an apple fall from a tree. When along comes Einstein with his General Theory of Relativity, do we say, "YOU'RE WRONG, ALBERT. YOUR THEORY IS A LOT MORE COMPLEX AND HARDER TO UNDERSTAND THAN A FALLING APPLE!"????

No, we don't, is the short answer to that. Same with explaining WELCOME TO SCOTLAND signs by reference to conscious rational minds acting purposively.
Printer Friendly | Permalink |  | Top
 
Jim__ Donating Member (1000+ posts) Send PM | Profile | Ignore Wed Dec-15-04 12:17 PM
Response to Reply #58
72. That naturalism proclaims human consciousness is the result
of blind, accidental chance is a misstatement of the usual naturalist position


Taylor discusses this example within the general context of human
rationality: how is human consciousness any different from the
accumulation of accidental markings upon a stone? Perhaps, given
enough time, nature is bound to evolve something as marvelous and
astonishing as human consciousness. But we cannot have it both ways,
says Taylor. We simply cannot consistently claim that human
consciousness is both the chance outcome of blind, accidental causes
and a reliable belief-forming apparatus by which we discern truths
about the world.


The forms of naturalism that I am familiar with hold that human consciousness arises, not out of "blind chance", but out of chance coupled with natural selection; i.e. while the initial change is random, the survivor is selected (statistically) by how well he fits his environment. A, say lifeform, that can detect what is going on in its environment, and react to this in such a way as to improve its chance of survival, has a selective advantage over one that can't. So, ultimately, the lifeforms that survive are those that discern the truth about the world. This does a fair job of explaining why humans, the result of eons of natural selection, can detect certain truths about the world.

Yet, how could we ever come to know (via accidentally-caused
belief-forming faculties) that our belief-forming faculties are the
result of blind, accidental causes?...


Well, first, as stated above, the premise is wrong.

...In order to achieve this
certainty, it seems human consciousness would require the ability to
"step outside" or otherwise transcend the blind, accidental nature of
the system in order to objectively evaluate that system. But this is
just what naturalism denies: nothing transcends the system.


Actually, since human consciousness had the chance to evolve precisely because the human brain could know certain truths about the world, it is not that surprising that it can know certain truths about itself, a part of the world.

... It is not clear how these could be a
contingent, expansionary function of matter; but yet the naturalist
must somehow maintain that they are. Wouldn't it just be better to
admit that some features of the world do not seem to be subject to
change or the result of blind, accidental causes? ...


Once again the basic premise is wrong.

...Quite frankly, there
doesn't seem to be anything foundational enough within the landscape
of naturalism upon which to establish the unchangeableness of things
like number, or justice. For, according to evolutionary theory,
everything is subject to change, to progress -- except for,
apparently, the doctrine of naturalism itself.


That's a somewhat amazing statement. Our beliefs about the world change all the time. As we learn more and more about nature, we come to a better understanding of the "how" and "why" of change; and the understanding of naturalistic processes are changing all the time. Today, we have an understanding of how selection allows, let's say, life forms, to improve. Tomorrow, we may come to know and understand the steps that brought about human consciousness. You may claim that the basic premise of materialism doesn't change; but, materialism has proven a fertile assumption as a basis for learning about the world. If we reach the point where it is no longer a useful assumption, then we will have reason to re-examine it.

As a matter of fact, I think one of the strong arguments for the naturalist approach is that it allows us to learn, and to change our ideas based on learning. What I find so disturbing about a position that all difficult problems can be resolved by postulating some "other" phantasmagorical realm and claim that all difficulties are magically resolved there, is that it condemns us to a a state of perpetual ignorance; or, to a self-proclaimed, but untested (and untestable) omniscience.
Printer Friendly | Permalink |  | Top
 
Stunster Donating Member (984 posts) Send PM | Profile | Ignore Wed Dec-15-04 01:36 PM
Response to Reply #72
76. Adaptiveness and reliably tracking the truth
Edited on Wed Dec-15-04 01:38 PM by Stunster
are not the same. The connection between them is logically contingent, not logically necessary.

Natural selection works by selecting features and traits which are adaptive to the environment. But there are an infinite number of ways a thinking organism could be well adapted to the environment and still have systematically false beliefs about the environment. Furthermore, the environment itself changes as a result of... what? Not natural selection. So even if it were somehow the case that natural selection managed to get a thinking organism to have true beliefs, these beliefs could come to be false over time. And unless they became false in a way that was connected to survival, and led to extinction, the organism could still be well-adapted while it was the case that many of its cognitive states would systematically fail to track the truth about the world.

In fact, there are reasons for thinking that belief in evolution is actually inconsistent with atheism. Some more thoughts on this:

EVOLUTION AND ATHEISM

If evolution is true, then what is the probability that the cognitive
faculties of human beings produced by evolution will be reliable?

Well, it depends. If evolution is guided and directed by God, then the probability that human cognitive faculties will be reliable is high. If, however, God does not exist and there is no intelligent creator or designer of evolution, but rather evolution is governed by the blind chance governing genetic mutation and changing environmental circumstances, then there is no good reason to think that evolution will produce reliable cognitive powers.

If, as atheistic evolutionists believe, the only thing driving evolution is a competitive process of genetic reproduction and expansion, then what will be produced is adaptive behavior. But is there any necessary connection between *adaptive behavior* and *true beliefs*?

Behavior may or may not be caused by belief. But it is often caused by desire, and often (we think) by some combination of belief and desire. For any given adaptive action, there will be many belief-desire combinations that could produce that action; and very many of those belief-desire combinations will be such that the belief involved is false.

Take a creature called Paul, for example:


Suppose Paul is a prehistoric hominid; a hungry tiger
approaches. Fleeing is perhaps the most appropriate behavior: I pointed out that this behavior could be produced by a large number of different belief-desire pairs. To quote myself:

Perhaps Paul very much likes the idea of being eaten, but when he sees a tiger, always runs off looking for a better prospect, because he thinks it unlikely that the tiger he sees will eat him. This will get his body parts in the right place so far as survival is concerned, without involving much by way of true belief. . . . . Or perhaps he thinks the tiger is a large, friendly, cuddly pussycat and wants to pet it; but he also believes that the best way to pet it is to run away from it. . . . or perhaps he thinks the tiger is a regularly recurring illusion, and, hoping to keep his weight down, has formed the resolution to run a mile at top speed whenever presented with such an illusion; or perhaps he thinks he is about to take part in a 1600 meter race, wants to win, and believes the appearance of the tiger is the starting signal; or perhaps . . . . Clearly there are any number of belief-cum-desire systems that equally fit a given bit of behavior (WPF pp. 225-226).

Accordingly, there are many belief-desire combinations that will lead to the adaptive action; in many of these combinations, the beliefs are false.

Without further knowledge of these creatures, therefore, we non-divinely directed evolution will produce reliable cognitive faculties.]
A problem with the argument as thus presented is this. It is easy to see, for just one of Paul's actions, that there are many different belief-desire combinations that yield it; it is less easy to see how it could be that most of all of his beliefs could be false but nonetheless adaptive or fitness enhancing. Could Paul's beliefs really be mainly false, but still lead to adaptive action?

Yes indeed; perhaps the simplest way to see how is by thinking of systematic ways in which his beliefs could be false but still adaptive. Perhaps Paul is a sort of early Leibnizian and thinks everything is conscious (and suppose that is false); furthermore, his ways of referring to things all involve definite descriptions that entail consciousness, so that all of his beliefs are of the form That so-and-so conscious being is such-and-such. Perhaps he is an animist and thinks everything is alive. Perhaps he thinks all the plants and animals in his vicinity are witches, and his ways of referring to them all involve definite descriptions entailing witchhood. But this would be entirely compatible with his belief's being adaptive; so it is clear, I think, that there would be many ways in which Paul's beliefs could be for the most part false, but adaptive nonetheless.

Now given that, in a process which involves chance and accident (such as atheists believe evolution to be), there will be very many ways in which adaptive behavior could be divorced from the possession of reliable cognitive faculties (i.e. faculties which arrive at the truth with a high degree of success), then the chances of such a process producing *reliable* faculties is low.

Alternatively, we simply WOULDN'T KNOW how likely it is that such a process would generate reliable faculties. But in either case, we would have NO GOOD REASON to believe that our cognitive faculties are
reliable (i.e. reliably lead us to true beliefs rather than false ones).

So if evolution is the result of chance, and not designed by God, then we have no good reason to believe in evolution, because in the case in which it is not governed by God or an intelligent designer, we have no good reason to believe that our cognitive faculties are reliable either in general, or when, specifically, they judge that evolution is true. If they are the product of blind chance genetic mutations interacting with an unpredictably changing environment (a meteor might strike at any moment, etc), it is, for all we know, quite possible, and maybe even likely, that our cognitive faculties are very, perhaps even systematically, *unreliable* as guides to the truth about the world, including the truth about how we came to be.

Therefore to believe that both atheism and evolution are true is incoherent, because we would have no reason to believe that our beliefs regarding atheism or evolution (or indeed anything else) are formed by a reliable cognitive process, and hence it would not be rationally warranted to believe in evolution or atheism (or much of anything, in fact).

This is really a modern version of an argument that goes back to
Descartes. Plantinga develops the argument in his book WARRANT AND PROPER FUNCTION, and in a number of related articles. Basically, he argues that for a belief to be rationally *warranted*, we must have good grounds for believing that our belief-formation system is *functioning properly*, and that if it is functioning properly, it will generally lead us to have *true* rather than false beliefs. How could a properly functioning belief-formation system generally lead us to truth unless it was designed to do so? Could evolution, driven forward by a series of sheer chance events, and without intelligent guidance, cause the emergence of cognitive equipment which, when functioning properly, reliably and systematically led to truth? Given that there are many ways that adaptive behavior could have evolved *without* us having reliable cognitive faculties, the belief that evolutionary processes alone could have generated such faculties is either improbably true or its probability is unknowable.

In the original Cartesian version of this argument, Descartes imagines that our thought processes are controlled by an evil but superior intelligence, such that we are in a constant state of illusion. Now, for the purposes of the argument, it doesn't have to be an evil demon or any other being endowed with intelligence. It could just as well be any *natural process* which renders our cognitive faculties, even when functioning properly, systematically
unreliable.

But if thought is to be governed by reasons, and if it in principle 'homes in on' reality when it is so governed, then reality itself
*has* to be intelligible to reason. Nothing could truly be a reason if reality itself is not rationally structured. And, (what this Cartesian argument is saying), the very concept of God is implied in supposing reality to be rationally structured and hence amenable to the reliability for discovering truth of rational thought, for God is that which endows reality with rational intelligibility and guarantees a connection between rational thought and reality. Without God, there can be no guarantee that thought will 'work', so to speak. It might be systematically set up to fail either by natural processes (affecting our brains, say), or (as in the original Cartesian version) by supernatural entities which are either maliciously deceive us, or
which make errors in their creation or handling of our minds. In both cases, it would be *a contingent matter* not merely whether our cognitive faculties would work on any given occasion, but whether our cognitive faculties were generally suited to successfully tracking the truth about reality. (The same goes for the case of our mental processes being affected by intelligent aliens.)

But all our thought presupposes that if it is carried out correctly by properly functioning cognitive faculties, then it is *guaranteed* to
be something that in principle can grasp reality correctly. In short, our presupposition is that the connection between correct thinking (thinking, that is, which functions in the way it is set up to function) and reality has to be necessary, it can't be contingent. (Our customary presupposition--outside of discussions of epistemology--is that 'correct thinking' just *means* 'will tend to converge on reality or truth').

But if the connection is a necessary one (and we in fact presuppose that it is), then there has to be a being which is necessarily non-malicious, necessarily infallible, and which exercizes beneficent control over natural processes and entities, and over any super- or non-natural entities. And this is in fact our concept of God.
So, if anything truly counts as 'a reason for belief' and as 'rational belief', then the concept of God is implicitly presupposed. But the notion that there are reasons for beliefs, that there rational beliefs, is itself one of our basic beliefs. And if we have that basic belief in the possibility of rational thought and in it having a necessary connection in principle to reality, then implicitly at least, we hold a basic belief in God's existence. And if we don't accept that, then we are in fact being irrational, since we necessarily presuppose the rational intelligibility of reality in all our thinking, when in fact it might not rationally intelligible and/or our cognitive processes might systematically diverge from the truth (if there is no God). It's easy to imagine, in fact, how systematic divergence from the truth might actually make us more adaptive to the environment. For example, fearlessness might be an adaptive trait caused by a belief that wild animals won't really hurt us if we try to capture them.

In either case (i.e., there being an evil demon or there being blind chance directing the development of our cognitive equipment) we would lack a warrant for believing that our cognitive faculties are, when properly functioning, systematically reliable. And hence we would lack a warrant for believing in evolution.

By contrast, evolution under God's control would be very likely to produce reliable cognitive faculties in beings God wished to create in God's 'own image'. The sense in which we are created in God's image is that we are created with the capacity to know truth, and love the good. Monkeys, on the other hand, don't have the capacity to know (much of) the truth, or love (much of) the good. Yet monkeys have adapted quite well to their environment over aeons. But if evolution is blind, we are not warranted in thinking that we are cognitively any better off than monkeys. If evolution is blind, then for all we know, the cognitive equipment of monkeys might be much more reliable than ours. They may know the ultimate laws of physics, while we're still trying to figure them out. We would not be warranted in thinking that it wasn't if cognitive ability is the product of chance. But if we are not warranted in believing that our cognitive faculties are better than those of monkeys, then we are not warranted in believing that evolution is true either. Nor could we be warranted in 'rationally' preferring atheism over theism.
Printer Friendly | Permalink |  | Top
 
Jim__ Donating Member (1000+ posts) Send PM | Profile | Ignore Wed Dec-15-04 08:31 PM
Response to Reply #76
86. Your assertions are quite beside the point
Your original post claimed:

We simply cannot consistently claim that human
consciousness is both the chance outcome of blind, accidental causes
and a reliable belief-forming apparatus by which we discern truths
about the world.


In this assertion, your premise about consciousness being the outcome of blind chance accidental causes is wrong - it's a strawman argument. But, also any claim that human consciousness formed through natural selection cannot be a reliable belief-forming system is wrong; for in spite of all your assertions about us maybe wanting to be eaten by the tiger, and running away because we believe this will lead to being eaten; it is just as feasible (and I claim far more feasible) that we run away from the tiger because we don't want to be eaten, i.e. random change coupled with natural selection is a feasible way for us to form a consciousness with reasonably accurate perceptions of the world.

To think that we can get just about everything about the world wrong, and still act in such a way as to out-survive all of our competitors is, to put it mildly, far-fetched. I doubt your claims about surviving in such a state would withstand a rigorous examination; but, even if they would, that doesn't invalidate the possibility that natural selection leads to a reliable perception of the world.
Printer Friendly | Permalink |  | Top
 
Stunster Donating Member (984 posts) Send PM | Profile | Ignore Wed Dec-15-04 09:15 PM
Response to Reply #86
87. Huh?
Edited on Wed Dec-15-04 09:23 PM by Stunster
Lots and lots of people have had lots and lots of false beliefs about the world.

They survived, nonetheless.

Evolution involves random chance genetic mutations. Some of them might be---indeed, on the evolutionary materialist worldview, must be---responsible for all these false beliefs.

So, adaptiveness does not guarantee reliable tracking of the truth.

Monkeys' cognitive equipment also evolved. Yet it is far from adequate for knowing lots of truths about the world.

We might be, for all we know, in the same position as monkeys.

That is, we might be, if evolution, conceived on materialist terms, is all there is to the development of our cognitive faculties. For in that case, the reliability of our belief in evolution might itself be undermined. In the simplest case, a mutation determines that all members of the species believe P, where P happens to be false. The problem is, how can we know that P is false if we've been determined by evolution, to believe that P, given certain environmental conditions? Well, ex hypothesi, we can't!

But P might be the proposition that human beings evolved by natural selection and random genetic mutation. Ergo, we can't know, if evolution is naturalistic, whether naturalistic evolution is true.
Printer Friendly | Permalink |  | Top
 
Jim__ Donating Member (1000+ posts) Send PM | Profile | Ignore Thu Dec-16-04 09:49 AM
Response to Reply #87
95. Are you conceding the point then?
From you post Huh?:

Lots and lots of people have had lots and lots of false beliefs about the world.

And, from your post Adaptiveness and reliably tracking the truth:

If evolution is guided and directed by God, then the probability that human cognitive faculties will be reliable is high.

So, since you seem to be able to tell us how god would evolve human cognitive faculties and, by your own words, our cognitive faculties don't meet that criteria, I think the burden falls on you to explain why god chose to give us cognitive faculties that are often wrong - i.e. unreliable.

My claim for purely materialistic evolution is that random change coupled with natural selection is a feasible way for us to form a consciousness with reasonably accurate perceptions of the world.

I maintain that people have reasonably accurate perceptions of the world - reasonably accurate meaning that we correctly perceive those things that are necessary for our survival, for instance, we correctly perceive a tiger as dangerous and correctly decide to try to avoid tigers.

Now, our cognitive faculties have evolved to the point that we form opinions about things that are not necessary to our survival, e.g. metaphysics. And, I think we all have false beliefs about metaphysics; but, generally, it doesn't matter to our survival. In other words, evolution didn't seem to guarantee that our cognitive faculties would be reliable for things that are not pertinent to our survival.

I would appreciate it if you would explain to us a few things about the mind of god; particularly, why he would probably have us evolve cognitive faculties that are reliable (I presume reliable in all things including metaphysics); and then why he didn't.
Printer Friendly | Permalink |  | Top
 
Stunster Donating Member (984 posts) Send PM | Profile | Ignore Thu Dec-16-04 10:49 AM
Response to Reply #95
102. Read some Descartes
Edited on Thu Dec-16-04 11:31 AM by Stunster
This is absolutely central to the whole of epistemology since his time.

Our cognitive faculties, when functioning as they were designed to function, are reliable.

But for a host of reasons, ranging from being dropped on your head as a baby to excessive intake of alcohol to sheer laziness, sometimes they don't function as they were designed to.

The point is that there is, if Descartes and other theists are right, a logically tight connection between cognitive faculties, when functioning properly, and tracking the truth. But that if naturalistic materialism is right, that connection is logically contingent, and we're not in a position to know one way or the other.

Actually, just skip Descartes and go straight to Alvin Plantinga's book WARRANT AND PROPER FUNCTION.
Printer Friendly | Permalink |  | Top
 
Jim__ Donating Member (1000+ posts) Send PM | Profile | Ignore Thu Dec-16-04 09:20 PM
Response to Reply #102
109. I have
But for a host of reasons, ranging from being dropped on your head as a baby to excessive intake of alcohol to sheer laziness, sometimes they don't function as they were designed to.

Ah, I see. Up until about 100 years ago, almost every one, including Kant, and presumably Descartes, had been dropped on their head as a baby. After all, up until about 100 years ago, every one held the "true belief" that we lived in Euclidean space. Now, almost everyone holds the "true belief" that we live in Einsteinian spacetime. And, if humanity survives for another 1000 years, people of that time can probably look back on us as just a bunch drunks.

But, it's good to know that our cognitive faculties are reliable, and our "true beliefs" can be relied upon as true. At least for today.
Printer Friendly | Permalink |  | Top
 
Stunster Donating Member (984 posts) Send PM | Profile | Ignore Fri Dec-17-04 10:23 AM
Response to Reply #109
111. I said 'a host of reasons'
You do the math.
Printer Friendly | Permalink |  | Top
 
Stunster Donating Member (984 posts) Send PM | Profile | Ignore Fri Dec-17-04 11:47 AM
Response to Reply #109
112. And btw,
are you actually saying that even when our cognitive faculties don't malfunction (for whatever reason), they're still unreliable?

?
?
?
Printer Friendly | Permalink |  | Top
 
Jim__ Donating Member (1000+ posts) Send PM | Profile | Ignore Sat Dec-18-04 07:55 AM
Response to Reply #112
120. From an evolutionary point of view, our faculties are quite reliable
From the point of view of leading us to "true beliefs," I consider their reliability to be in serious doubt.

are you actually saying that even when our cognitive faculties don't malfunction (for whatever reason), they're still unreliable?

It depends on what you mean by reliable. From an evolutionary point of view, our cognitive faculties are quite reliable. They tell us what we need to know in order to survive. But, in your post Adaptiveness and reliably tracking the truth, you claim that reliable cognitive faculties arrive at the truth with a high degree of success. Based on that definition, our cognitive faculties do not appear to be very reliable. From the time of Euclid until relatively recently, most Westerners believed that we lived in Euclidean space. Now, we believe that is incorrect. I'm not sure what you consider a "high degree of success"; but, certainly a large number of Westerners, some of them quite serious thinkers, held incorrect (i.e. "untrue") beliefs about some of the most fundamental aspects of our existence - time and space. And, of course, there are a myriad of other examples in which we now believe people had it wrong throughout most of history (e.g. the nature of solid objects).

Now, if humanity should manage to survive for another 1,000 or 10,000 years, I'd be willing to bet that the people of that period will consider us to have been wrong in most of our beliefs about the true nature of reality. So, based on your criteria that reliable cognitive faculties arrive at the truth with a high degree of success; our cognitive faculties are not very reliable.
Printer Friendly | Permalink |  | Top
 
Stunster Donating Member (984 posts) Send PM | Profile | Ignore Sat Dec-18-04 12:14 PM
Response to Reply #120
122. The horns of the dilemma for the evolutionary materialist,
given what you seem now to be admitting---namely, that there is only a contingent relation between adaptiveness and truth-tracking---are these:

---the more you regard our cognitive faculties to be reliable for truth-tracking when functioning as they're supposed to, the more it looks like they were designed to be by an intelligent designer.

--the less you regard our cognitive faculties to be reliable for truth-tracking, the less warrant you have for relying on them with respect to a belief in evolutionary materialism itself.

And certainly that belief is far removed from any connection to adaptiveness.
Printer Friendly | Permalink |  | Top
 
Jim__ Donating Member (1000+ posts) Send PM | Profile | Ignore Sat Dec-18-04 06:26 PM
Response to Reply #122
123. There is no dilemma
Just about every thinking person I know accepts that human knowledge is uncertain. History proves it.

---the more you regard our cognitive faculties to be reliable for truth-tracking when functioning as they're supposed to, the more it looks like they were designed to be by an intelligent designer.

Nonsense. Clearly, a system that is adapted to perform well in an environment, is most likely to have adapted to perceive that environment accurately, accurate to the point necessary for survival. Your speculation that we could have it all exactly backwards and still be optimized for survival is absurd. A self-correcting process, like evolution, seems highly likely to be reliable. The process obviates the need for a designer.

The real risk for human knowledge is not that it perceives its environment inaccurately; it's that it perceives its environment incompletely. There are most likely many characteristics of our environment that are not particularly critical to our short-term survival. It is these aspects of our environment for which we have not developed reliable faculties. The task in acquiring knowledge is to use our well-adapted faculties to test our hypotheses about those things we do not naturally perceive well.

--the less you regard our cognitive faculties to be reliable for truth-tracking, the less warrant you have for relying on them with respect to a belief in evolutionary materialism itself.

I believe our perceptual and analytical faculties, both necessary for survival, are reliable. It's our speculative faculties whose reliability I doubt. That is why I believe our speculations must be tested. Testing can bring our speculations in line with reality. Over time, untested speculations built on top of other untested speculations are likely to wander far from any relation to reality.
Printer Friendly | Permalink |  | Top
 
Stunster Donating Member (984 posts) Send PM | Profile | Ignore Sat Dec-18-04 08:03 PM
Response to Reply #123
124. Vast numbers of species evolve and survive
Edited on Sat Dec-18-04 08:09 PM by Stunster
with extremely limited cognitive equipment, relative to our own (or so we are inclined to think). Some of their cognitive equipment appears to be quite different from ours. What's it like to be, say, a bat, or a cat, or a kangaroo, or a shark?

For some species, it is questionable to what extent we can even attribute beliefs to them.

What is important for survival is not beliefs, then, but behavior. As long as our behavior is fitted to our environment sufficiently well so that our behavior will ensure our survival, then belief may go its own way. And given the nature of other species, true beliefs may be unnecessary altogether, or else we may only require a very limited stock of them. There may be a tight logical relation between survival and behavior. But there is no such tightness of logical relation between survival and belief, since many thriving species have no rich set of beliefs.

This is not just a problem with metaphysical beliefs. Other species don't have scientific beliefs either, nor did humans until relatively recently. So survival of the species is logically independent of any science. So, if our belief-forming processes are not reliable beyond what is necessary to ensure survival, then the whole of our scientific belief-forming processes may be unreliable. But that would also affect any scientific belief in evolutionary materialism----unless, that is, our belief-forming processes were intelligently designed to be reliable when functioning as they were designed to.

So the dilemma remains. If evolutionary materialism is true, then we lack proper warrant for it. But if we have proper warrant for our scientific beliefs in general, formed in accordance with the proper functioning of our belief-formation system, then evolutionary materialism is probably not true.
Printer Friendly | Permalink |  | Top
 
Jim__ Donating Member (1000+ posts) Send PM | Profile | Ignore Sun Dec-19-04 11:06 AM
Response to Reply #124
125. Your argument is very powerful, unless you happen to
Edited on Sun Dec-19-04 11:09 AM by Jim__
open your eyes and look around.

Human beings aren't just surviving in the environment they found. Human beings have altered their environment to make their short-term survival easier. Now, are our environment altering skills based on adaptively acquired traits, i.e. traits acquired through purely materialistic evolution. Clearly the answer can be yes.

In complex species, the ability to observe the environment and to react to observations is a clear adaptive advantage. Human beings have a well-developed capacity for observation and reaction. Look at those species closest tp us on the evolutionary tree. Do they exhibit intelligence? Yes. Intelligence is an adaptive advantage.

What is the human selective advantage over its competitors? Clearly it is our superior intelligence. Our ability to remember facts, - which way did the deer run last time -, and to adapt our behavior based on this memory, - where should we hide the spearmen. Memory and analytical skills are precisely our adaptive advantage. What makes trees grow? Can we control that? When is the best time to sow and when to reap? Is there any regularity in the seasons? When will it rain? All scientific questions, all resulting from our adaptive advantage. Indeed, it would be amazing if evolution developed in us analytical skills that could not be applied more generally, i.e. if evolution developed in us analytical skills that made us better hunters, but did not allow us to track the seasons. And, having learned to sow and reap, to predict seasonal changes and some ability to anticipate daily weather changes, it would be surprising if we didn't ask more difficult questions - can we get to the moon?

Now, are our answers to these questions likely to be correct. Well, at first attempt, probably not. The first time we climb a tall mountain with the hope of jumping to the moon, we'll fail. But can we learn from our mistakes? Yes. The same way we learned where the deer would most likely run, and where we should spring our trap. All traits developed through material evolution that have a more general application than mere day-to-day survival.

Now if you tell me that our untested beliefs are not likely to be true, e.g. our metaphysical beliefs, then I agree. Our metaphysics is likely wrong. But the solution to this is not to throw up our hands and either abandon the search for metaphysical truth; nor is it to just pick a set of metaphysical beliefs and declare them to be correct. The solution is to bring our metaphysical claims within the realm of testable claims and then test. This is precisely what a materialistic approach allows us to do.
Printer Friendly | Permalink |  | Top
 
Stunster Donating Member (984 posts) Send PM | Profile | Ignore Sun Dec-19-04 12:24 PM
Response to Reply #125
126. The argument is powerful even looking around us
Now, are our environment altering skills based on adaptively acquired traits, i.e. traits acquired through purely materialistic evolution. Clearly the answer can be yes.

Clearly, the answer cannot be 'yes, we know that we have good warrant for believing that this is the case', unless you are assuming the truth of the explanandum---that our cognitive equipment, when functioning in accord with its evolutionary history, is generally reliable.

Perhaps, for all we know on the basis of what is warranted by materialist assumptions, we would be tremendously more adept than we are at enhancing our chances of short-term survival, if our cognitive equipment had developed differently, and maybe we are in fact struggling to make it as a species because of our rather maladapted cognitive equipment. Just looking around and observing the human world doesn't suggest to me, at any rate, any obviousness in the claim that we are excellent at enhancing our short-term or long-term survival.

Let's assume that monkeys are a good deal less intelligent than we are. On the materialist viewpoint, this is a contingent fact. But in that case, it's may also be a contingent fact that we are far less intelligent than we might have been, far less adapted to our environment than we might have been. Relative to some evolutionary possibility, we may be as unintelligent as monkeys seemingly are by comparison to us. That possible evolutionary path which we might have taken, but did not, may be such that had we taken it, we would be aware that the actual cognitive capacity of humans is relatively very deficient in lots of ways---as, or maybe even much more deficient, as monkey cognition seems to us to be in comparision with human cognition. From materialist assumptions, there's no way to know, since on those assumptions, the level of intelligence we have may be very low compared to some evolutionary possibility. We could be, on the range of potentially evolved intelligence, remarkably dumb.

Certainly the amount of time modern homo sapiens has been around, relative to evolutionary timescales, is not long enough to suggest in any very strong way that we aren't. We may destroy our own species within a century or two, which from an evolutionary perspective is the height of dumbness. That is, we don't know if what we take to be human intelligence is really adaptive or not.

Just as there are some indications that our type of intelligence is an adaptive trait, so there are some indications that our type of intelligence may not generate survival-conducive behavior---even though we might have a richer stock of true beliefs than other currently living species. There is just no tight logical connection between having a rich stock of true beliefs (such as scientific beliefs) and adaptive behavior, given a materialist worldview. Maybe having the type of intelligence or cognitive equipment which generates such a stock of beliefs will turn out to be one of the worst traits a species can have.

The point is, if evolutionary materialism is correct, there's simply no way to know how tight the connection is. In which case, on materialist assumptions, our scientific beliefs may be unwarranted. In which case, our belief in evolutionary materialism itself may be unwarranted. But if it may be, and there's no way to know, then it is.
Printer Friendly | Permalink |  | Top
 
Stunster Donating Member (984 posts) Send PM | Profile | Ignore Sun Dec-19-04 12:38 PM
Response to Reply #125
127. Addendum
http://hisdefense.org/articles/ap001.html">This is not too long, and summarizes Plantinga's argument
Printer Friendly | Permalink |  | Top
 
Stunster Donating Member (984 posts) Send PM | Profile | Ignore Sun Dec-19-04 12:41 PM
Response to Reply #125
128. Further reading
http://hisdefense.org/articles/ap001.html">This is not too long, and summarizes Plantinga's version of this argument.
Printer Friendly | Permalink |  | Top
 
Stunster Donating Member (984 posts) Send PM | Profile | Ignore Sun Dec-19-04 05:24 PM
Response to Reply #128
130. That link is meant to be to this
Printer Friendly | Permalink |  | Top
 
Stunster Donating Member (984 posts) Send PM | Profile | Ignore Sun Dec-19-04 05:27 PM
Response to Reply #130
131. I like this quote from Darwin
"With me," says Darwin, "the horrid doubt always arises whether the convictions of man's mind, which has been developed from the mind of the lower animals, are of any value or at all trustworthy. Would any one trust in the convictions of a monkey's mind, if there are any convictions in such a mind?"
Printer Friendly | Permalink |  | Top
 
Jim__ Donating Member (1000+ posts) Send PM | Profile | Ignore Mon Dec-20-04 11:11 AM
Response to Reply #125
134. Aquinas, god, and the number of angels that can sit on the head of a pin
Perhaps, for all we know on the basis of what is warranted by materialist assumptions, we would be tremendously more adept than we are at enhancing our chances of short-term survival, if our cognitive equipment had developed differently, and maybe we are in fact struggling to make it as a species because of our rather maladapted cognitive equipment. Just looking around and observing the human world doesn't suggest to me, at any rate, any obviousness in the claim that we are excellent at enhancing our short-term or long-term survival.

Well, I look at these speculations you throw around and I look for something to back it up. I mean something like data, facts, information. I see nothing; just some completely unbacked speculation. But, let's take a somewhat closer look.

for all we know on the basis of what is warranted by materialist assumptions, we would be tremendously more adept than we are at enhancing our chances of short-term survival, if our cognitive equipment had developed differently,How much more adept we "might" have been, is quite beside the point.

OK. You claim we could be tremendously more adept at enhancing our short-term survival. Well, the short-term we're interested in is, of course, the time to reproductive age. I don't really have time now to go out and search down the numbers; but, I can find a quick one, mortality rate for children under one in the United States. In 2002, it was 695 out of 100,000. By my computation that's 0.00695%. (Source) Now, the death of any child is tragic and I am not trying to minimize that; but, I'm not sure that we could call any improvement from 99.99305%, a tremendous improvement. The mortality rate drops precipitously after age one. I do believe we both can and will improve on those numbers; but, your speculation that we can "tremendously" improve upon it just doesn't hold water.

Now, my argument is that infant motality in the US is as low as it is because our powers of observation are quite good; as we would expect them to be since they've been sharpened across billions of years of evolution. If god did it, why are infant mortality rates so much higher in non-technological societies?

Let's assume that monkeys are a good deal less intelligent than we are. On the materialist viewpoint, this is a contingent fact. But in that case, it's may also be a contingent fact that we are far less intelligent than we might have been, far less adapted to our environment than we might have been. Relative to some evolutionary possibility, we may be as unintelligent as monkeys seemingly are by comparison to us. That possible evolutionary path which we might have taken, but did not, may be such that had we taken it, we would be aware that the actual cognitive capacity of humans is relatively very deficient in lots of ways---as, or maybe even much more deficient, as monkey cognition seems to us to be in comparision with human cognition. From materialist assumptions, there's no way to know, since on those assumptions, the level of intelligence we have may be very low compared to some evolutionary possibility. We could be, on the range of potentially evolved intelligence, remarkably dumb.

I have no doubt that human cognitive capacity is deficient in lots of ways. I expect that there are aspects of our environment that are not critical to our short term survival; that we either don't perceive at all, or perceive quite poorly; e.g. atoms. My argument is that our powers of observation allow us to deduce their existance and then our analytical abilities allow us to test our deductions. Observation, analysis and test, so far, is working out pretty well for humanity. As to our long-term survival, I think it is seriously compromised by the fact that aspects of our nature were selected for because they contributed to short-term survival, e.g. our war-like nature.

As to the paper you cited; I find it to be the usual creationist pap; for instance, from the cited paper:
"With me," says Darwin, "the horrid doubt always arises whether the convictions of man's mind, which has been developed from the mind of the lower animals, are of any value or at all trustworthy. Would any one trust in the convictions of a monkey's mind, if there are any convictions in such a mind?"
Ah, 2 unsourced sentences from Darwin. I'm convinced. Besides the fact that no matter the context of Darwin's quote, it contains an opinion, not an argument. I've told you before, I find expert opinion totally unconvincing; expert arguments are usually another thing; but, you seem remarkably short of that.

And then, I have a lot of respect for Thomas Aquinas, but he did live over 700 years ago, and his quotes: Since human beings are said to be in the image of God in virtue of their having a nature that includes an intellect, such a nature is most in the image of God in virtue of being most able to imitate God (ST Ia q. 93 a. 4);

And:

Only in rational creatures is there found a likeness of God which counts as an image . . . . As far as a likeness of the divine nature is concerned, rational creatures seem somehow to attain a representation of type in virtue of imitating God not only in this, that he is and lives, but especially in this, that he understands (ST Ia Q.93 a.6).

Please. Some definitions here; you can start with god. Then you can tell us how we know humans are in the image of god; how do we know rational creatures are in the "likeness" of god (and what does that mean), how do we know no non-rational creature is in the likeness of god.

I'm sorry, but if this is the basis of your speculations about, what should we call it, this place where all problems are resolved without any explanantion of how they're resolved? How about MagicLand? And, to propose the speculative, untestable existence of MagicLand as a superior alternative to observable, testable facts and the testable hypotheses based on these facts; defies belief. We might as well spend our time speculating about how many angels can sit on the head of a pin.


Printer Friendly | Permalink |  | Top
 
Stunster Donating Member (984 posts) Send PM | Profile | Ignore Wed Dec-22-04 08:05 AM
Response to Reply #134
148. Uh, no
Edited on Wed Dec-22-04 08:16 AM by Stunster
First of all, you take the infant mortality rate for the US in 2002. Talk about trying to bias your sample! But in any case, one can't measure the adaptiveness of a species by one measure at one time in one region of that species' habitat. That would be like pointing to falling birth rates and saying that means we're becoming less well adapted.

Secondly, this does nothing to prove that the human race couldn't be tremendously more adept at enhancing our short-term survival. I'll skip past the fact that the infant mortality rate per se doesn't tell us anything about our rate of reproduction. But even if you gave the figures for our rate of reproduction, that wouldn't tell us if the rate couldn't be a lot higher than it is. Assuming the 'selfish gene' hypothesis for a moment, we might ask why we haven't figured out a way to support a population of 60 billion, or 600 billion. Maybe if we were smarter, we could quadruple life-expectancy and the average number of child-bearing years. Some aliens might be observing us and commenting, What a lousy species they are---they've only managed to reach a population of 6 billion in 200,000 years, and our calculations show that they are doomed to extinction within the next two centuries. During what will be their terribly brief existence, the average rate of human infant mortality and the average rate of human reproduction have been grossly inferior to the galactic average. What miserable, unpleasant, maladapted beings!

Thirdly, your argument is circular. You are suggesting that our scientific intelligence must be fairly reliable because we're fairly well adapted to the environment. And you're suggesting that we're fairly well adapted to the environment because of the fair reliability of our scientific intelligence. But there must be some independent criterion of scientific intelligence beyond appearing to be fairly well adapted, since lots of other species appear to be even better adapted, yet we don't attribute scientific beliefs to them.

Fourthly, trying to establish that humans are well-adapted to the environment is irrelevant to reliability of our scientific truth-tracking, or else it's question-begging about that reliability (by collapsing reliability into adaptiveness). The argument is not that humans aren't well-adapted, it's that the connection between adaptive behavior and having true beliefs is logically contingent. Other intelligent life-forms in the universe might regard human intelligence as dreadfully inferior and prone to error, and might observe that if we were of even average intelligence, we'd have colonized the solar system 10,000 years ago and would now have a population numbering in the tens or hundreds of billions. They might regard our science as laughably primitive and fumbling, and tell not 'blonde', but 'tiny brain' jokes at our expense:

Hey, the biopartner and I took a vacation on Earth. No don't laugh--wait, on second thoughts, carry on laughing. It was almost as funny as backwards time travel. Anyhow, once we'd undergone multidimensional energy reconfiguration---boy was that a drag--they popped me out as an orangutan at first---well, anyhow, once I looked like a tinybrain, I was trying to explain to a tinybrain how quantum gravity theories were all a big mistake. He was a Nobel Laureate in physics. He was all "But Einstein this, and Schrodinger that..."---and you should have seen the equations he was writing on the board! I'm looking at this stuff and thinking, "You guys shoulda stuck to cave painting." Still, that's evolution for you---some poor schmucks have to draw the short straw.

Bottom line: on evolutionary materialist assumptions, there's no good reason to think that cognitive equipment is particularly reliable. But that undermines the possibility of having a warranted belief in evolutionary materialism.


Printer Friendly | Permalink |  | Top
 
Jim__ Donating Member (1000+ posts) Send PM | Profile | Ignore Wed Dec-22-04 08:25 PM
Response to Reply #148
153. Yes.
First of all, you take the infant mortality rate for the US in 2002. Talk about trying to bias your sample! But in any case, one can't measure the adaptiveness of a species by one measure at one time in one region of that species' habitat. That would be like pointing to falling birth rates and saying that means we're becoming less well adapted.

You ignore the main point, stated quite clearly in my post: Now, my argument is that infant motality in the US is as low as it is because our powers of observation are quite good; as we would expect them to be since they've been sharpened across billions of years of evolution. If god did it, why are infant mortality rates so much higher in non-technological societies?

Technological development clearly correlates with decreased infant mortality. The capacity for technological development was not directly selected for by evolution. It is a by-product of what was selected for by evolution, namely, observation and analysis. And, our technology is reliable. Therefore, it stands as evidence that the evolution of observation and analysis skills are reliable outside the narrow confines for which they were selected.

Secondly, this does nothing to prove that the human race couldn't be tremendously more adept at enhancing our short-term survival.

That's quite beside the point.

Thirdly, your argument is circular. You are suggesting that our scientific intelligence must be fairly reliable because we're fairly well adapted to the environment. And you're suggesting that we're fairly well adapted to the environment because of the fair reliability of our scientific intelligence. But there must be some independent criterion of scientific intelligence beyond appearing to be fairly well adapted, since lots of other species appear to be even better adapted, yet we don't attribute scientific beliefs to them.

See above. Our scientific intelligence is beyond what was selected for by evolution; yet, we are clearly able to do things that were not selected for by evolution; e.g. fly to the moon, apply extraordinary medical care (e.g. decrease the infant mortality rate), take atoms apart, and much, much more. To make the obvious explicit; the fact that our acquired scientific knowledge appears accurate under testing, is a good reason to believe that what we learn about the universe through science is reliable. Certainly, we know of no reliable alternative to science.

Fourthly, trying to establish that humans are well-adapted to the environment is irrelevant to reliability of our scientific truth-tracking, or else it's question-begging about that reliability (by collapsing reliability into adaptiveness).

That's not the argument I'm making.

Bottom line: on evolutionary materialist assumptions, there's no good reason to think that cognitive equipment is particularly reliable. But that undermines the possibility of having a warranted belief in evolutionary materialism.

One more time, evolutionary selection did not select for scientific skills. Yet, we have developed scientific skills that are reliable; e.g. we can fly across the solar system. There is no doubt that if we knew more, we could accomplish more. That's quite beside the point. The fact that we can rely on our powers of observation and analysis to reliably learn about the universe, tells us that our skills have reliability outside of the narrow scope for which they were originally selected. We know our scientific skills have reliably answered questions about the nature of the universe (unless you doubt that we live in a sun-centered solar system and that the earth orbits the sun along with a number of other planets). Therefore, we have reason to rely on our scientific skills to continue to learn about the universe.

Now, what reason do we have to rely your untested speculations about MagicLand?
Printer Friendly | Permalink |  | Top
 
Stunster Donating Member (984 posts) Send PM | Profile | Ignore Thu Dec-23-04 02:28 AM
Response to Reply #153
155. No; really
Edited on Thu Dec-23-04 02:30 AM by Stunster
Now, my argument is that infant motality in the US is as low as it is because our powers of observation are quite good; as we would expect them to be since they've been sharpened across billions of years of evolution. If god did it, why are infant mortality rates so much higher in non-technological societies?

I suspect that the good technological-society infant mortality rate has had a bad effect on the bad non-technological society infant mortality rate. In other words, it's not God's fault, it's ours. Or rather, the fault of our 'selfish genes'.

Our technology is of recent vintage on evolutionary timescales. That it is reliable (if it is) is remarkable, and suggests intelligent design of our cognitive faculties---there is no obvious connection between adaptiveness and, say, the ability to understand properly the operations of particle accelerators. As Eugene Wigner put it, the efficacy of our mathematical reasoning is 'unreasonable'. Wigner had it almost exactly correct----we have no right to expect our reasoning to be so extraordinarily efficacious if it is the by-product of materialistic evolution, and not intelligent design/creation.

Secondly, this does nothing to prove that the human race couldn't be tremendously more adept at enhancing our short-term survival.

That's quite beside the point.

In the Christmas spirit, let me shout, "Oh no it's not!" It's precisely the point.

One more time, evolutionary selection did not select for scientific skills. Yet, we have developed scientific skills that are reliable; e.g. we can fly across the solar system. There is no doubt that if we knew more, we could accomplish more. That's quite beside the point. The fact that we can rely on our powers of observation and analysis to reliably learn about the universe, tells us that our skills have reliability outside of the narrow scope for which they were originally selected.

In other words, on evolutionary materialist assumptions, there is no reason---no warrant---to think that our scientific beliefs are well-founded. Not only are adaptive behavior and scientific belief logically independent, but now you make the stronger point for me--that our scientific beliefs, and their reliability, are completely unnecessary from the standpoint of natural selection. But then a fortiori are they unwarranted; and given the size of even the observable universe, our ability to navigate our solar system may be as intellectually unremarkable as a baby's ability to roam across a back yard or garden.

Though as for our ability to navigate the solar system--though I take your point--you may want to check out the latest difficulties:

http://www.latimes.com/news/local/la-sci-pioneer21dec21,1,957815.story

http://www.google.com/search?hl=en&q=Pioneer+anomaly&btnG=Google+Search
Printer Friendly | Permalink |  | Top
 
Jim__ Donating Member (1000+ posts) Send PM | Profile | Ignore Sun Dec-26-04 11:06 AM
Response to Reply #155
171. I agree. It's technology, not god.
I suspect that the good technological-society infant mortality rate has had a bad effect on the bad non-technological society infant mortality rate. ...

It's not difficult to check these numbers; so, it's somewhat baffling that you choose to guess rather than check. But, your guess would be wrong. Check here: Table of Infant Mortality Rates; if that link doesn't work, try thisAnd follow to tables. The infant mortality rate is down rather dramatically across the world from 1960 to 2002; from 196 in 1960 to 82 in 2002.

...In other words, it's not God's fault, it's ours. Or rather, the fault of our 'selfish genes'. ...

Well, my argument all along has been that it's us, not god, that's responsible for technology. So, it seems that once again you're conceding the point.

Secondly, this does nothing to prove that the human race couldn't be tremendously more adept at enhancing our short-term survival.

That's quite beside the point.

In the Christmas spirit, let me shout, "Oh no it's not!" It's precisely the point.

No. The point is that technology and science are by-products of skills that were selected for by natural selection; specifically the skills of observation and analysis. That "better" skills may have developed has nothing to do with this point. Indeed, if you argue that much better skills could have develolped (I don't disagree) then I am quite entitled to ask why a god who wanted us to develop these skills had us develop sub-optimal skills.

In other words, on evolutionary materialist assumptions, there is no reason---no warrant---to think that our scientific beliefs are well-founded. ...

We believe our scientific skiils are well founded based on empirical evidence; for instance, see above on infant mortality rates between 1960 and 2002.

Not only are adaptive behavior and scientific belief logically independent, but now you make the stronger point for me--that our scientific beliefs, and their reliability, are completely unnecessary from the standpoint of natural selection.

Not quite. While our scientific abilities are unnecessary from the standpoint of natural selection, scientific abilities themselves rest on skills that are selected for, again, observation and analysis.

Once again, your post does nothing to defend your position; indeed, in places it contradicts your position. You tell me that there are difficulties with the materialist position. Yes, our understanding is incomplete. But, difficulties with the materialist position do not validate "other" positions, like the "god" position, which have more difficulties and lead us nowhere toward an understanding of the universe.

Materialism leads somewhere. So far, it has led to a tremendous growth in our understanding of the universe. Speculations about god lead nowhere except to further speculations.



Printer Friendly | Permalink |  | Top
 
ulysses Donating Member (1000+ posts) Send PM | Profile | Ignore Wed Dec-15-04 09:45 PM
Response to Original message
90. "agnostically silly"? Come now, Rev.
You're missing the point of agnosticism, and I think you know it.
Printer Friendly | Permalink |  | Top
 
enki23 Donating Member (1000+ posts) Send PM | Profile | Ignore Thu Dec-16-04 10:58 PM
Response to Reply #90
110. no i'm not.
there was a rule for writing we learned in early high school. we called it the "purple carrot" rule. the rule said, essentially, that there was no point in writing "he bit into the orange carrot." it's useless verbiage. we all know carrots are orange. if the carrot is purple, then you've gotta mention it. because it matters. well, uncertainty in life is the orange in a carrot. it goes without saying. i'm not certain i'm not just stuck on the finger of a cosmic bozo performing a puppet play. i'm not *certain*. but i'm pretty god damned sure, about as sure as i am that there is no christian god.

and i'm not going to pretend that there is any *serious* chance either of those is true, any more than i'm going to pretend i have a real chance i'll find a winning lottery ticket on my pillow. there's a chance for anything, but some are clearly so remote there's no point worrying about them. the various christian gods are in that category. as are the jewish gods, the muslim gods, the hindu gods, the buddhas, the saints, the angels, and all the rest. every time a "god" is mapped out, its probability of existance goes down near zero.

all we have left is speculating wildly about "a god." the one we can't define well enough to assess. but that question is like asking whether 'round' tastes like 'chocolate.' it doesn't mean anything, unless you're brain damaged.
Printer Friendly | Permalink |  | Top
 
ulysses Donating Member (1000+ posts) Send PM | Profile | Ignore Fri Dec-17-04 09:46 PM
Response to Reply #110
119. but if uncertainty in life is the orange in the carrot,, the given,
isn't agnosticism on quite a bit more solid ground than atheism?
Printer Friendly | Permalink |  | Top
 
SlackJawedYokel Donating Member (446 posts) Send PM | Profile | Ignore Sat Dec-18-04 11:41 AM
Response to Reply #119
121. My question for Agnostics is this.
Are you as "uncertain" about all unknowns?

I suspect the answer is no and if it is, why not?

If you are as uncertain about all unknowns how do you function?

It seems to me that such a position is only viable when it is applied to the preconceived notion of "god"(in its infinite definitions) and nothing more... at least nothing useful... and presupposes the validity of the claim to "god"(in its infinite definitions).

I've reframed the question of my Atheism this way.
Theists(and actually anyone who posits anything supernatural) say that "god"(in its infinite definitions) or the supernatural exists.
Ok, fine.
Define this "god" and then provide evidence for it.
Provide evidence for this supernatural thing that *anyone* can experience.

And when they point to "everything"(as it typically the case for the unknowing Pantheist), I'll ask for the exact evidence that points to this "god" as opposed to natural forces.
The very specific "god" fingerprint, if you will.

And then I'll point out the well-documented history of humanity creating and destroying its "gods" as social fictions.

Cletus
Printer Friendly | Permalink |  | Top
 
ulysses Donating Member (1000+ posts) Send PM | Profile | Ignore Sun Dec-19-04 10:23 PM
Response to Reply #121
133. no.
Are you as "uncertain" about all unknowns?

No. Most of the unknowns I come across are things that I believe are knowable but about which I simply don't have the necessary knowledge.

I suspect the answer is no and if it is, why not?

See my first answer.

It seems to me that such a position is only viable when it is applied to the preconceived notion of "god"(in its infinite definitions) and nothing more... at least nothing useful... and presupposes the validity of the claim to "god"(in its infinite definitions).

It seems to me that if one is going to address the question of a god at all, one should probably proceed from at least a generalized common definition. It would be easy, yes, to declare oneself an atheist if one declares that "god" is a pink elephant dancing atop one's head, but I don't see how that is particularly useful.

I'm supposing that the rest of your post doesn't really apply to me, since I, personally, don't posit the existence of a supernatural being. Then again, I don't posit its non-existence either. I genuinely don't care one way or the other about your atheism - knock yourself out; you may well be correct - but I think you're barking up the wrong tree in demanding "evidence".
Printer Friendly | Permalink |  | Top
 
SlackJawedYokel Donating Member (446 posts) Send PM | Profile | Ignore Mon Dec-20-04 11:41 AM
Response to Reply #133
135. Short and to the point... I like that.
No. Most of the unknowns I come across are things that I believe are knowable but about which I simply don't have the necessary knowledge.
Then why is "god" different?
Is it because you've decided that "god"(in its infinite definitions) is unknowable in advance or better yet, by definition(which leads directly to the Invisible Pink Unicorn)?

Understand that I'm aware and actually accept the divergence of mythos and logos(as defined by Karen Armstrong, anyway) and if kept within their perspective arenas are quite useful.
There is a need, perhaps genetic even, for humans to believe in something greater than themselves... an offshoot of our tribal/co-operative urges perhaps, and I don't have *any* problem with that need to believe or the object of that belief, whatever that may be.
I comprehend the functional necessity of such a thing.
Just don't try to tell me it is a *real* object without evidence to support it.

It would be easy, yes, to declare oneself an atheist if one declares that "god" is a pink elephant dancing atop one's head, but I don't see how that is particularly useful.
Actually, it doesn't matter if "god" is defined as a pink cranial-dancing elephant, an omni-powerful war god, or Bob.
If someone posits it I wanna see the evidence for it first.

I'm supposing that the rest of your post doesn't really apply to me, since I, personally, don't posit the existence of a supernatural being.
Ok and yes, it was a general sort of posting.

Then again, I don't posit its non-existence either.
And just for kicks, what would be your criteria for determining the existance of the supernatural?
How would you distinguish it from something natural?

I genuinely don't care one way or the other about your atheism - ...
This is good, because I don't ask anyone to care, one way or the other.
But then *caring* was never the issue. :D

knock yourself out;
Thanks I will. At least so long as America remains a semi-free country.

you may well be correct -
I'd rather be *proved* right/wrong than simply told I was right/wrong.

but I think you're barking up the wrong tree in demanding "evidence".
Why?

Wouldn't you want to see something concrete if someone claims something to be true without providing any evidence?
Especially when that "something" is being used as a justification for social engineering?

I mean I could claim that clams are omnipotent creatures who've spoken to me and demand that everyone pray to them in public schools.

Cletus
Printer Friendly | Permalink |  | Top
 
trotsky Donating Member (1000+ posts) Send PM | Profile | Ignore Mon Dec-20-04 01:17 PM
Response to Reply #135
136. OMG, I love clam chowder!
Will the clams forgive me? Please please please say yes!
Printer Friendly | Permalink |  | Top
 
SlackJawedYokel Donating Member (446 posts) Send PM | Profile | Ignore Mon Dec-20-04 01:47 PM
Response to Reply #136
137. You poor Hell Bound Sinner!
I will pray to the Clam Gods on your behalf.
Prepare yourself for their wrath!
:D

Cletus


Printer Friendly | Permalink |  | Top
 
ulysses Donating Member (1000+ posts) Send PM | Profile | Ignore Mon Dec-20-04 08:27 PM
Response to Reply #135
138. I try.
Then why is "god" different?
Is it because you've decided that "god"(in its infinite definitions) is unknowable in advance or better yet, by definition(which leads directly to the Invisible Pink Unicorn)?


By definition. As to the "infinite definitions" thing that you keep bringing up, if god is a creation of man then there cannot, by definition, be infinite definitions of what god is as we are finite both in number and scope. If god is a concept that is true in any way outside of human creation, it may be that there are infinite definitions to that being (for lack of a better word), but I don't think that helps your argument much. :)

As to the Invisible Pink Unicorn, that seems like a tangent to me. Your further paragraph re: Karen Armstrong etc is relevant here. If people had posited Invisible Pink Unicorns for millennia, I suppose I would be an agnostic about Invisible Pink Unicorns. I'm not, as it happens.

Just don't try to tell me it is a *real* object without evidence to support it.

What agnostic has tried to tell you that???

And just for kicks, what would be your criteria for determining the existance of the supernatural?

I have none. I'm an agnostic. :D

Wouldn't you want to see something concrete if someone claims something to be true without providing any evidence?
Especially when that "something" is being used as a justification for social engineering?


Sure, if their claim is available to proof. If not, the question is still one of the social engineering rather than one of the validity of their underlying premise. Thus the separation clause.

To wit...

I mean I could claim that clams are omnipotent creatures who've spoken to me and demand that everyone pray to them in public schools.


So you could, and it would do me no harm until you and your bivalve minions stood for the next school board seat. :)
Printer Friendly | Permalink |  | Top
 
SlackJawedYokel Donating Member (446 posts) Send PM | Profile | Ignore Tue Dec-21-04 01:29 PM
Response to Reply #138
143. Sorry for the length.
By definition.
Ah, well that solves that, don't it?

As to the "infinite definitions" thing that you keep bringing up, if god is a creation of man then there cannot, by definition, be infinite definitions of what god is as we are finite both in number and scope.
Only a little bit of a stretch on my part.
Humanity is both finite in number and scope, but since we are capable of a multitude of positions, some completely contradictory, I was simply covering all the bases.

If god is a concept that is true in any way outside of human creation, it may be that there are infinite definitions to that being (for lack of a better word), but I don't think that helps your argument much. :)
I disagree completely... of course. :)
If "god" is a concept that is true then "god" becomes a real thing and there can then only be one(or at least a limited number) definition for it.
It can then be quantified and studied as can anything we can define.

However, given that the only thing currently true(just waiting for that evidence) about the concept of "god" is that it is a human construct completely lacking all supporting evidence, the idea that the definition of "god" changes, literally from person to person, makes it a work of fiction and therefore a non-real thing as "true" as any other work of fiction.
Pi is not 3, no matter what the bible says, nor is it 5, 12, 2.2, 0 or any other number that individuals are comfortable with and therefore think is "true".

As to the Invisible Pink Unicorn, that seems like a tangent to me.
Nah, in fact it is directly related to your contention(correct me if I'm wrong, please) that "god" is unknowable *by definition*.
The Invisible Pink Unicorn(may her hooves trod only virgin grass) is *invisible* AND *pink*, by definition.
Invisible because we can't see her and pink because we have faith.

Your further paragraph re: Karen Armstrong etc is relevant here.
Thanks. I try. :D

If people had posited Invisible Pink Unicorns for millennia, I suppose I would be an agnostic about Invisible Pink Unicorns. I'm not, as it happens.
And yet I am baffled as to why you are not.
What makes "god" so much more special than Invisible Pink Unicorns?

Oh and it isn't just that people have posited Invisible Pink Unicorns for millennia, it is that the Invisible Pink Unicorn is the creator of all things and has set everything up this way so as to trick theists.
She has a wicked sense of humor.

Ok, now back to the real issues... why does someone positing something "for millennia" make it more valid than someone positing something today?
Variation on fallacy ad populum?

What agnostic has tried to tell you that???
Sorry, didn't mean to lose you there. Wasn't referring to Agnostics was referring to believers.

I have none. I'm an agnostic. :D
See and this strikes me almost as badly as the believers.
How *could* you know then?

Because at some point the idea that something is unknowable *by definition* makes it illogical(see Invisible Pink Unicorn) or at best embraces a form of willful ignorance... you may not ever know something for certain but you certainly won't if you refuse to look.

And along the way the God of the Gaps fallacy comes to play.
A weak theist bozo who claimed he was Agnostic actually contended that "we can't know" because "god" might be on the other side of the universe(thereby redefining "god" as less than omni-powerful).
What sort of "god" has to hide?

Sure, if their claim is available to proof.
*IF*?
There wouldn't *be* an issue if there was an *if*, even!
:D

If not,
Have you *ever* seen *any* theist present concrete, valid evidence for the existence of *any* "god"?

"If", pee-shaw!

the question is still one of the social engineering rather than one of the validity of their underlying premise.
And if the social engineering affects no one else but you and yours negatively?

Thus the separation clause.
Novel response, I'll admit.
The "separation clause" was hardly designed to keep believers from attempts at social engineering based on their beliefs... hell, it was really just a reminder to avoid hereditary monarchies.

So you could, and it would do me no harm until you and your bivalve minions stood for the next school board seat. :)
Exactly.
And then what?
Or, if like the Mormons in Utah and Arizona, we completely infiltrate every aspect of government and use the democratic process for social engineering purposes?

You will be *bathing* in garlic-butter sauce buddy, that's what!
:D

Cletus
Printer Friendly | Permalink |  | Top
 
ulysses Donating Member (1000+ posts) Send PM | Profile | Ignore Tue Dec-21-04 02:42 PM
Response to Reply #143
144. not a problem at all.
I may match length for length (!) this time.

Ah, well that solves that, don't it?

I don't really see how it can be otherwise. We're discussing the existence/non-existence of a being out-of-time. We exist in time. I may find particular descriptions of that being as unlikely (and I do, be it a bearded old man or your clams) but we don't have the capacity to experience the eternal, should "the eternal" itself even exist.

I'll grant you that proposing the existence of a being we can't even indirectly experience isn't a rational proposal, but we're not entirely rational beings.

If "god" is a concept that is true then "god" becomes a real thing and there can then only be one(or at least a limited number) definition for it.
It can then be quantified and studied as can anything we can define.


Must it be quantifiable in order to be true? What is truth? Are mine the same as yours?

:P

However, given that the only thing currently true(just waiting for that evidence) about the concept of "god" is that it is a human construct completely lacking all supporting evidence, the idea that the definition of "god" changes, literally from person to person, makes it a work of fiction and therefore a non-real thing as "true" as any other work of fiction.

That's a somewhat limited view, imo. Would you admit to the existence of other concepts, such as beauty, that lack such supporting evidence?

Pi is not 3, no matter what the bible says, nor is it 5, 12, 2.2, 0 or any other number that individuals are comfortable with and therefore think is "true".

Pi is not god, either. Apples and oranges.

The Invisible Pink Unicorn(may her hooves trod only virgin grass) is *invisible* AND *pink*, by definition.
Invisible because we can't see her and pink because we have faith.


Ok. Consider me a skeptic on the Invisible Pink Unicorn question, then. :)

What makes "god" so much more special than Invisible Pink Unicorns?

Semantics.

See and this strikes me almost as badly as the believers.
How *could* you know then?


You're looking at it from the wrong side. I'm agnostic because I don't believe that it is possible to know whether or not the supernatural exists. Why would I have a set of criteria that I have no reason to believe I can use?

Because at some point the idea that something is unknowable *by definition* makes it illogical

As I said above, the idea of god is not rational, at least to my mind.

you may not ever know something for certain but you certainly won't if you refuse to look.

I don't think you quite understand my outlook. I enjoy nothing more than discussing religion with both intelligent believers and nonbelievers. I do look. Should I, some day, stumble upon an answer, I will perhaps no longer be an agnostic. Until then, I'm quite happy where I am.

A weak theist bozo who claimed he was Agnostic actually contended that "we can't know" because "god" might be on the other side of the universe

That's stupid. I hope you don't imagine that I hold to anything like that.

There wouldn't *be* an issue if there was an *if*, even!

I suppose not. You asked. :)

Have you *ever* seen *any* theist present concrete, valid evidence for the existence of *any* "god"?

No, neither do I expect one to.

And if the social engineering affects no one else but you and yours negatively?

Me and mine who?

The "separation clause" was hardly designed to keep believers from attempts at social engineering based on their beliefs... hell, it was really just a reminder to avoid hereditary monarchies.

I'm not so sure about that, but the thing is there nonetheless, and it's where I prefer to put my energies in such matters.

Or, if like the Mormons in Utah and Arizona, we completely infiltrate every aspect of government and use the democratic process for social engineering purposes?

It's one of the dangers of democracy, I'll admit, and why we should be vigilant. You're not going to get rid of religion in people, though, and therefore you're not going to get rid of it in politics.

You will be *bathing* in garlic-butter sauce buddy, that's what!


I can imagine far worse ways to go out. :9
Printer Friendly | Permalink |  | Top
 
enki23 Donating Member (1000+ posts) Send PM | Profile | Ignore Sun Dec-19-04 02:18 PM
Response to Reply #119
129. no. because it's redundant.
i believe, based on lots of evidence and reasoning, that there is no creator god, or controller god. i believe this in the same way i believe human beings are the product of evolution, partially driven by natural selection. i don't consider myself an "evolution agnostic." and i don't consider myself a god agnostic. both for the same reasons.

but that's just me. you may think the evidence for god is just compelling enough that it merits a little more consideration than i give it. if we want to consider agnostics to be those who are 40-90% sure there is no god (or 10-60% sure there is) then fine. or we can draw the mark wherever we like. but as i'm about 99.99999999% sure none of the christian gods exist, and about equally sure of any other given god, i feel pretty confident in calling myself an atheist. just as i'm 99.9999999999% sure i'm not actually a butterfly dreaming i'm a human.

sorry about sounding all growly, by the way, uly. i'm a pissed off american these days, and ready to growl at damned near anything. ;-)
Printer Friendly | Permalink |  | Top
 
ulysses Donating Member (1000+ posts) Send PM | Profile | Ignore Sun Dec-19-04 10:03 PM
Response to Reply #129
132. ok.
i believe, based on lots of evidence and reasoning, that there is no creator god, or controller god. i believe this in the same way i believe human beings are the product of evolution, partially driven by natural selection. i don't consider myself an "evolution agnostic." and i don't consider myself a god agnostic. both for the same reasons.

The problem I have here is that it's all apples and oranges. I have a hard time seeing belief in evolution, a scientific fact, and belief that there is no god, a proposition unavailable to science on either end, through the same lens.

if we want to consider agnostics to be those who are 40-90% sure there is no god (or 10-60% sure there is) then fine. or we can draw the mark wherever we like.

That's where I think you're misunderstanding agnosticism. There is no mark. There is no percentage of likelihood that I take into account. For me, the truth regarding any proposition regarding a deity or any being out of time is unknowable to me because I exist *in time*. I simply cannot know, and I'm fine with that.

sorry about sounding all growly, by the way, uly. i'm a pissed off american these days, and ready to growl at damned near anything. ;-)

No need to apologize, my friend. :)
Printer Friendly | Permalink |  | Top
 
Tafiti Donating Member (1000+ posts) Send PM | Profile | Ignore Mon Dec-20-04 08:55 PM
Response to Reply #132
139. I'm also truly an agnostic.
I do however, depending on my mood, call myself an atheist at times as well. I guess it depends on who I'm talking to.

For me, as I suspect it is for you, it doesn't really matter to me whether there is a "God" or not. I am, however, 100% sure that the Christian's (and Jew's and Muslim's) big guy with a beard passing judgment, and punishing, and isolating, and killing, et. al., does not exist. But if we're talking about a "creative force", if you will, OK sure, I can at least entertain the idea. After all, who can really KNOW for sure? But either way, who cares? I don't. If such a "god" exists, then it began the process of evolution and the rest is history. No involvement in human affairs, no prayer-taker/answerer, again, no big guy w/ a beard upstairs.

I do enjoy the logical discourse on the existence or non-existence of God here recently.



Printer Friendly | Permalink |  | Top
 
lazarus Donating Member (1000+ posts) Send PM | Profile | Ignore Tue Dec-21-04 05:42 AM
Response to Reply #119
141. You're missing the point
agnosticism isn't in the same category as atheism. Atheism and theism are two poles. Agnosticism is simply a descriptor of these. One can be an agnostic atheist, or a gnostic atheist. One can be an agnostic theist, or a gnostic theist.

Agnostic/gnostic is a descriptor of the belief. It's like saying someone is "very". Very what? Agnostic about what? I can be agnostic about anything; I can only be atheistic about god(s).
Printer Friendly | Permalink |  | Top
 
ulysses Donating Member (1000+ posts) Send PM | Profile | Ignore Tue Dec-21-04 06:51 AM
Response to Reply #141
142. "It's like saying someone is "very". much disagreed, laz.
Agnosticism isn't a lesser atheism, or a stopping point on the way to atheism or theism. I understand that agnosticism applies to other concepts besides god, but when applied to god, the belief that the existence or non-existence of god is unknowable is its own position.
Printer Friendly | Permalink |  | Top
 
lazarus Donating Member (1000+ posts) Send PM | Profile | Ignore Tue Dec-21-04 05:25 PM
Response to Reply #142
145. Agnosticism has nothing to do with atheism or theism
except as a descriptor.

Here's how it works:

Do you believe in god? Y/N

That's the basic atheist/theist question.

Now, add in the question Do you have knowledge that god exists/doesn't exist, or can that knowledge be attained?

That introduces agnosticism/gnosticism.

See the difference?
Printer Friendly | Permalink |  | Top
 
ulysses Donating Member (1000+ posts) Send PM | Profile | Ignore Tue Dec-21-04 06:01 PM
Response to Reply #145
146. there's a third answer to your original question, though.
"I don't know." :)
Printer Friendly | Permalink |  | Top
 
lazarus Donating Member (1000+ posts) Send PM | Profile | Ignore Wed Dec-22-04 04:26 AM
Response to Reply #146
147. Heh
Good try, but...

It's a simple yes or no question. If you don't know if you believe in a god, then you don't actively believe in one, therefore you're an atheist.

It's not so bad, being an atheist. One nice thing is, you can always change your mind if you want.

Atheists run the gamut. There are atheists who simply don't believe, and there are atheists who firmly believe god doesn't exist.

Me, I go way on past all of these. As Harlan Ellison once said, I'm so extreme on this issue they haven't come up with a name for it yet. :evilgrin:
Printer Friendly | Permalink |  | Top
 
ulysses Donating Member (1000+ posts) Send PM | Profile | Ignore Wed Dec-22-04 08:14 AM
Response to Reply #147
149. ?
If you don't know if you believe in a god, then you don't actively believe in one, therefore you're an atheist.

Seems a bit overbroad a definition. I'd be an odd sort of atheist to admit even the possibility of the existence of god, wouldn't I?

As Harlan Ellison once said, I'm so extreme on this issue they haven't come up with a name for it yet.

You'll always be "Notorious L.A.Z." to me. :D
Printer Friendly | Permalink |  | Top
 
lazarus Donating Member (1000+ posts) Send PM | Profile | Ignore Wed Dec-22-04 05:45 PM
Response to Reply #149
151. Not at all
Most people think atheists actively disbelieve in a god.

There's a key difference depending on the placement of the negative. Observe:

I do not believe in a god.

I believe there is not a god.

Those are completely different statements. Both are atheistic. Some call them soft atheism and hard atheism.

Could a god of some form exist? I guess so. I don't believe one does exist, though. So I'm an atheist, until further evidence comes in.
Printer Friendly | Permalink |  | Top
 
ulysses Donating Member (1000+ posts) Send PM | Profile | Ignore Wed Dec-22-04 05:48 PM
Response to Reply #151
152. ok.
I do not believe in a god.

I believe there is not a god.


I get that. Neither statement is one of agnosticism, though. "I do not believe in a god", your soft atheism, is not the same as "I do not, and can not, know if a god exists".
Printer Friendly | Permalink |  | Top
 
lazarus Donating Member (1000+ posts) Send PM | Profile | Ignore Fri Dec-24-04 12:06 AM
Response to Reply #152
156. It's not exclusionary, though
"I do not believe in god" makes no claim whatsoever about knowledge. That's where the agnosticism comes in.
Printer Friendly | Permalink |  | Top
 
Az Donating Member (1000+ posts) Send PM | Profile | Ignore Fri Dec-24-04 08:20 AM
Response to Reply #152
158. Look to the epistemology of the words
See the thread clarity below.
Printer Friendly | Permalink |  | Top
 
Az Donating Member (1000+ posts) Send PM | Profile | Ignore Fri Dec-24-04 08:18 AM
Response to Reply #142
157. Clarity
Lets look at the words.

Theist. A theist is someone that believes in god or gods. Simple enough. We don't know which particular god(s) or why they believe. We only know that they posess a belief in a god(s). We don't even know if they are part of some particular religion.

Atheist. Derived from the word theist combined with the prefix 'a' meaning 'without'. Thus an atheist is simply someone that is without a belief in god or gods. Again we know nothing beyond this factor. They may be religious (ie Buddhist) or they may simply be ignorant of the concept. They may have pondered the subject for quite some time. Without further information we cannot know.

Gnostic. Meaning to know something. In an absolute sense. This is not about belief. It is about knowledge. A gnostic is someone that has direct knowledge of a matter.

Agnostic. From the word gnostic with the prefix 'a' meaning without. An agnostic is someone without direct knowledge of a matter. On its own we cannot fathom what it is they are lacking knowledge of. It can be applied to either side of the matter as it applies to what a person knows and not what they believe.

Belief and knowledge are two different things. A person at any given moment can either believe in a thing or not believe in a thing. There is no capacity for the brain to posess both qualities. There may be a very low level of certainty concerning the degree of belief. But at any given moment the mind is going to be of one position or the other. You either do or do not believe a thing. If you are without a belief in god then you are an atheist. If you do believe there is a god then you are a theist.

If you have direct knowledge that there is a god then you are a gnostic theist. If you do not have direct knowledge or maintain some skepticism about the matter then you are an agnostic theist. If you have direct knowledge that there is no god then you are a gnostic atheist. If you do not have direct knowledge that there is no god then you are an agnostic atheist.

Printer Friendly | Permalink |  | Top
 
ulysses Donating Member (1000+ posts) Send PM | Profile | Ignore Fri Dec-24-04 10:25 AM
Response to Reply #157
161. some comments
Atheist. Derived from the word theist combined with the prefix 'a' meaning 'without'. Thus an atheist is simply someone that is without a belief in god or gods.

Is there room here for the two kinds of atheism that lazarus mentions? Particularly, is the lack of belief in god or gods the same thing as an active belief that there is no god or gods?

Gnostic. Meaning to know something. In an absolute sense. This is not about belief. It is about knowledge. A gnostic is someone that has direct knowledge of a matter.

In an absolute, but not necessarily *scientific* sense. From Merriam-Webster:

Etymology: Greek gnOsis, literally, knowledge, from gignOskein
: esoteric knowledge of spiritual truth held by the ancient Gnostics to be essential to salvation


(emphasis added)

My Liddel & Scott has this for :

a judicial inquiry / a knowing, knowledge / acquaintance with a person / a knowing, recognizing / a being known, fame, credit

I don't necessarily get the sense of knowledge as a result of direct observation in the use of the word.

Agnostic. From the word gnostic with the prefix 'a' meaning without. An agnostic is someone without direct knowledge of a matter. On its own we cannot fathom what it is they are lacking knowledge of. It can be applied to either side of the matter as it applies to what a person knows and not what they believe.

This cuts to the heart of the thing. I think you're separating belief and knowledge too strictly. Belief very often follows knowledge - I believe that black holes exist, although I cannot experience them directly, because I have knowledge of their effects, to give an example.

Black holes are natural phenomena. To the extent that I have a statable belief concerning god, it is that the word does not represent a corporal concept - therefore, knowledge of god via evidence is neither present nor absent. It's impossible. Thus faith.

Belief and knowledge are two different things.

I don't think they are, entirely.

A person at any given moment can either believe in a thing or not believe in a thing. There is no capacity for the brain to posess both qualities. There may be a very low level of certainty concerning the degree of belief. But at any given moment the mind is going to be of one position or the other. You either do or do not believe a thing.

But if, as you say, belief and knowledge are completely different things, I don't at all know that belief is binary in this way. I've found that my mind, anyway, is quite capable of simultaneously holding competing ideas, and I suspect we all have that capacity when abstract ideas such as "god" are concerned.

If you are without a belief in god then you are an atheist. If you do believe there is a god then you are a theist.

And if you believe that it is impossible to know one way or the other, then you are an agnostic. :)

If you have direct knowledge that there is a god then you are a gnostic theist. If you do not have direct knowledge or maintain some skepticism about the matter then you are an agnostic theist. If you have direct knowledge that there is no god then you are a gnostic atheist. If you do not have direct knowledge that there is no god then you are an agnostic atheist.

Again, this doesn't seem to hold unless you define knowledge and belief as polar opposites.

Would "faith" be a better word here than "belief"?
Printer Friendly | Permalink |  | Top
 
Az Donating Member (1000+ posts) Send PM | Profile | Ignore Fri Dec-24-04 12:29 PM
Response to Reply #161
162. Good questions, I shall try to address them
Edited on Fri Dec-24-04 12:30 PM by Az
Is there room here for the two kinds of atheism that lazarus mentions? Particularly, is the lack of belief in god or gods the same thing as an active belief that there is no god or gods?

Of course there is room. There are more than just those two as well. The issue here is on simple definitions verses groups and labels. An atheist by itself is just a simple term. It conveys very basic informat. It requires adjectives and other describers to increase our knowlegde of the individual. Thus if I said someone was a strong atheist we would then know that they did not believe in god and were prepared to offer arguments demonstrating the nonexistance of god.

I think much of the trouble has come from various groups trying to find acceptance and prominence in this complex society. They attempt to assert titles and names with authority. And language being the social construct it is the words associated with these take on new meanings. This is largely what has happened to these terms in question. Groups like American Atheists redefine the terms to fit their own thinking. Thus atheist takes on the vaneer of aggressive individuals that deny the existance of god.

In an absolute, but not necessarily *scientific* sense. From Merriam-Webster:

Etymology: Greek gnOsis, literally, knowledge, from gignOskein
: esoteric knowledge of spiritual truth held by the ancient Gnostics to be essential to salvation


This gets us back to the label issue. The Gostics took their name from the word gnostic. That is there is a basic simple word gnostic and there is a group called the Gnostics. Madonna does not mean a woman that grew up in Michigan and became a superstar by wearing revealing clothing.

My Liddel & Scott has this for :

a judicial inquiry / a knowing, knowledge / acquaintance with a person / a knowing, recognizing / a being known, fame, credit

I don't necessarily get the sense of knowledge as a result of direct observation in the use of the word.


To take directly from the definition given: knowing, knowledge / acquaintance with a person / a knowing, recognizing / a being known. These terms aim directly to the core of the issue. I can say I believe there is a man in Russia named Putin but I have never known the man or had aquaintance with him. I have knowledge based on second hand at best information but this is not direct knowledge.

This cuts to the heart of the thing. I think you're separating belief and knowledge too strictly. Belief very often follows knowledge - I believe that black holes exist, although I cannot experience them directly, because I have knowledge of their effects, to give an example.

Black holes are natural phenomena. To the extent that I have a statable belief concerning god, it is that the word does not represent a corporal concept - therefore, knowledge of god via evidence is neither present nor absent. It's impossible. Thus faith.


Knowledge leads to belief. The thing of it is that knowledge is based on informative issues while belief is predicated on emotional acceptance of issue. It has to do with the way our brains work. We accumulate information and knowledge all the time. This knowledge comes tagged with emotional relevance. Fire HOT. Stick falling somewhere off in the distance not so important. The degree of emotional relevance is recorded in our minds along with the knowledge itself. As we form our identity we gather up these accumulated bits of knowledge and weigh the emotional relevance of them. We compare competing concepts with the emotional weight being the deciding factor. Then when the scales stabelize we discard or bury the bits that do not fit the balance of our emotional weighting. This is how we come to know our beliefs.

Knowledge accumulates. Whether it is true or not. Repitition of ideas increase their emotional relevance. Association with other positive events also increase their emotional relevance.

Knowledge derives from events and imparted information. Belief derives from sorting our own internal accumulation of information. What we make of the knowledge we have.

But if, as you say, belief and knowledge are completely different things, I don't at all know that belief is binary in this way. I've found that my mind, anyway, is quite capable of simultaneously holding competing ideas, and I suspect we all have that capacity when abstract ideas such as "god" are concerned.


Belief is binary in that the mind cannot simultaneously maintain two opposing beliefs. This does not mean it can't change its position over time. There are a miriad of ideas and thoughts flowing through the brain. It is the most complex thing we know of. All knowledge contained and remembered does not present itself at the same time. The dominant or more emotionally relevant issues take precidents. But the slower a less emotionally compelling eventually show up and vie for dominance. And if over time enough accumulates it can cause a shift.

Consider why so many lose faith at junctions in their life when tradgedy occurrs. It is because a major emotional shift has occurred in their mind. Things they had strong ties to emotionally can suddenly be overwhelmed by the impact and massiveness of the tradgedy. This sudden shift can rip their beliefs away from them. It is because their mind no longer favors belief in the way it used to and has a new balance.

Thus we see belief hinges on emotional balancing. And at times the mind can be close to balance but forces a resolution. Thus a person one moment can believe but then even the impact of questioning whether they believe or not can change the balance.

Would "faith" be a better word here than "belief"?

Faith is a number of things. Mark Twain suggested it was believing in what you know ain't true. I suspect faith is a reflection of what a person has learned they are supposed to believe. It forms the pattern they try to fit their beliefs to. It can become the thing which shatters their beliefs as well when they find their beliefs do not fit what their faith is supposed to be. Faith is a shell. If their beliefs fit their faith they find contentment. But when their beliefs do not fit their faith they find stress and depression.

Printer Friendly | Permalink |  | Top
 
ulysses Donating Member (1000+ posts) Send PM | Profile | Ignore Fri Dec-24-04 12:50 PM
Response to Reply #162
163. thanks.
I think much of the trouble has come from various groups trying to find acceptance and prominence in this complex society. They attempt to assert titles and names with authority. And language being the social construct it is the words associated with these take on new meanings. This is largely what has happened to these terms in question. Groups like American Atheists redefine the terms to fit their own thinking. Thus atheist takes on the vaneer of aggressive individuals that deny the existance of god.

Much the same thing happens with "agnostic" (and "Christian", and...). That's partly why I think it can be limiting to make strict definitions based on ancient semantics.

Madonna does not mean a woman that grew up in Michigan and became a superstar by wearing revealing clothing.

Sure it does, at least as much as it means anything else!

I can say I believe there is a man in Russia named Putin but I have never known the man or had aquaintance with him. I have knowledge based on second hand at best information but this is not direct knowledge.

But you would recognize him from that secondhand knowledge, would you not? My point is that "gnosis" includes, but isn't limited to, direct experience.

I would respond in greater depth, but I'm trying to get out the door to head to my mother-in-law's for the holiday. David Hume is sitting somewhere wondering if she exists. ;-)

I'll be back on Sat. night or Sunday. Talk to you then.
Printer Friendly | Permalink |  | Top
 
ulysses Donating Member (1000+ posts) Send PM | Profile | Ignore Fri Dec-24-04 05:25 PM
Response to Reply #162
168. seems I have a little more time before I leave.
Consider why so many lose faith at junctions in their life when tradgedy occurrs. It is because a major emotional shift has occurred in their mind. Things they had strong ties to emotionally can suddenly be overwhelmed by the impact and massiveness of the tradgedy. This sudden shift can rip their beliefs away from them. It is because their mind no longer favors belief in the way it used to and has a new balance.

This is awfully mechanistic, and I'm not sure where the balance thing comes from. Seems to me that when someone does lose their faith during a time of tragedy, it's because they believed that such things wouldn't/shouldn't happen to *them*, not because their mind is out of whack.

It's also worth noting that many folks' faith also *increases* during a tragedy, even though that often requires holding to two competing ideas - that god is good, and that god is also capable of unleashing some pretty harsh stuff on folks. One may sort of resolve that dissonance into mysticism of a kind or in other ways, but those competing ideas still remain.

My mom is somewhat religious, and became quite a bit more so when I was eleven and had cancer. I don't know what she would have done had I died, but I suspect that she would not have lost her faith. As it happened, the experience deepened it, even before I was out of the woods.

Anyway...
Printer Friendly | Permalink |  | Top
 
Az Donating Member (1000+ posts) Send PM | Profile | Ignore Sun Dec-26-04 03:56 PM
Response to Reply #168
172. It is dependent
A tradgedy imparts a tremendous emtional shift. It is not necissarily slated against a particular belief. But it can induce a shift. It entirely depends on how the mind takes in the event and relates it to their existing beliefs.

It is not an instantaneous process. It is organic and not always predictable. It takes place in the most complex structure we know of. It is mechanistic and like all extremely complex mechanistic processes it is upredictible because of the Chaos effect.

We could sit down and sort through all the individual memes that are maintained and balanced by the mind and determine how they will interact with this new input. But the process becomes so complex so quickly that our ability to plot its path becomes quickly obscured. Thus a singular event cannot tell us absolutely how the individual is going to react.

Thus a family struck by tradgedy will have any number of varying reactions. Some may turn to their religious convictions even stronger while others may have their's shattered. Some may remain unphased if the tradgedy figures in with the existing perception of the world.
Printer Friendly | Permalink |  | Top
 
Stunster Donating Member (984 posts) Send PM | Profile | Ignore Wed Dec-22-04 01:29 PM
Response to Reply #90
150. Agnosticism and epistemology
It is a truism that what we know and can know depends on how we define knowledge. It might be thought that relativism follows from this,
but it doesn't unless all definitions are equally valid or equally adequate--and they're not.

In the context of knowledge-claims regarding God and other religious matters, it has been common in the English-speaking world to invoke the 18th century Scottish philosopher, David Hume. It is held that his strictures regarding knowledge-claims about God constitute powerful arguments against such claims. What is usually missed, or
not even known about, by many of those who invoke Hume is that his arguments are equally cogent against almost all scientific knowledge-claims, since if the arguments are valid, they also undermine all inductive and all causal reasoning. Notoriously,
Hume's arguments also undermine all claims to knowledge of the self, and all claims of personal identity--for example, that you know
or can know that you are the 'same person' you were ten years ago. Hume, in other words, is a thoroughgoing epistemological sceptic. He doesn't think that science can really be proved true in any strongly rational sense. It is simply a matter of
habitual ways of thinking, which we can't help having, but which, on his view, don't stand up very well to rational scrutiny upon more careful investigation.

"Reason is, and ought to be, the slave of the passions", he wrote. His target was rationalism as an epistemological theory--the idea that
reason is an independent source of genuine knowledge. In the place of rationalism, Hume would posit empiricism.

But the question which should naturally occur at this point is, what is the empirical justification for empiricism? And the answer,
of course, is, there isn't any. You don't and can't know that empiricism is true on the basis of sense perceptions of empirically observable or detectable phenomena. A moment's thought makes this obvious. Empiricism, in other words, is as much a product of reason as rationalism is, and must therefore be subject to the same strictures. Given empiricist criteria of knowledge, Hume's views don't pass muster either, if taken to be knowledge-claims about the nature of knowledge itself. The idea that only empirical observations constitute or ground justified beliefs is itself not the result of anyone's empirical observation of anything, ever. Any attempt to justify empirically our sensory, perceptual, and abstract cognitive apparatus runs into the problem of having to rely on the very cognitive apparatus that one is attempting to show provide reliable warrant for our beliefs. Such attempts must be circular, and thus logically invalid.

This consequence might not have worried Hume (though it's clear that it did) had he been content to say that all of science and (more
importantly for him) all of morality and one's very self cannot be objects of knowledge. (Hume held that the self is never observed--all that is observed are particular sensations, particular images, particular ideas, etc--but not the self supposedly having them). But of course, the radically sceptical implications of his epistemology
did bother Hume, and have bothered others since his time even more, especially as the culture has become more dominated by scientific knowledge-claims.

For Hume, causation, like the self, is never observed; it's just
one damn thing after another. For Hume, induction is never justified--just because the chicken has always been fed by the farmer every morning doesn't mean that the farmer won't ring its neck this morning. (There are troubling problems of circularity and self-referential incoherence with Hume's account, since he relies on induction to explain our belief in causation, and relies on causation to explain our belief in induction.)

There is a sense in which we don't know that there exist minds other than our own. It seems to be logically possible and coherent to suppose that there are no other minds than our own, and that what seem to be other people are in fact just zombies, with no inner experience or consciousness at all. But who thinks this? Solipsism is an extremely rarely held belief! But if that is what a radically empiricist definition of knowledge must lead to, doesn't that indicate that there's something wrong or highly inadequate about that
type of definition?

A very common but not very convincing idea is that unless you can demonstrate something beyond all possible or all reasonable doubt to everyone's satisfaction, then it can't be a item of knowledge.
But isn't this, in fact, a quite ridiculous idea? Ludwig Wittgenstein came to think so. He thought that it was nuts to think that you didn't know this was your hand, even though you can't 'prove' it. In fact, he thought it was not so much wrong to think that you might not know if this was your hand, but that it didn't even make sense to doubt it. No 'proof' offered (and what the heck could such a proof even look like?) could possibly be as strongly credible as the belief that this is your hand itself, without the proof.

Never mind God! What does it mean to say that you don't know that this is your hand, or that you don't know that there exists an external material world independently of your mind, or that you don't know that there exist minds other than your own? The Kantian or neo-Kantian thought (Kant having read Hume) is that the very possibility of language, meaning, experience, thought, and reasoning *presupposes* that you exist, in a material world, and that unless you're an amputee, that this is indeed your hand, and that other people have minds which can know this fact about you too.

Thus the mere fact of the logical conceivability of there not existing any bodies, or any minds, is surely not adequate grounds for doubting
claims to knowledge of the existence of bodies or of minds in general. But if this is so, then the mere fact of the logical conceivability
of God not existing is surely not adequate grounds for doubting claims to know that God exists. If this is supposed to be the justification for agnosticism, it's a lousy one.

When evaluating knowledge-claims, we must look at what is reasonable to believe given the necessary background presuppositions of all possible experience, what makes sense of all the data, what best explains the data, what is logically coherent, logically elegant, logically predictive of phenomena, and so forth. Knowledge is a holistic enterprise. It cannot be reduced to just pure logic, nor pure observation. It cannot be held to impossibly high standards of demonstrability. And it must rely on faith---faith in the general reliability of our cognitive equipment, including both our powers of inference and our powers of observation, since these powers cannot be demonstrated systematically to be reliable, since any purported demonstration would itself rely on them.

There is an argument deriving from Descartes, a sophisticated version of which has been developed by Alvin Plantinga in modern times,
that for knowledge to be possible at all, God must exist. Plantinga develops the argument in his book WARRANT AND PROPER FUNCTION, and in a number of related articles. Basically, he argues that for a belief to be rationally warranted, we must have good grounds for believing that our belief-formation system is functioning
properly
, and that if it is functioning properly it will generally lead us to have true rather than false beliefs.

But, he asks, how could a properly functioning belief-formation system generally lead us to truth unless it was intelligently designed to do so? Could evolution, by sheer chance, and without intelligent guidance, cause the emergence of cognitive equipment which, when functioning properly, reliably led to truth?

Given that there are many ways that adaptive behavior could
have evolved without us having reliable cognitive faculties with regard to true belief (i.e. many adaptive behaviors are logically compatible with systematically false beliefs about the world), the belief that evolutionary processes operating by sheer chance alone could have generated such faculties is either improbable,
or its probability is at best unknowable. Just because (we think) we're smarter than monkeys or tigers or sharks or jackdaws doesn't mean that we are necessarily liable to be correct in our scientific beliefs---other Earth species presumably don't have scientific beliefs, but some of them are rather well adapted to the environment. In cosmic terms, we may be relatively stupid, even among 'intelligent species'. We just wouldn't know where we fall on the cosmic distribution curve of intelligence or cognitive reliability with respect to tracking the truth, if all intelligence and cognitive function was the product of undesigned material evolution.

Hence, if our cognitive faculties evolved by undesigned evolutionary chance, without any higher intelligent guidance, then we have
no knowledge of how reliable our cognitive faculties really are, and there's a possibly high probability that they are unreliable, especially with respect to the formation of beliefs which are not necessary for survival---such as scientific beliefs. But if we have faith that when our cognitive equipment functions in accordance with its nature (i.e. is not malfunctioning), it is generally reliable for scientific truth-tracking (which is not necessary for adaptiveness, as a vast number of species appear to demonstrate), then implicit in that is a belief that it is more probable than not that the evolutionary development of our cognitive powers was intelligently guided. Alternatively, our faith in the reliability of our cognitive powers with regard to non-religious subject-matter will itself be unwarranted, and hence irrational. Atheistic evolution gives us no reason to assume that they are reliable for tracking the truth about the world in general. But if so, we have no reason to assume that a belief in atheistic evolution will be reliable (for such a belief is certainly not essential to the survival of the species.)

A simplistic objection to knowledge-claims regarding God is that we do not see God, or otherwise have sense perception of God. This is the 'invisible pink dragon' objection. But it's a lousy objection because invisible pink dragons play no explanatory role in any theory of reality. There are, however, lots of invisible things which do play an explanatory role in a theory of reality and are consequently accepted
as being real. Physicists postulate many of them, even though they don't just happen to be invisible, but are intrinsically so: quarks, gluons, gravity, black holes, the electromagnetic field, and indeed anything that existed less than 300,000 years after the Big Bang, since it was only then that photons could travel freely and so render the universe visible at all. So we have plenty of examples of invisible entities being postulated by science--these are only a tiny few from a long list. They are accepted as legitimately real, because they are the conclusions of inferences to the best explanation of what we can see or experience physically.

But even all that we can experience physically is only part of what we ordinarily take to exist. Our conscious minds are not objects of physical experiences. But we take it that many minds exist. Ordinarily we take rational thoughts, emotions, concepts, numbers, goodness,
moral obligations, meanings, logical relations, beauty and so on to exist. These, not just physical things we can see, form a central part
of our experience of reality. Hence if physicists are allowed to postulate invisible things in making inferences to the best explanation of the physical phenomena we perceive (and they do, by the barrowload), then non-physicists need not in principle be precluded from postulating invisible realities in making inferences to the best explanation of these other, non-physical phenomena or 'data' (reason, value, consciousness, meaning, etc). I.e, there is no lapse in rationality here, at least none in principle. Unless of course you think that physicists are equally guilty of irrationality in postulating invisible entities to explain their data. You might not agree with a particular inference being made, but it would not be sufficient to justify this disagreement merely to say that the inference involved the postulation of an invisible entity. Such postulations are a commonplace in all of rational inquiry, including physical science.

Theism could thus be posited as the hypothesis which best explains a wide range of data, such as the existence and structure of logical and mathematical relations, thought, meaning, rationality in general, consciousness in general, moral phenomena, aesthetic phenomena, and the phenomena of value in general, as well as the marvellously intelligible and apparently finely-tuned laws and structure governing the physical universe itself.

Given this overall picture of what knowledge is, then agnosticism about God's existence is lent no support by any general epistemological considerations. That is, there appear to be no generally acceptable reasons for holding that God's existence, any more than the existence of anything else, is intrinsically unknowable. If anything is knowable, if in other words knowledge is possible at all, then it is possible to know whether God exists, and agnosticism in the strict sense is false. And indeed if knowledge is truly possible at all, then this fact itself may give us grounds for believing that it was consciously designed to be possible by an intelligent creator, and is not just a chance occurrence.

The agnostic can escape these conclusions, but only, I think, by giving up on the idea of knowledge in general, and not just specifically with regard to knowledge of God. In short, religious agnosticism, if it is to be rationally credible at all, must be a species of general epistemological skepticism.
Printer Friendly | Permalink |  | Top
 
Az Donating Member (1000+ posts) Send PM | Profile | Ignore Fri Dec-24-04 08:26 AM
Response to Reply #150
159. Militant agnostic
The belief that I don't know and neither do you.

Belief and knowledge can mix oddly. Belief can bring with it emotional necessity. Thus a person can so strongly need to believe a thing that they come to believe that they have knowledge of the matter.

Furthermore a person can become deluded as to the origin of their knowledge. Because they believe the nature of the source they cannot pierce its falsehood. Thus they base their claims of both knowledge and belief on this false base and proclaim themself both believer and gnostic.

The term gnostic refers to an absolute. It is an extreme. In a world of greys it is black and white. Our ability to know the truth is flawed by I own ability to collect information. Thus absolute knowledge eludes us. To such an extent that to make the claim that you truly know a thing may be an indication of delusion in and of itself.
Printer Friendly | Permalink |  | Top
 
Az Donating Member (1000+ posts) Send PM | Profile | Ignore Fri Dec-24-04 08:55 AM
Response to Original message
160. Proving a negative
You can prove some forms of negative statements. ie There are no married bachelors. There are no square circles. These are abstract constructs. In others words we define the rules that apply to them. And that is where the catch comes in. When we apply definitions to the things we are speaking of we contrain them. But when dealing with things in reality we have a bit more open of a probelm.

Prove to me there are no smurfs. Its theoretically possible. But to do so you would have to have the ability to show the entirety of the universe (and beyond).

You can create negative abstract constructs that are demonstrable. But when dealing with open ended reality you cannot prove a negative.
Printer Friendly | Permalink |  | Top
 
Zhade Donating Member (1000+ posts) Send PM | Profile | Ignore Fri Dec-24-04 03:43 PM
Response to Original message
165. ELVIS DIED FOR YOUR SINS!!!!
Interesting post, thanks for the words. Gonna go chew on 'em for a bit.

Printer Friendly | Permalink |  | Top
 
catbert836 Donating Member (1000+ posts) Send PM | Profile | Ignore Fri Dec-24-04 07:03 PM
Response to Original message
169. Every part of our body is evenly porportioned with the number 1.618
-Earth is in the perfect zone to inhabit life
-If the temperature of the universe at the Big Bang was just 0.000000001 degrees Fahrenheit hotter, none of us would be here.
-There's just the right conditions on Earth to support life like ours.
-If any of the laws of the universe that we take for granted (gravity, constants, inertia, etc.) were not in place we would not be here.

In all, I think that and more definetly suggests that the universe we live in had an intelligent creator.
Printer Friendly | Permalink |  | Top
 
Az Donating Member (1000+ posts) Send PM | Profile | Ignore Sun Dec-26-04 04:05 PM
Response to Reply #169
173. Cart before the horse
-Earth is in the perfect zone to inhabit life

The life that has developed on earth did so to fit the environment that exists here.

-If the temperature of the universe at the Big Bang was just 0.000000001 degrees Fahrenheit hotter, none of us would be here.

Again, the laws of physics define the life that forms. It is not life that defines the laws of physics.

-There's just the right conditions on Earth to support life like ours.

If you entered a forest with saws, hammers, and nails and were asked to build a house you wouldn't expect to find a house made out of aluminum when you were done. Life on earth developed to fit earth. Earth was not developed for the life that was going to come.

-If any of the laws of the universe that we take for granted (gravity, constants, inertia, etc.) were not in place we would not be here.

Again. We are the result, not the reason.

Go rent Carl Sagans Cosmos series. There is an episode that deals wonderfully with this issue. They suppose what life would be like if it had developed on Jupiter. Life develops to suit its environment.

And as to the supposed notion that the universe has some special wonderful constants there is a fairly well respected theory that there are a virtual infinite number of universes each with their own particular set of constants and dimensions. This would suggest that a universe with life in it is inevitible.
Printer Friendly | Permalink |  | Top
 
catbert836 Donating Member (1000+ posts) Send PM | Profile | Ignore Sun Dec-26-04 05:47 PM
Response to Reply #173
174. No, there would be no life, not just ours.
If some of the conditions I decribed before had existed, there wouldn't be absolutely ANY life in the universe, as far as our scientists can tell. For example, if the Big Bang had been 0.0000000001 or however small degrees higher. This would have caused not only human life developing, but also any other kind of life. Also, the laws of the universe that I mentioned, like gravity. No gravity=no life of any kind. There's no doubt about it. As far as developing to fit our surroundings, I agree with that. But I also think that the intelligent creator (God?) had just a little to do with setting that up.
Why do I believe in God? It's purely scientific. There had to be something to start something. Like the Big Bang. We know that all the matter and energy was once in a singularity that exploded into our universe, but where did that come from? And what caused it to explode? There had to be something to have caused that, or a first mover, is the scientific term. A lot of scientists I know are perfectly comfortable saying: "Yeah. The first mover is/was God." And I totally agree with them.
Printer Friendly | Permalink |  | Top
 
Az Donating Member (1000+ posts) Send PM | Profile | Ignore Mon Dec-27-04 02:23 AM
Response to Reply #174
176. Problems and possible solutions
You have created a circular argument. You establish a law that says there had to be something to start something. You claim the universe had to have been started by something because of this. So you claim god started it. But you leave open the question what started god. Because you established the law that there had to be something to start something you cannot simply say that god is exempt from this rule. To do so allows the other points to have access to the same excuse thus eliminating the law.

It honestly sounds like you are interesting in this issue but you don't seem to be fully up to date on current theories. This is perfectly understandable. Its not like scientists convene congregations and extoll their theories to people in attendance. Science can be difficult to understand and does not lend itself to propogation easily.

You seem hooked on the issue of creatio ex nilhio. Creation out of nothing. There are a number of potential theories that address this issue within the confines of the laws of nature. One example is called M-Theory. One aspect of M-Theory suggests that there are essentially an infinitie number of universes continuously giving rise to succesive universes. That is small quantum collapses of localised space give rise to extrusions consisting of entirely new dimensions. Each one complete with a variation of the constants and dimensions such as we have.

Thus the Big Bang is simply the rapid expansion of this universe as it extrudes into it's own dimense.

Now as to what it takes to create life. First we have to figure out what life is. Life is simply a state of matter. It is matter undergoing a process. Life as we know it is the result of a particular molecule replicating itself. Change the rules and constants and you change the nature of the types of processes that can occurr. You do not eliminate them.

And as to scientists that are comfortable with saying that god was the first mover. Thats fine and dandy. But its not science. Not every word that comes from a scientist is science. They guess and wonder too. Now if they had found some direct evidence that god did exist and were able to test it repeatedly, then they could form a theory about the positive existance of god in a scientific manner.
Printer Friendly | Permalink |  | Top
 
catbert836 Donating Member (1000+ posts) Send PM | Profile | Ignore Mon Dec-27-04 03:31 PM
Response to Reply #176
177. A few ones of my own
"You have created a circular argument. You establish a law that says there had to be something to start something. You claim the universe had to have been started by something because of this. So you claim god started it. But you leave open the question what started god. Because you established the law that there had to be something to start something you cannot simply say that god is exempt from this rule. To do so allows the other points to have access to the same excuse thus eliminating the law."

What started God? I don't know, and nor do I pretend to know. That would be very pretentious, because only God knows that, if there is one. But in the end, you can go as far back as "what created the creator of the creator of the creator of God" and there still has to be something that did it. I do believe something started God, but I don't know what.

"It honestly sounds like you are interesting in this issue but you don't seem to be fully up to date on current theories. This is perfectly understandable. Its not like scientists convene congregations and extoll their theories to people in attendance. Science can be difficult to understand and does not lend itself to propogation easily."

As for current theories, I have one of my own for you. That is the theory that if the universe, or as you suggest the multiverse is eternal then matter is also eternal, which all scientists agree that it isn't. So the "multiverse" isn't eternal. New theories in astrophysics seem to be pointing to a "beginning point" if you will in space/time, which means something started it.

Az, if you ask what created God then you obviously are lending yourself to the assumption that God exists. And if you ask what created God if there is one, obviously the answer is: nothing. A god or superior being is his own answer in this repect.

As for the M-Theory, it certainly is possible. But then one might ask, "What started these paralell universes?" So you see, something had to start something, even if there is these paralell universes.

I think your problem is that you see God in human terms, which is certainly understandable. But let's take a look at God outside of human terms. If God exists outside of our own certainly limited 3 dimensional universe, then the problem is solved. If God exists outside our terms, then he dosn't need to abide by our laws in our universe and cold have started us.
Printer Friendly | Permalink |  | Top
 
DemBones DemBones Donating Member (1000+ posts) Send PM | Profile | Ignore Sun Dec-26-04 09:09 PM
Response to Original message
175. You may believe anything you want, but you can not prove

God does not exist.

You are an atheist because you believe God does not exist. I'm a theist because I believe God exists. Neither of us can prove our belief but we can each be content in it.
Printer Friendly | Permalink |  | Top
 
DU AdBot (1000+ posts) Click to send private message to this author Click to view 
this author's profile Click to add 
this author to your buddy list Click to add 
this author to your Ignore list Thu Apr 25th 2024, 12:02 PM
Response to Original message
Advertisements [?]
 Top

Home » Discuss » Topic Forums » Religion/Theology Donate to DU

Powered by DCForum+ Version 1.1 Copyright 1997-2002 DCScripts.com
Software has been extensively modified by the DU administrators


Important Notices: By participating on this discussion board, visitors agree to abide by the rules outlined on our Rules page. Messages posted on the Democratic Underground Discussion Forums are the opinions of the individuals who post them, and do not necessarily represent the opinions of Democratic Underground, LLC.

Home  |  Discussion Forums  |  Journals |  Store  |  Donate

About DU  |  Contact Us  |  Privacy Policy

Got a message for Democratic Underground? Click here to send us a message.

© 2001 - 2011 Democratic Underground, LLC